Sie sind auf Seite 1von 355

.

677
http://www.cksu.com/vb/showthread.php?t=100804&page=344

*** . : 4745 << % () 633 % 53 . 4:3 4533 . . . 533 .

. . 655 . . 677 (&) . ( ) amerobida@yahoo.com ; 4766 < 5 5345 . 4:3 http://www.cksu.com/vb/showthread.php?t=100804&page=170 . ( ) . : Dental decks2 Dental Secrets - Oxford Handbook of Clinical Dentistry, 4th Edition : - Art and science of operative dentistry 2000 - Atlas Of Oral MedicineCaranza periodontology - PDQ Oral Disease Dx Tx 2002 - Pathway of the pulp 9ed 1st Contemporary Fixed Prosthodontics - Wikipedia : - 5339 - .. -

! - - . 6 ***. . . . . . . . : . . . - Dr.maestro d.somod@hotmail.com

:
a. b. c. d. 1. The periodontal tissues comprise which of the following tissues: Gingiva and the PDL. Gingival, PDL, and alveolar bone. Gingival, PDL, alveolar bone, and cementum.*** Gingival, PDL, alveolar bone, cementum, and enamel. The periodontal tissues = the periodontium http://www.link.vet.ed.ac.uk/clive/cal/dentistry/website/periodontal/pertissu es/periodontium.html 2. The periodontium comprise which of the following tissues: Gingiva and the PDL. Gingival, PDL, and alveolar bone. Gingival, PDL, alveolar bone, and cementum. *** Gingival, PDL, alveolar bone, cementum, and enamel. Periodontium - Wikipedia, the free encyclopedia The following four tissues make up the periodontium: Alveolar bone Cementum Gingiva or gums Periodontal ligament a. b. c. d. 3. The following chemically bond to the tooth: Composite resin. Dental sealants. Glass ionomer cement. *** All of the above. 4. In countries with higher annual population growth rates, the need for community based preventive programs would be greater for: Dental caries. Periodontal disease Dentofacial anomalies Dental floozies. 5. The following medical conditions may precipitate a syncope: Hypoglycemia. *** Mild hyperglycemia. Anti hypertensive drugs with ganglionic blocking agent. Anti depressant therapy. All of the above. Oxford Handbook of Clinical Dentistry - 4th Ed. (2005) page 303

e. f. g. h.

a. b. c. d. a. b. c. d. e.

A faint may mimic far more serious conditions, most of which can be excluded by a familiarity with the patient's PMH. These include strokes, corticosteroid insufficiency, drug reactions and interactions, epileptic fit, heart block, hypoglycaemia, and MI. 6. whats the first sign of Syncope a) Paleness b) nose bleeding (epistaxis( c) Miosis 7. Most frequent cause of fainting in dental office: a. Vaso-vagal shock. *** b. Diabetes. c. Fear. "dental decks 1909" vaso-vagal syncope - 8. Loss of consciousness most frequent cause: a. Syncope. b. CO2 ... "The most common cause of loss of consciousness in the dental office is syncope" 9. Orthognathic ridge relationship (class II) presents several problems which should be taken into consideration when constructing complete denture prosthesis. These include all EXCEPT: a. Require minimum interocclusal distance. *** b. Have a great range of jaw movement. c. Require careful occlusion, usually cuspless teeth are indicated. Complete Denture 17th Ed page 16 RESIDUAL RIDGE RELATIONSHIP: Class II or retrognathic is usually difficult as the patient looks toothy, often holds the mandible forward to improve appearance with subsequent TMJ problems, usually have a great range of jaw movements in function, require careful occlusion, and usually needs a large interocclusal distance. Class Ill or prognathic is usually easier if not extreme. The patient usually functions on a hinge (little or no protrusive component ) and requires a minimum of interocclusal distance. In any case, do not set the teeth for a retrognathic or prognathic patient in a normal relationship, unless there is only a moderate deviation from Class I. 10. Class III jaw relation in edentulous PT: a. It will affect size of maxillary teeth.

b. Affect retention of lower denture. c. Affect esthetic and arrangement of maxillary denture. d. All of the above.
Abstract
Further to some reminders concerning terminology and resorption, this article presents the full denture prosthetic treatment for two patients with total edentation and a prognathic malocclusion. The first case presents a tridimensional symptomatology, described by: a progeny; a voluminous mandibular terrain; a concave profile; a rather closed mandibular angle; a mandible ridge which circumscribes the upper jaw; The particularities of the prosthetic treatment in this case are: a lowering of the posterior occlusal plane; a mounting of the teeth in a limited external position, that is to say, the maxillary lingual cuspids and the mandibular mesio-distal groove on the inter-crest line; the mandibular incisors and cuspids placed with a lingual inclination to obtain an edge-to-edge occlusal contact. The second case is of vertical symptomatology type, described by: a macrogeny; a moderate concavity of the profile; a very marked increase of the vertical dimension; an open mandibular angle; a short ramus; a long mandibule; a very marked overjet between the anterior crests. The particularities of the prosthetic treatment for this case are: a posterior teeth placement, avoiding cross-bite position, taken into account the strong convergence of the intercrest lines; a strong lingual inclination of the mandibular incisors and cuspids, in order to obtain an edgeto-edge occlusal contact. In the light of these two specific cases, emphasis is placed on the particularities of the treatment and on the teeth placement proposed in order to avoid any cross-bite teeth mounting.

a. b.

a. b. c.

11. Planning centric occlusion for complete denture, it is advisable to have: 1-2 mm of vertical and horizontal overlap of upper and lower anterior teeth with no contact. *** Definite tooth contact of upper and lower anterior teeth in order to facilitate the use of anterior teeth for incision. 12. The posterior extension of max complete denture can be detected by the followings EXCEPT: Hamular notch. Fovea palatine. *** Vibrating line. "" " " .)" ( . . . "

a. b. c. d. e.

13. The distal palatal termination of the maxillary complete denture base is dictated by the: Tuberosity. Fovea palatine. Maxillary tori. Vibrating line. *** Posterior palatal seal. heartwell 5th ed - page 224 Vibrating line is determined the posterior extension of the posterior palatal seal

14. Vibrating line: a. Between hard & soft palate. b. Between mobile and non mobile soft tissue. *** 15. oral surgeon put his finger on the nose of the patient and the patient asked to blow. This done to check: a. anterior extention of posterior palatal seal. *** b. lateral extension of posterior palatal seal c. posterior extension of posterior palatal seal. d. glandular opening Dental Decks - page 440 Land marks for posterior palatal seal: The posterior outline: is formed by the "Ah" line or vibrating line and passes through the two pterygomaxillary (hamular) notchs and is close to the fovea palatine. The anterior outline: is formed by the "blow" line and is located at the distal extent of the hard palate. 16. Pt. Presented after insertion of complete denture complaining of dysphagia and ulcers what is the cause of dysphagia? -over extended. *** -over post dammed. -under extended -under post dammed : : .

. . , 17. Pt with denture has swallowing problem and sore throat. The problem is: Posterior over extension at distal palatal end. *** Over extension of lingual. Over extension of hamular notch. 18. Nausea is a complaint that a new denture wearer might encounter. It may result from: Thick posterior border. Denture under extended. Denture slightly over extended. A & b are correct. *** Complete Prosthodontics (problems,diagnosis&managment) page 78 Gagging (nausea): loose denture thick distal termination of upper denture lingual placement of upper denture occlusal plane low. Complete Denture 17th Ed page 128 3. Gagging: a. Psychogenic -starts in mind, very difficult to treat b. Stomatogenic -starts in body (usually dentures), treatable c. Dental causes: 1. Lack of retention 2. Poor occlusion 3. Insufficient or excessive palatal seal. 4. Crowded tongue due to a thick palate or poor tooth placement 5. Excessive salivation 6. Excessive vertical dimension (often seen in new dentures) And http://web.wits.ac.za/NR/rdonlyres/F...esManualV6.pdf Denture over-extension onto the soft palate may stimulate a gag reflex directly by continuous contact or indirectly by intermittent contact brought about by the activity of the soft palate or posterior third of the tongue. An under-extended denture (or an unstable denture from occlusal interferences) will lack a posterior seal, will dislodge intermittently, irritate the posterior third of the tongue and thus cause nausea.

a. b. c.

a. b. c. d.

A palpable and thickened posterior border will also irritate the tongue. Interference with tongue space, as in an excessively large vertical dimension which causes compensatory protrusion of the tongue, or in a narrow arch which forces the tongue to occupy an unnatural position, may also manifest as nausea. ) ( Protusive imbalance : . 19. After insertion of complete denture, Pt came complaining from pain in TMJ and tenderness of muscle with difficulty in swallowing, this could be due to: a. High vertical dimension. *** b. Low vertical dimension. c. Thick denture base. d. Over extended denture base. Dental Decks - page 398 20. Most common complete denture post insertion complaint after 24 hrs: a. Rough. b. Overextension causing laceration. c. Pt not used to new vertical dimension. 21. Which palatal form is more retentive and offers better stability to complete denture: a. V shaped b. Wide palate c. U shaped *** d. Flat palate 22. All relate to retention of maxillary complete denture EXCEPT: a. Tongue movement. *** b. Type of saliva. 738 . : : www.studentals.com/uploaded/2_11205876676.doc : : / :) (

. :) ( : " ) ( . 23. Best instrument to locate vibrating line with it is T burnisher. 24. We can use to palatal posterior seal: 1. Le jao carver. 2. Kingsley scraper. *** 25. Compomer restorative materials are: a. Glass ionomer with polymer components b. Resin systems with fluoride containing glasses. *** c. Composite resin for cervical restorations only. http://en.wikipedia.org/wiki/Dental_compomer The composition of compomers is similar to that of a dental composite however it has been modified, making it a polyacid-modified composite. This results in compomers still requiring a bonding system to bond to tooth tissue. And Sturdevant's art and science of operative dentistry, 4th edition, page 209 Although the name compomer implies that the material possesses a combination of characteristics of both composite and glass ionomers, these materials are essentially polymer-based composites that have been slightly modified to take advantages of the potential fluoride-releasing behavior of glass ionomers. 26. The most frequent cause of failure of a cast crown restoration is: a. Failure to extend the crown preparation adequately into the gingival sulcus. b. Lack of attention in carving occlusal anatomy of the tooth. c. Lack of attention to tooth shape, position, and contacts ***. d. Lack of prominent cusps, deep sulcus, and marginal ridges. Dental Decks - page 466

27. An examination of the edentulous mouth of an aged Pt who has wore maxillary complete dentures for many years against six mandibular teeth would probably show: a. Cystic degeneration of the foramina of the anterior palatine nerve. b. Loss of osseous structure in the anterior maxillary arch. *** c. Flabby ridge tissue in the posterior maxillary arch. d. Insufficient inter occlusal distance. Dental decks - page 390 When a patient wears complet maxillary denture agansit the six mandibular anterior teeth its very common to have to do a reline so often de to loss of bone strucutrein anterior maxillary arch. 28. Dental carries is an endemic disease "means that the disease is: a. Occurs clearly in excess of normal expectancy. b. Is habitually present in human population. c. Affect large number of countries simultaneously. d. Exhibit a seasonal pattern. 29. Pt on treatment with steroids are placed on antibiotic after oral surgical procedure because: a. The Pt is more susceptible to infection. b. Antibiotics are synergistic to steroids. c. Antibiotic inhibits kerksheimer reaction. d. Antibiotic protect the Pt from steroid depletion. 30. The post operative complication after the removal of impacted third molar is: a. Secondary hemorrhage. b. Swelling. c. Pain. d. Alveolar osteitis. e. All of the above. 31. If the oral tissues are inflamed and traumatized, impression for making a new denture: a. Should be started immediately in order to prevent further deterioration. b. The occlusion of the existing denture is adjusted, and tissue condition material is applied, and periodically replaced until the tissue are recovered, then making impression take place. *** c. The Pt is cautioned to remove the denture out at night. d. A & B are correct. e. All of the above are correct. 32. Balanced occlusion refers to: a. The type of occlusion which allows simultaneous contact of the teeth in centric occlusion only.

b. The type of occlusion which allows simultaneous contact of the teeth in centric and eccentric jaw positions.*** c. A type of occlusion which is similar to the occlusion of the natural teeth. Dental Decks - page 548 33. Polyether impression materials: a. Are less stable dimensionally than poly sulfide rubber. b. Are less stiff than poly sulfide rubber. c. Can absorb water and swell if stored in water. *** . 34. The indication for the use of lingual plate major connector include: a. For the purpose of retention. b. When the lingual frenum is high or when there is a shallow lingual sulcus. c. To prevent the movement of mandibular anterior teeth. d. All of the above. *** Dental decks 641 . . - : 35. Lingual plate: a. Shallow sulcus*** b. Mobile anterior teeth*** c. Deep sulcus d. A+b *** e. All of above Dental decks 641 36. . Lingual bar contraindication except : a- shallow lingual sulcus b- long lingual frenum c- too crowded lower anterior teeth*** d-Mobile anterior teeth 37. . Lingual bar indication: a- short lingual frenum b- deep lingual sulcus c- too crowded lower anterior teeth d-all of the above ***

a. b. c. d.

. . : 38. In class I partially edentulous lower arch, selection of major connector depend on: a. Height of lingual attachment. b. Mandibular tori. c. Periodontal condition of remaining teeth. d. All of the above. *** 39. An anterior fixed partial denture is contraindicated when: Abutment teeth are not carious. An abutment tooth is inclined 15 degrees but otherwise sound. There is considerable resorption of the residual ridges. *** Crown of the abutment teeth are extremely long owing to gingival recession. Dental Decks - page 452 48 . Dental decks 640 - - - : . - -

40. In registering the vertical dimension of occlusion for the edentulous patient. The physiological rest dimension: a. Equals the vertical dimension of occlusion. b. May be exceeded if the appearance of the patient is enhanced. c. Is of little importance as it is subject to variations. d. Must always be greater than vertical dimension of occlusion. *** Dental Decks - page 496 41. Three weeks after delivery of a unilateral distal extension mandibular removable partial denture, a Pt complained of a sensitive abutment tooth, clinical examination reveals sensitivity to percussion of the tooth, the most likely cause is: a. Defective occlusion. *** b. Exposed dentine at the bottom of the occlusal rest seats. c. Galvanic action between the framework and an amalgam restoration in the abutment tooth. Dental Decks - page 618

42. PT with lower complete denture, intraoral examination show with slightly elevated lesion with confirmed border, PT history of ill fitting denture. It is by: a. Immediate surgical removal. b. Instruct PT not to use denture for 3 weeks then follow up. *** c. Reassure PT and no need for treatment. 43. Examination of residual ridge for edentulous PT before construction of denture determine stability, support and retention related to the ridge: a. True. *** b. False. 44. Upon examination of alveolar ridge of elderly PT for construction of lower denture easily displaceable tissue is seen in the crest of ridge. Management: a. Minor surgery is needed. b. Inform the PT that retention of denture will decrease. c. Special impression technique is required. *** "Questions and Answers " : . 45. In recording man-max relation,the best material used without producing pressure is: a. Wax. b. Compound. c. Bite registration paste (zinc oxide & eugenol paste). *** 46. The posterior seal in the upper complete denture serves the following functions: a. It reduces Pt discomfort when contact occurs between the dorsum of the tongue and the posterior end of the denture base. b. Retention of the maxillary denture.*** c. It compensate for dimensional changes which occur in the acrylic denture base during processing. d. B & C are correct. 47. function of post dam: a. Prevent tongue from palate touch increase comfort b. increase retention*** c. to compensate acrylic dimension d. all 5339 :

. 48. In recording jaw relation, best to use: a. Occlusal rim with record base. *** b. Occlusal rim with base wax. c. Occlusal rim with nacial frame. Dental Decks - page 428 Occlusal rims: make maxilla-mandibular jaw records. 49. during 3/4 crown preparation on premolar, bur used to add retentive grooves is: a. radial fissure b. Tapered fissure 50. The goal of construction of occlusion rims is: 1. To obtain the occlusal plane, vertical dimension, tentative centric relation, face low transfer, placement of the teeth. 2. To obtain the protrusive condylar guidance. 3. To obtain the lateral condylar posts and incisal guide. 4. None. 51. A temporary form representing the base of a denture which is used for making maxillo-manibular (jaw) relative record for arranging teeth or for trail insertion in the mouth is: 1. Bite rims. 2. Custom tray. 3. Set up. 4. Base plate. *** 52. To recheck centric relation in complete denture: a. Ask PT to w and close. b. Ask PT to place tip of tongue in posterior area and close. *** c. To wet his lip and tongue. d. All of the above. 53. Recent years, there has been an evidence that the prevalence and intensity of the caries attack has been diminishing in the more economically developed countries, mainly because of the wide spread use of: a. Artificial water fluoridation. ***

b. Fluoride toothpaste c. Dental health education programs d. A & c. %93-83 dental deck pedo 2004 47; 54. Cost effective method to prevent dental caries water fluoridation. *** flouridated tooth paste Dental Decks - page 1664 55. In recent years caries reduced in developed countries mainly due to: a. Water fluoridation. *** b. Fluoride toothpaste c. Dental health education programs. D. Individualized oral hygiene care. 56. In terms of caries prevention, the most effective and most cost effective method is: a. Community based programs. b. Individually based programs. 93 . One-to-one in the clinical environment. This is usually the most successful approach, because the message can be tailored to the individual and reinforcement is facilitated. However, it is expensive in terms of manpower. 57. In terms of caries prevention, the most effective and most cost effective method is: a- Community based programs. b- Private based programs 58. The effects of natural fluoride versus added fluoride in reducing dental caries as it relates to the concentration are: a. Greater b. Less *** c. The same . 59. When does child should be first exposed for using tooth brush:

a. As eruption of first tooth. *** b. One year old. c. Two years old. d. Primary school year. 60. When a child must first exposed to the use of the tooth brush: a-Of age of 2 years. b-Of age of 4 years. c-Immediately after eruption of first tooth. *** 61. When a child must first exposed to the use of the tooth brush: a-Of age of 2 years. b-Of age of 4 years. c-after eruption of primary teeth. *** Oxford Handbook of Clinical Dentistry - 4th Ed. (2005) - page 25 Brushing using a fluoride toothpaste should start as soon as the first teeth erupt (about 6 months of age). Parents should supervise brushing up to at least 7 yrs of age to avoid over-ingestion of toothpaste and ensure adequate plaque removal. 62. When tooth paste is used the child is advised a- not swallow b- swallow a small amount*** c- do not rinse d- none )( 63. Fluoridated toothpaste for 3 years child is a. recommended *** b. Not recommended c. Common d. Non of above Fluoridated toothpaste for 3 years child is recommended but under supervision in small pea-sized amount National Fluoride Information Centre - Guide to Fluoride 64. Fluoride which we use in the clinic doesnt cause fluorosis because: a. It's not the same fluoride that cause fluorosis.

b. Teeth already calcified. *** c. Calcium in the mouth counter. d. Saliva wasnt out. Dental Decks - page 2229 65. Mentally ill Child The best way to apply fluoride a. acidulated phosphate fluoride b. Natural Sodium Fluoride c. Fluoride varnish*** d. Stannous fluoride.

66. Fluoride decrease dental caries by remineralization of enamel: a. True. *** b. False. . 67. Person drinking fluoridated water, using toothbrush with fluoride, rinsing with fluoride mouthwash, then no need to put pit and fissure in his permanent teeth: a. True. b. False. *** 68. Dental fluorosis: a. Is indicative of systemic fluorosis. *** b. Can be contracted at any age. c. Becomes less noticeable with age. d. Is reversible. e. Is largely preventable. 69. Fluorides are most anticaries effective when: a. Incorporated in the tooth enamel. *** b. Present in the blood stream. c. Present in the plaque and tissue fluids bathing the newly erupted tooth. d. Present in the ingested foods. e. Present on the intraoral mucous membranes. 70. Fluoride is not taken up systemically from which of the following sources: a. Water. b. Food. c. Dentifrices. d. Topical applications of fluoride. *** 71. Fluoride reduces caries activity by: a. Reduces bacterial adhesion and carbohydrate storage (antimicrobial

activity). b. Enhances the precipitation of insoluble fluoroapitite into the tooth structure. c. Fluoride enhances remineralization of the noncavitated carious lesions. d. All of the above. E. B & C. *** Dental decks 2228 Dental decks 2232 . 72. Communities with high annual population growth need education about: a. Caries b. Perio disease c. Dentofacial anomalies d. Dental fluorosis 73. Optimawater fluoridation: a) 1-1.5 mg\liter. *** b) 0.2-0.5 mg\liter. C) 2-3 mg\liter 74. Optimawater fluoridation: a) 1 mg\liter. *** b) 0.2-0.5 mg\liter. C) 2-3 mg\liter d) 0.5 mg\liter 75. Optimawater fluoridation: a) 0.5 0.8 mg\liter. *** b) 0.2-0.5 mg\liter. C) 2-3 mg\liter d) 1-5 mg\liter 78< : . . 4,5-3,: Dental Decks - page 2230 76. Acidulated phosphat flouride a-1% fluoride ions b-1.23% *** c-2% d-2.23% f-3% Dental Decks - page 2230

77. Actual destruction of micro-organisms in the root canal is attributed mainly to: a. Proper antibiotic thereby. b. Effective use of medicament. c. Mechanical preparation and irrigation of the canal. *** d. None of the above. 78. A tooth very painful to percussion, doesnt respond to heat, cold or the electric pulp tester. The most probable diagnosis is: a. Reversible pulpitis. b. Irreversible pulpitis. c. Acute apical periodontitis. *** 79. During post insertion examination of a 3 unit ceramometal fixed partial denture. One of the retainers showed chipping of porcelain at the ceramometal junction. In order to avoid the problem the dentist must: a. Reduce the metal to 0.3 mm. b. Have uniform porcelain thickness. c. Have occlusion on metal. d. Keep porcelain metal junction away from centric contacts. *** () . 4.8 80. What is a Pier abutment? a. Single tooth holding one pontic. b. A tooth that supports a removable partial denture. c. All of the above. d. None of the above. *** . . . 81. Which are the ways in which the proximal contacts can be checked? a. Use a pencil. b. Use a shim stock. c. Use a silicone checker. d. Use a dental floss. e. Only b & d. *** .

Journal of Oral Rehabilitation Volume 14 Issue 1, Pages 91 - 94 A total of 969 proximal contacts in forty volunteer subjects was examined for proximal contact integrity with shim stock. It passed uninhibited through 88% of the contacts. Neither sex nor age affected the evaluation; however, shim stock was more likely to pass through contacts with enamel surfaces than those which were restored 82. The incisal reduction for a metal ceramic restoration should be: a. 1.5 mm. b. 2 mm. *** c. 3 mm. d. 4 mm. "Contemporary Fixed Prosthodontics" 83. The occlusal reduction for an all metal veneer crown should: a. Be as flat as possible to enable an easy fabrication of occlusion anatomy. b. Follow the occlusal morphology with a clearance rating from 1 to 1.5 mm with the opposing dentition. *** c. Follow the occlusal morphology with a clearance of no more than 0.5 mm with the opposing dentition. d. Be the last step in the tooth preparation. 4.8-4 . . 4.8-4 3.8 84. Gingival retraction is done: a. To temporarily expose the finish margin of a preparation. b. To accurately record the finish margin of a portion of uncut tooth surface apical to the margin in the final impression. c. Even in the presence of a gingival inflammation. d. By various methods but the most common one is the use of retraction cord. e. A and b. f. A, b and c. g. A, b and d. *** 85. Regarding tissue retraction around tooth: a. Short duration of retraction of gingival margin during preparation of finishing line. b. Retraction of gingival margin during taking final impression to take all details of unprepared finish line. c. Usually retracted severely inflamed gingival margin.

d. Retraction of gingival margin can be done by many ways one of them is retraction cord. e. A, b and c. f. B, c and d. g. A and d. *** 86. Loose enamel rods at the gingival floor of a class II amalgam cavity should be removed using : a. Straight chisel. b. Hatchet. c. Gingival curetla. d. Gingival marginal trimmer. *** Dental Decks - page 2180 Gingival marginal trimmers are primarily used for beveling gingival margins, and rounding or beveling of the axiopulpal line angle of Class II preparations. . Art and science of operative dentistry 2000 page 315 The gingival margin trimmer is designed to produce a proper bevel on gingival enamel margins of proximoocclusal preparations. 87. Removal of Undermined Enamel in Class II cavity is done by : A) Chisel. *** B) Angle former C) Excavator Oxford Handbook of Clinical Dentistry - 4th Ed. (2005) page 145 PROXIMAL (CLASS II): A chisel can be used to plane away unsupported enamel from the margins of the completed preparation to produce a 90 butt joint. 88. Hand instrument which we used to make internal angles retentive grooves and preparation of cavity walls in the cavity is: a. Angle former. *** b. Chisel c. File d. Enamel hatched Art and science of operative dentistry 2000 page 314 A special type of excavator is the angle former. It is used primarily for sharpening line angles and creating retentive features in dentin in preparation for gold restorations. FUNDAMENTALS OF OPEERITIVE DENTISTRY page 318 Retention grooves are placed with a No 1/2 or 1/4 bur.

( ) Dental Decks - page 2180 82 - 84

89. What is the cavo-surface angle of prep for amalgam restoration: a. 30 degree b. 60 degree c. 90 degree *** d. 130 degree. : Principles of OPERATIVE DENTISTRY .< 3-:3 . Dental Decks - page 2310 90. To provide maximum strength of amalgam restoration the cavo-surface angles should: 1. Approach 75 with outer surface. 2. Approach 90 with outer surface. 3. Be supported by sound dentine. 4. Be located in area free of occlusal stress. a) 1+3 and 4. b) 1+3. c) 2+3+4. ***

d) 3+4. 91. Which of the following materials has been shown to simulate reparative dentine formation most effectively when applied to the pulpal wall of a very deep cavity: a. Copalite varnish. b. Calcium hydroxide preparation. *** c. Zinc phosphate cement. d. Anhydrous class inomer cement. 92. Calcium hydroxide is best pulp capping material because: 1. It has best seal over pulp. 2. It is alkaline + less irritating to pulp. 3. It induces reparation dentine formation. *** 93. The most accurate impression material for making the impression of an onlay cavity: a. Impression compound. b. Condensation type silicone. c. Polyvinyl siloxane *** d. Polysulfide. 94. patient came to dentist after previous stressful procedure complaining of burning & discomfort of his lip on examination u found lesions on the palate, diagnosis is: - contact dermatitis - allergy - aphthous ulcer - herpes simplex (herpetic gingivostomatitis) *** 95. a. aphthous ulcer b. recurrent herpes ulcer*** c. allergic stomatitis 96. Aphthous ulcer, compared with herbes ulcer is: a. More characteristic in histology. b. Leaves scar. c. Less response to stress. d. Occur in lining mucosa. *** Dental Decks - page 1552 The following picture for Aphthous ulcer(canker sores)

97. Syphilis first appearance: a. Multiple vesicle. b. Erythematous reaction. c. Ulcer. *** d. Bullae. 98. child with vesicle on the hard palate with history of malaise for 3 days what is the possible diagnosis: 1/ herpes simplex. *** 2/ erythema multiform 473

99. Which virus is present in the patient's mouth all his Life? a. Herpes Simplex b. Herpes zoster

c. Varecilla Virus d. None of the above *** Herpes Simplex - Symptoms, Treatment and Prevention Cold sores are caused by the Herpes Simplex Virus. Once infected, they plague the patient for life. Herpes can be treated but not cured. Symptoms appear briefly and then disappear; the disease lies dormant in nerve cells, but it may be reactivated by stress or illness. Shingles Shingles is caused by the varicella zoster virus, which also causes chickenpox. If you have had chickenpox, the varicella virus remains in a group of nerves in your central nervous system, but doesn't cause any symptoms. This is called a dormant virus. The central nervous system consists of the brain and spinal cord, which are connected to the nerves in the body. When the virus becomes active again, it causes the symptoms of shingles. eMedicine - Varicella-Zoster Virus : Article by Wayne E Anderson Varicella-zoster virus (VZV) is the cause of chickenpox and herpes zoster (also called shingles). )HSV1( Herpes Simplex :) ( 100. Clinical failure of the amalgam restoration usually occurs from: a. Improper cavity preparation *** b. Faulty manipulation. c. Both of the above d. None of the above 101. It has been proven that amalgam restoration has the following characteristics: a. Micro leakage decrease with aging of the amalgam restoration. b. It is the least techniques sensitive of all current direct restorations. c. High dimensional changes. d. A, b and c. e. A and c. f. A and b. *** g. B only. Art and science of operative dentistry 2000 page 156 - Page 169 - During electrochemical corrosion of low-copper amalgams, The Sn-Hg phase is oxidized into Sn-O and/or Sn-O-Cl. The oxychloride species is soluble. The

oxide Precipitates as crystals and tends to fill up the spaces Occupied by the original Sn-Hg phase. Along the margins Of the amalgam, Sn-O helps seal the space against Microleakage. - During setting, most amalgams undergo very little Dimensional change. Http://www.dentaldiamond.ee/dental-materials/amalgam-dentalamalgam/4/ - The dimensional change during the setting of amalgam is one of its most characteristic properties. Modern amalgams mixed with mechanical amalgamators usually have negative dimensional changes. - The only exception to this statement is the excessive delayed dimensional change resulting from contamination of a zinc-containing alloy with water during tritura-tion or condensation. Dental Decks - page 2308 102. When polishing the amalgam restoration: a. Avoid heat generation by using wet polishing paste. b. Wait 24 hours. c. A and b. *** d. B only. e. A only. Dental Decks - page 2304 103. Maximum time elapsed before condensation of amalgam after titration: a. 1minute. b. 3minutes. *** c. 9minutes. : 104. After amalgam titrations, the mix should be placed within: a. 1 min. *** b. 3 min. *** c. 5 min. d. 10 min. 105. MOD amalgam restoration with deep mesial box, PT come with pain related to it after 1 month due to: a. Pulp involvement. *** b. Supraocclusion. c. Upon contact. d. Gingival recession.

a. b. c. d.

106. Reduction in amalgam restoration should be: 1-1.5 mm. 1.5-2 mm. *** 2-3 mm. 3-5 mm. Art and science of operative dentistry 2000 page 659 "it must have a minimum thickness of 0.75 to 2 mm (because of its lack of compressive strength)"

107. Depth of amalgam restoration should be: 1. 1 1.5 mm. 2. 1.5 2 mm. *** 3. 2 3 mm. 4. 3 5 mm. 108. Silicate cement: a. First tooth colored restoration. b. It can be used as permanent filling. c. It contains 15 % fluoride. d. A, b and c. e. 1 and 2. f. A and c. *** g. A only. Sturdevant's art and science of operative dentistry page 475 Silicate cement , the first translucent filling material, was introduced in 1878 by Fletcher in England dental material & thier selection2002 58-45 Dental Decks part2 2007-2008 - page2066 ZOE,reinforced ZOE, ZOE-EBA, Silicate and zinc phosphate cements are no longer routinely used to permanently cement restorations 109. Treatment of gingival trauma from faulty oral hygiene is mainly: a. To advice the patient to change their faulty habits immediately *** b. Reassure the patient that it will disappear by it self. c. To buy a new brush. 110. Which of the following statement is true regarding dental calculus: a. It is composed entirely of inorganic material. b. It is dens in nature and has a rough surface. c. It is mineralized dental plaque. d. All of the above.

e. B & C only. *** f. None of the above. . () 111. Overhanging restoration margins should be removed because: a. It provides ideal location for plaque accumulation. b. It tears the gingival fibers leading to attachment loss. c. Stimulate inflammatory reaction directly. d. Its removal permits more effective plaque control. e. A & d. *** 112. Main use of dental floss a. Remove calculus. b. Remove over hang. c. Remove bacterial plaque. *** d. Remove food debris. the goal of flossing your teeth is to scrub dental plaque it can be done in primary and permanent 113. What is the benefit of rinsing the mouth with water: A) Plaque removal B) calculus removal C) washing the food debris. *** 114. What is the benefit of rinsing the mouth with water a. Plaque removal b. Prevent the formation of plaque c. Dilute the concentration of bacteria 115. The water reins devices for periodontal therapy has a main goal which is a-remove plaque b-prevent plaque attachment c-dilute bacterial toxin d-remove dental pocket 116. One of these is less exposed to extensive dental caries: a- Obes, malnourished b- Pt has xerostomia c- Less plaque score. *** 117. Calculus induce further periodontal lesion due to: a) Directly stimulates inflammation

b)more plaque adhere to it. *** c) irritate the gingiva 118. Missing lower six and tilted 7 a- Uprighting of molar by orthodontics. b- Proximal half crown. c- Telescope crown. d- Non rigid connector. *** Dental Decks - page 492 None rigid connector: it is indicated when retainers cannot be prepared without excessive tooth reduction. Oxford handbook, 3rd edition, page 303 . " " " "

119. Floss used to: a. Remove interproximal plaque. *** b. Remove overhangs c. Stimulate gingival d. . 120. Plaque consists of: a. Bacteria b. Inorganic material c. Food 121. To prevent perio problem MOST effective method is: a. Community program. b. Removal of plaque. *** c. Patient education. d. Water floridation 122. Length of pins must be equals in both tooth and restoration by a depth of: a. 1 mm. b. 2 mm. *** c. 3 mm.

d. 4 mm. 5 . 123. Stainless steel pin is used in amalgam for: a. Increase retention. *** b. Increase resistance. c. Increase streangth. d. A and b. . Dental decks 2210 124. Calcium channel blockers cause increase saliva secretion. a. True. b. False. *** 125. RCT contraindicated in: a. Vertical fracture of root. *** b. Diabetic Pt. c. Periodontally involved teeth. 126. What can we use under composite restoration: a. Ca (oh). b. ZOE. c. ZINC phosphate cement. d. A and C *** 127. Gutta percha contain mainly: a. Gutta percha 20%. b. ZINC oxide %. *** c. ZINC phosphate. .transpolyisoprene, barium sulfate, zinc oxide : 128. Single rooted anterior teeth has endodontic treatment is best treated by: a. Casted post and core. *** b. Performed post and composite. c. Performed post and amalgam. d. Composite post and core 129. Post fracture decrease with prefabricated post ready made post casted post. ***

metal post 130. Teeth with RCT and you want to use post, which post is the least cause to tooth fracture : 1. Ready made post. 2. Casted post. 3. Fiber post. *** 4. Prefabricated post. 131. During post removal the first thing to do is: A) remove the G.P B) remove all the old restoration & undermined enamel & caries. *** C) insertion of post immediately 132. For root canal treated tooth u choose to put post & amalgam this depend on remaining coronal structure *** root divergencepresence of wide rootothers 133. Post length increasing will .increase retention. *** .increase resistant .increase strength of restoration 134. For post preparation we should leave mm of GP: a. ???? b. 10mm c. 5mm. *** : Http: //www.experts123.com/q/how-much...placement.html Post and core - Wikipedia, the free encyclopedia 135. Which of the following endodontic failure may be retreated only with surgery: a. Missed major canal. b. Persistent interappointment pain. c. Past and core. *** d. Short canal filling.

136. Which of the following failure may be treated nonsurgically: a. Post filling that has removed. *** b. Severe apical perforation. c. Very narrow canal with a periapical lesion and the apex can not be reached. d. None of the above. 137. In post and core preparation must: a. Extend to contrabevel. b. Extend to full length tooth preparation. c. Take same shape of natural tooth. d. Take shape of preparation abutment. e. A & d. *** f. A & b. g. D & c. h. B & c. .

138. After RCT, for insertion of post dowel: a. Post applied pressure. b. Post should be lose. c. Insert it without pressure but with retention. *** 139. Amount of G.P should after post preparation: a. 1 mm. b. 4-5 mm. c. 10 mm. d. None of the above. Oxford Handbook of Clinical Dentistry - 4th Ed. (2005) page 154 As a general guide the post should be at least equal to the anticipated crown height, but a minimum of 4 mm of well-condensed GP should be left. A periodontal probe is helpful to check prepared canal length. 140. Post retention depends on: a. Post length. b. Post diameter. c. Post texture. d. Core shape. e. Design of the preparation. f. A and b

g. A, b and c. h. All of the above. - 7 : 4 .

a. b. c. d.

141. The best restoration for max central incisor that has received RCT through conservatively prepared access opening would be: Post-retained metal-ceramic crown. Post-retained porcelain jacket crown. Composite resin. *** None of the above.

142. One of anatomical land mark is: a. Ala tragus line. *** b. Ala orbital. c. Frank fort plane. . 143. The PH of the calcium hydroxide is: a. 7.2 b. 12 *** c. 19 d. 5.5 hydroxide was first introduced as a pulp capping agent in 1930 by Hermann and since then its use in endodontic therapy has increased. the calcium hydroxide dressing may both prevent root canal re-infection and interrupt the nutrient supply to the remaining bacteria. Its alkalizing pH (around 12.5) promotes a destructive effect on cell membranes and protein structure Brazilian Dental Journal - In vitro antimicrobial activity of calcium hydroxide pastes and their vehicles against selected microorganisms Dycal (pH of 11) and Pulpdent (pH 12) 144. Hyperemia result in: a. Trauma of occlusion. b. Pain of short duration.

c. Radiographic changes. d. All of above. 145. Tooth had trauma resulted in hyperemia? a) pain increased with cold b) reverible condition c)pain of short duration d) B and c e) all 146. The x- ray of choice to detect the proximal caries of the anterior teeth is: a. Periapical x-ray. b. Bitewing x-ray. c. Occlusal x-ray. d. None of the above. ) ( 147. In primary teeth, pathologic changes in radiographs are always seen in: a. Periapical area. b. Furcation area. *** c. Alveolar crest. d. At base of developing teeth. Dental Decks - page 1534

148. Pulpities in decidous teeth in radiograph see related to furcation. *** apex of root lateral to root 149. In deciduous tooth the first radiographic changes will be seen in: 1. Bifurcation area. 2. Apical area. 3. External root resoption. *** " ." 150. Eruption cyst "eruption hematoma" can be treated by: a. No treatment. *** b. Immediate incision.

c. Complete uncoverage d. Observe for one week then incise Oral pathology clinical pathologic correlation,3rd edition, Page 296 No treatment is needed because the tooth erupt through the lesion.

151. After trauma a tooth become yellowish in color, this is due to: a. Necrotic pulp. b. Irreversible pulpitis. c. Pulp is partially or completely obliterated. *** d. Hemorrhage in the pulp. Endo Principles and Practice of Endodontics WALTON page 45 A yellowish discoloration of the crown is often a Manifestation of calcific metamorphosis 152. Step deformity of the Mandibular body fracture may due to: a. Forward pull of lateral pterygoid muscle. b. Upward pull of masseter and temporalis. *** c. Toward pull of medial pterygoid muscle. d. Downward pull of geniohyoid and myalohyoid. 153. What is the copper ratio that eliminates gamma phase 2: a. 2% copper b. 4% copper c. 10 % copper d. 13 % copper *** 154. to prevent discoloration under amalgam filling: a- use Zn phosphate box b- use cavity varnish c- wash the cavity with NaOCL b4 filling d- use the correct amalgam-alloy ratio 155. Inorganic material in bone compromise: a. 65%. *** b. 25% c. 10% d. 95% 156. Polishing bur have:

a. b. c. d.

Less than 6 blades. 6-7 blades. 10-12 blades. More than 12 blades. .53-45

157. Pain during injection of local anesthesia in children could be minimized by: a. Slowly injection. b. Talking to the child during injection. c. Using long needle. d. A and b. *** 158. Rubber dam is contraindicated in: a. Pt with obstructive nose. *** b. Mentally retarded Pt. c. Un comparative child. d. A and b. 159. With children rubber dam not use with: - Hyperactive patient - obstructive nose. *** - patient with fixed orthoappliance - mildly handicapped and uncooperative. 160. The most common type of biopsy used in oral cavity is: a. Excisional biopsy. *** b. Incisional biopsy. c. Aspiration through needle. () d. Punch biopsy. 161. In maxillary 1st molar 4th canal is found in: a. MB canal. *** b. DB canal. c. Palatal root. 162. Formicrisol when used should be: a. Full Saturated. b. Half saturated. c. Fifth saturated. *** d. None of the above. Pediatric dentistry ,infancy through adolescence,pinkham, 3rd edition, : page348

163. 10 years old child present with bilateral swelling of submandibular area, what could be the disease: a. Fibrous dysphasia. b. Cherubism *** c. Polymorphic adenoma. White and pharaoh,oral radiology principles and interpretation,4th,462 Cherubis.the most common presenting sign is a painless,firm,bilateral enlargement of the lower face. It develops n early child hood 164. Pt complain from pain in 45 whitch had gold onlay. The pain could be due to: Chemicals from cement. High thermal conductivity of gold. *** Related to periodontal ligament. Cracked tooth or fractured surface. Dental Decks - page 2134 Disadvantages of gold restoration: Esthetics cost time consuming difficulty of technique the need to use cement (the weakest point in the cast gold restoration) gold has high thermal conductivity. 165.

a. b. c. d.

a. Related to periodontal ligament. 166. The irrigation solution is good because: a. Lubricate the canals. b. Flushes the debris. c. None of the above. d. All of the above. *** http://medind.nic.in/eaa/t03/i2/eaat03i2p19.pdf Root canal irrigants are used during biomechanical preparation of the root canal to remove the organic/inorganic debris, lubricate endodontic instruments and minimize the number of micro-organisms. 167. Which is most common: a. Cleft lip.

b. Cleft palate. c. Bifid tongue. d. Cleft lip and palate. *** : . 4: - 47 83 ; 4 (most common & most complicated) (Source: Petersons page 841) Clefts of the upper lip and palate are the most common major congenital craniofacial abnormality. Atlas of Oral Diseases in Children Cleft lip and palate ar more common together than is cleft lip alone. 168. Which cranial nerve that petrous part of temporal bone houses: a. Trigeminal n V. b. Facial n VII. *** c. Vagus n IX. ) ( d. Vestibalcochealer n VII. 169. 21 years old Pt with pathological exposure in 35. Management: a. Direct pulp capping. b. Indirect pulp capping. c. Root canal treatment. *** 170. Hypercementosis: a. Occur in Pajet disease. b. Difficult to extract. c. Bulbous root. d. Easy to manage by elevator. e. A and b. f. A and d. g. All the above. *** ) ( 171. Hypercementosis: a. Occur in Pajet disease. b. Difficult to extract. c. Bulbous root. d. Easy to extract by elevator. e. A and b. f. A and d. g. A + b + c *** h. All the above.

Dental secrets page 256 Hypercementosis increases the difficulty of tooth removal. Dental secrets page 113 If hypercementosis is present, t he periodontal ligament space is visible around the added cementum; that is, the cementum is contained within and is surrounded by the periodontal ligament space. Condensing osteitis, by contrast, is situated outside the periodontal ligament space.

Enlarged root of tooth no. 29, particularly in the apical area. The root of tooth no. 28 also shows some widening. The periodontal ligament space surrounds the tissue that has been laid down, and the lamina aura is visible outside the Extraction sockets. The appearance of a healing or ket may present a problem. The ssistance in distinguishing between the two include the density of e or absence of a canal, and

Worth HM: Principles and Practice of Oral Radiologic Interpretation. Chicago, Year-Book, 1963, pp periodontal ligament space. B, An opacity situated outside the periodontal ligament 172. For onlay preparation, reduction of functioning cusp should be: a. 1.5 mm. *** b. 2 mm. c. 1 mm. "Contemporary Fixed Prosthodontics ROSENSTIEL" + Dental Decks - page 2122 4 4.8 173. Thickness of porcelain should be: a. 03-05 mm. b. 0.05-0.15 mm. c. 0.5-1.5.mm. *** Dental Decks - page 442 The necessary thickness of the metal substucture is 0.5 mm the minimal porcelain thickness is 1.0 - 1.5 mm 174. Thickness of porcelain should be: a. 0.3-0.5 mm. b. 0.05-0.15 cm.*** c. 0.5-1.5 cm. *** 175. Class II composite resin is lined by: a. G.I. *** b. Reinforced ZOE. c. ZOE with epoxy cement. d. Cavity varnish. 176. Occlusal plane is: a. Above the level of the tongue. b. Below the level of the tongue. *** " mcqs in Dentistry" : " the tongue rests on the occlusal surface" 177. Lateral pterygoid muscle has how many origin: a. 1. b. 2. *** c. 5.

d. 7. . 178. Embryo become fetus in: st a. 1 week b. 1st month. c. 2nd month. d. 3rd month. *** 179. All are single bone in the skull EXCEPT: a. Lacrimal. *** b. Occipital. c. Sphenoid. d. Parietal.

180. The scientific evidence in dictating that oral Lichen planus is a "premalignant Lesion" is: a. Very strong b. Non-existent c. Moderately strong d. Weak. ***

- The premalignant potential of oral lichen planus and the malignant transformation rate is cited as 0.4-3.3 per cent. Whether or not patients with atrophic or erosive forms of oral lichen planus are more susceptible to malignant change has yet to be proved by long-term prospective studies. And - Good prognosis; rare malignant transformation (0.53%) a. b. c. d. e. 181. Oral lesions of lichen planus usually appear as: White striae. *** Red plaque. Shallow ulcers. Papillary projections. Builae. Dental Decks - page 1340 Cawson Essintials of Oral Pathology and Oral Medicine 7th ed Striate pattern . a. b. c. d. e. 182. The oral lesions of the lichen planus: Are usually painful. Rarely appear before lesion elsewhere on the body. May be part of a syndrome in which lesions also appear on the skin, conjunctiva and genitalia. *** Often appear in nervous, high-strung individuals. Heals with scarring. PDQ Oral Disease Dx Tx 2002 page 22 Skin sites: forearm, shin, scalp, genitalia. Etiology: Unknown Autoimmune T cellmediated disease targeting basal keratinocytes. Lichen Planus Variants: reticular (most common oral form) erosive (painful) - atrophic, papular, plaque types; bullous (rare) Cawson Essintials of Oral Pathology and Oral Medicine 7th ed . .

a. b. c. d. e.

183. All of the following are oral features of acquired immunodeficiency syndrome AIDS EXCEPT: Candidiasis. Erythema multiform. *** Hairy leukoplakia. Rapidly progressing periodontitis. Kaposi's sarcoma.

184. Hairy trichoglossia may be caused by: a. Broad spectrum antibiotic. b. H2o2 mouth wash. c. Systemic steroid. d. Heavy smokers. e. All of the above. *** " DENTAL SECRETS Second Edition " : .

185. In hairy tongue, which taste buds increase in Length: a. Fili form. *** b. Fungi form. c. Foliate. d. Circumvallates. Dental Decks - page 1337 186. Coronal suture is between: a. Occipital and temporal bone. b. Frontal and parietal bone. *** c. Occipital and tympanic bone.

187. During instrumentation, sudden disappear of root canal due to: a. Bifurcation of main canal. ***

b. Apical perforation. c. Calcification. Dental decks 154 . 188. Space loose occur in: a. Proximal caries. b. Early extraction. c. Ankylosis. d. All of the above. *** 189. What is the number of pharyngeal "brancheal" arches: a. 4. b. 5. c. 6. *** d. 7. " Anatomy of the Human Body" : 190. What is the name of first pharyngeal "brancheal" arches: a. Maxillary. b. Mandibular. *** c. Thyroid d. hyoid Hyoid . 191. Stomodeum and fugi separated by: 1/buccopharyngeal arch 2/ectodermal cleft http://en.wikipedia.org/wiki/Stomodeum The stomodeum is lined by ectoderm, and is separated from the anterior end of the fore-gut by the buccopharyngeal membrane. 47-46 ) ( stomodeum .buccopharyngeal membrane

. 192. In cavity preparation, the width of the cavity is: a. 1/2 inter cuspal distance. b. 1/3 inter cuspal distance. *** c. 2/3 inter cuspal distance. 193. Polyvinyl siloxanes compared with polysulfide: a. Can be poured more than once. b. Can be poured after 7 days. c. Less dimensional stability. d. A and b. *** 194. the most accurate impression material for making impression of an oral cavity is: a- impression compound. b -condensation type silicon. c- polyvinyl siloxanes. *** d- poly sulfide. http://www.medicaljournal-ias.org/7_1/Keyf.pdf the addition silicones are the best choice of the rubber impression materials Addition curing silicones have the least amount of shrinkage on setting making them the most accurate class of rubber impression material (9). The poly(vinylsiloxanes) are characterized by excellent dimensional accuracy and long-term dimensional stability. 195. Hand over mouth technique is used in management of which child: a. Mentally retarded. b. Positive resistance. c. Uncooperative. d. Hysterical. *** 196. Pits and fissure sealants are indicated in: a. Deep Pits and fissure. b. Newly erupted teeth. c. A and b. *** . 197. Pit and fissure sealants are indicated to prevent dental caries in pits and fissure:

a. In primary teeth b. In permanent teeth c. A & b. *** Dental Decks - page 2250 198. The rationale for pit-and-fissure sealants in caries prevention is that they: a. Increase the tooth resistance to dental caries. b. Act as a barrier between the sealed sites and the oral environment. *** c. Have anti-microbial effect on the bacteria. d. None of the above answers is correct. Dental Decks - page 2250 199. Teeth that have lost pits and fissure sealant show a. The same susceptibility to caries as teeth that have not been sealed b. Higher susceptibility than non sealed teeth c. Lower susceptibility than non sealed teeth. *** d. The same susceptibility as teeth with full retained sealant PRIMARY PREVENTIVE DENTISTRY - 6th Ed (2004) Teeth that have been sealed and then have lost the sealant have had fewer lesions than control teeth.This is possibly due to the presence of tags that are retained in the enamel after the bulk of the sealant has been sheared from the tooth surface. When the resin sealant flows over the prepared surface, it penetrates the finger-like depressions created by the etching solution. These projections of resin into the etched areas are called tags. 200. Pit and fissure sealant: a. New erupted teeth b. Deep fissure and pits in molars c. Proximal caries d. A &b. *** 201. Year old patient all first molars carious and suspected pit and fissure areas of the second molars. Treatment plan: a. Restore all first molars and observe second molars. B. Restore all first molars and topical fluoride on second molars. C. Restore all first molars and seal pits and fissures of second molars. *** d. Restore first and second molars with composite. E. Restore first and second molars with amalgam.

202. Most tooth surface affected by caries: a) Pit and fissure. *** b) Root surface. C) Proximal surface. D) ..x 203. Pit &fissure least effective with: a/tweny-four month year b/primary molar c/2nd molar 204. Pit &fissure least effective with: a/tweny-four month year b/primary molar c/2nd primary molar d/ 5 years old child 205. Procedure done before applying pit & fissure sealant: a- Acid etch by phosphoric acid. *** b- floride Dental Decks - page 2224 206. Success of pit & fissure sealants is affected mainly by: 1)increased time of etching 2) contamination of oral saliva*** 3) salivary flow rate 4) proper fissure sealant Dental Decks - page 2250 207. Management knifedge ridge in complete denture: a. Reline with resilient material. b. Maximum coverage. *** c. Wide occlusal label. d. All of the above. Dental Decks - page 208. The antibiotic of choice in pregnant: a. Metronidazole. b. Penicillin. *** c. Tetracycline.

209. Verrucous carcinoma: a. Malignant. *** b. Benign. c. Hayperplastic d. Non of above 210. Suture commonly used in oral cavity: a. Black silk. *** b. Catgut. c. Chromic. " Questions and Answers " http//www.bethesda.med.navy.mil/careers%5Cpostgraduate_dental_school% 211. In combined endo-perio problem: a. Start with endodontic IX. *** b. Start with periodontic IX. 212. Tooth fracture during extraction may be occur due to: a. None vital tooth. b. Diabetic PT. c. Improper holding by forceps. d. A and c. *** 213. Caries consist of: a. Bacteria. *** b. Fluid. c. Epithelial cells. 214. Amount of reduction in PFM crown: a. 1.5-2. *** b. 1.7-2. c. 2-5. 5 4.8

215. AH26 is root canal sealer consist of: a. ZOE. b. Epoxy resin. *** 216. Patient comes to your clinic complaining that the denture become tight, during examination you notice nothing, but when the patient stand you notice that his legs bowing (curved). What you suspect: A) Pagets disease.. ***

217. The most prevalent primary molar relationship a-flush terminal plane. *** b-mesial step terminal plane = = = c-end = = = d-distal 218. In primary teeth. The ideal occlusal scheme is: a. Flush terminal. b. Mesial step. *** c. Distal step. Dental Decks - page 252 : - :4 "" :5 : . 6-5 ... . 219. When you give a child a gift for good behavior this is called: a. Positive reinforcement. *** b. Negative reinforcement. 220. In distal extension p.d during relining occlusal Rest was not seated: a. Remove impression and repeat it. *** b. Continue and seat in after relining. c. Use impression compound. 221. After taking alginate impression: a. Wash with water and spray with sodium hydrochloride for 10 sec. b. Same but wait 5-10 min and then put in sealed plastic bag.*** 222. Many parts of bones are originally cartilaginous that replaced by bone: a. True. *** b. False.

a. b.

a. b. c. d.

223. Buccal object role in dental treatment of maxillary teeth: MB root appear distal to P if cone is directed M to D. DB root appear mesial to P if cone is directed M to D.*** we are talking about the position of the MB, or DB root in relation to the P root if the cone will be directed from M to D(both opption) so , according to the BUCCAL OBJECT RULE it says Same Lingual Opposit Buccal (SLOB), i think that MB and DB root is NATURALY on the buccal aspect to the Palatal root, so IT SHOULD , an MUST MOVE TO THE OPPOSIT DIRECTION OF THE CONE , becouse we know that they are on the buccal of P root , the question stated that the cone direction is from M to D in RELATION TO PALATAL ROOT, so the MB and DB roots shuld move and look in the mesial aspect of the palatal becouse again WE KNOW that it is BUCCAL to P root i think option b(B) is the correct one 224. Occlusal plane should be: Parallel to interpupillary line. Parallel to ala tragus line. At least tongue is just above occlusal plane. All of the above. *** 225. Pt come for check up, no complaining, after radiograph u see well circumscribe radiolucent area in related to mandibular molar & the periphery was radiopaque which not well defined border the diagnosis is: simple bone cyst. *** 226. Scallopped border above inferior alveolar canal between roots of mandibular molars, this lesion is: a) solitary cyst. b) aneurysmal bone cyst. c) traumatic bone cyst(simple bone cyst. *** 4 . This is the radiographic finding for the the trumatic bone cyst Radiographically, these lesions tend to appear as smoothly outlined radiolucencies that scallop around the roots of the teeth. They do not displace teeth or resorb roots, and the lamina dura is left intact. They may range from very small (<1 cm) to very large (involving most of the mandible). They tend to occur above the inferior alveolar canal And

Dental decks page 160 A traumatic bone cyst is not a true cyst since there is no epithelial lining. Found mostly in yong people, asymptomic. Radiolucency which appears to scallop around the roots of teeth. Teeth are usually vital. 227. Cyst between roots of mandibular molars , above the mandibular canal has Scallopped border above inferior alveolar canal, on inspiration straw like exudate, the teeth were vital, no periodontal involvment this lesion is : a. static bone cyst b. stafne bone cyst c. aneurismal bone cyst d. unicameral bone cyst*** 228. Radiographic radioulucency in the interradicular area: a. Invasion of furcation. b. Periodontal abcess. c. Periodontal cyst. ) ( ORAL RADIOLOGY 5th ed page 321 Clinical Outline of Oral Pathology 229. Mental foramen appear in radiograph as radiolucent round area to the area of: a. Mandibular premolars. *** b. Mandibular incisors. c. Maxillary canine. 230. Radiographic diagnosis of a well-defined, unilocular radiolucent, area between vital mandibular bicuspias is more likely to be: Residual cyst. The mental foramen. *** A radicular cyst. Osteoporosis. None of the above. 231. Which cyst is not radiolucent? Globulomaxillary cyst. Follicular cyst. Dentigerous cyst. *** Nasopalatine cyst. .

a. b. c. d. e. a. b. c. d.

232. Radiolucent are cover the pericornal part of the 3rd molar is: a- Dentigerous cyst b- Central 233. Cyst in x- ray: 1. Radiolucent with bone expansion. 2. Radiolucent with bone resorption *** "MASTER DENTISTRY- Oral and Maxillofacial, Surgery, Radilolgy, Pathology and Oral Medicine page 149" Cyst growth Several mechanisms are described for cyst growth, including: epithelial proliferation internal hydraulic pressure bone resorption. 234. Which of the following lesions has more tendency to show well defined multilocular radiolucency: Lateral periodontal cyst Squamous cell carcinoma of jaw bones Primordial cyst. Ameloblastoma. *** Osteomylitis of the mandible. white and pharaoh, oral radiology principles and interpretation,4th ed , page 386-389

a. b. c. d. e.

235. Radiographic diagnosis of bone destructive in the mandible without evidence of bone formation is: a. Osteomyelitis. b. Malignancy. *** c. Fibro-osseous lesion. d. Fracture. e. osteoradionecrosis. Dental secrets page 115 Malignant lesions destroy bone uniformly. In ost eomyelitis, areas of radiographically normal-appearing bone are frequently seen between the areas of destruction. Sequestra are not present in malignant lesions. 236. A 60-year-old man has been treated for a (t2nomo) squamous cell carcinoma by radical radiotherapy. He has a history of chronic alcoholism and was a heavy smoker. Six years after treatment, he develops a painful ulcer in the alveolar mucosa in the treated area following minor trauma.

His pain worsens and the bone became progressively exposed. He is treated by a partial mandibular resection with graft. The diagnosis is a. Acute osteomylitis b. Gerre,s osteomylitis c. Osteoradionecrosis *** d. Chronic osteomylitis 237. Osteoradionecrosis is more in a: maxilla b: mandible. *** c: no difference 238. Radiographic diagnosis of bilateral expansible radio opaque areas in the canine premolar region of the mandible is: Hematoma. Remaining roots. Tours mandibularis. *** Internal oplique ridge. Genial tubercle.

a. b. c. d. e.

239. -In radiographs, which disease cause multiple radiolucencies: a. Hypothyroidism. b. Hyperparathyroidism. *** c. Ricket disease. 240. The following are multilocular radiolucencies in x-ray EXCEPT: a. Ameloblastoma. b. Odontogenic keratocyst. c. Adenomatoid Odontogenic cyst. *** d. Myxoma. aneurismal bone - central giant cell reparative granuloma : Odontogenic - brown tumor of hyperparathyroidism - cyst - Hypopharynx Abscess - Cherubism Myxoma/Myxofibroma Simple Bone Cyst: Solitary bone cyst, traumatic bone cyst, bone cavity, unicameral hemorrhagic bone cyst, hemorrhagic cyst, idiopathic bone cyst Mucoepidermoid Carcinoma - Fibrous Dysplasia : 241. 33 years old female PT come with slow growing swelling in the angle of the mandible. Radiograph show radio-opaque with radio-lucent border diagnosis: a. Osteoma. b. Osteosarcoma.

c. Cementoblatoma. *** 242. Female . Swelling in left of mandible, slowly increasing , radio opaque surrounded by radiolucent band: a. Osteoma. b. Ossifying fibroma. c. Cementoblastoma. *** d. Osteosarcoma. 243. Patient came to your clinic with severe pain, on x-ray the right side of the mandible has a radiolucency with a radiopaque border that resembles the sunshine rays. Your diagnosis is : A) ossifying fibroma B) osteosarcoma. *** C) acute osteomyelitis http://jmedicalcasereports.com/content/3/1/9327 http://smj.sma.org.sg/4902/4902pe1.pdf Cortical integrity, periosteal reaction and soft tissue Slow-growing lesions often cause expansion with cortical bowing, while cortical destruction denotes aggressive inflammatory or neoplastic lesions. Presence of periosteal reaction and soft tissue is also suggestive of an inflammatory or malignant aetiology. Some types of periosteal reactions are quite specific, like the sunburst type in osteosarcoma.

Periapical and occlusal radiography and orthopantomography revealed diffuse bone destruction on the left side of the mandible due to the presence of a lesion of variable appearance, presenting dense radiopaque, mixed and completely radiolucent areas. The lesion was extensive and poorly delimited, with the periosteum showing the classical "sunray" reaction on occlusal and periapical radiographs 244. Child 12 years old with swelling in the mandibular premolars area, first premolar clinically missing, in X ray examination we found Radiolucent is cover the percoronal part of the 3rd molar is: 1. Dentigerous cyst. ***

245. Osteomyelitis more common a. Maxilla b. Mandible *** c. Zygoma d. Nasal septum e. Condyle 246. Diabetic patient came to clinic with pain & swelling & enlarged mandible, on radiograph it showed moth eaten appearance, your diagnosis is : a) acute osteomyelitis. *** b) focal sclerosing osteomyelitis. c) diffuse sclerosing. http://www.dent.ucla.edu/pic/visitors/teethloss/page1.html OSTEOMYELITIS : Radiographically the "moth-eaten" appearance is quite characteristic Dental Decks - page 170 247. Patient suffering from pain in the area of the mandibular molars with paresthesia in the lower lip. By clinical and radiographic examination your diagnosis: A) Acute osteomyelitis. *** Dental secrets page 95 Oral paresthesia may be caused by manipulation or inflammation of a nerve or tissues around a nerve, direct damage to a nerve or tissues around a nerve, tum or impinging on or invading a nerve, pnmary neural tumor, and central nervous system tumor.

248. The x ray show scattered radiopaque line in the mandible jaw, the diagnosis will be: A- Paget disease. *** B- Garres syndrome C- Fibrous dysplasia D- Osteosarcoma 249. The most common type of malignant bone tumor of the jaws is: a. Osteochondrosarcoma. b. Osteosarcoma. *** c. Leiomyosarcoma.

d. Chondrosarcoma. " Oral pathology clinical pathologic correlation" : . 250. 20 years old male PT came with severe pain on chewing related to lower molars. Intraoral examination reveals no caries, good oral hygiene, no change in radiograph. PT give history of bridge cementation 3 days ago. Diagnosis: a. Pulp necrosis. b. Acute apical periodontitis. *** c. Chronic apical abscess. d. None of the above. 251. Pt came with fracture because of blow in the right side of his face. he has ecchymosis around the orbit in the right side only .and subjunctional bleeding in the maxillary buccal vestible .with limited mouth open what is ur diagnosis? A- le fort 1 b- lofort 2 c- lefort 3 c-zygomatic fracture. *** Zygoma fracture: clinical flattening of the cheekbone prominence paraesthesia in distribution area of infraorbital nerve diplopia, restricted eye movements - subconjunctival haemorrhage - limited lateral excursions of mandibular movements - palpable step in infraorbital bony margin 252. Moon face appearance is not present in: a. Le fort I. B. Le fort II. C. Le fort III d. Zygomatic complex. *** e. Le fort II and Le fort III. Bhatia's Dentogist: mcqs in Dentistry (Clinical Sciences) with Explanatory Answers 253. Open bite is seen in a: lefort 2 b: unilateral condyle fracture c: bilateral condyle fracture *** 254. what is the first sign if there is fracture in the face ? 1. Fluid paranasal. ***

2. Suture. 3. Overlap of bone. 4. All the above. MASTER DENTISTRY- Oral and Maxillofacial, Surgery, Radilolgy, Pathology and Oral Medicine The eyes are examined for double vision (diplopia), any restriction of movement and subconjunctival haemorrhage. The condyles of the mandible are palpated and movements of the mandible checked. Swelling, bruising and lacerations are noted together with any areas of altered sensation that may have resulted because of damage to branches of the trigeminal nerve. Any evidence of cerebrospinal fluid leaking from the nose or ears is noted, as this is an important feature of a fracture of the base of the skull. An intra-oral examination is then carried out, looking particularly for alterations to the occlusion, a step in the occlusion, fractured or displaced teeth, lacerations and bruises. The stability of the maxilla is checked by bimanual palpation, one hand attempting to mobilise the maxilla by grasping it from an intra-oral approach, and the other noting any movement at extra-oral sites such as nasal, zygomatic-frontal and infraorbital. 255. By aging, pulp tissue will: a. Decrease in collagen fibers. b. Increase cellularity and vascularity. c. Decrease in size. *** . 256. Complete blood count "CBC" is a laboratory test important in dentistry: a. True. *** b. False. " Dental secrets" . a. b. c. d. e. 257. Diagnosis prior to RCT should always be based on: Good medical and dental history. Proper clinical examination. Result of pulp vitality test. A periapical radiographs. All of the above. *** 258. Which of the following may be used to disinfect gutta percha points

a. b. c. d.

Boiling. Autoclave. Chemical solutions. *** Dry heat sterilization. 259. to disinfect gutta percha points use: a- h2o2 b- 5.2% NaOCL c- d- A & B Endodontics 4th edition page 126-128 The primary GP point selected should be sterilized with NaOCL or H2O2 or Chlorhexidine 260. Most convenient and effective form of sterilization of dental instruments: a. Boiling b. Autoclave. *** 261. The role of good sterilization: 1. Washing, inspection, autoclave, drying, storage. *** 2. Inspection, autoclave, drying, storage. 3. Autoclave, drying, storage. 4. Autoclave is enough. 262. Protocol of sterilization Initial cleaning, inspection , cleaning, sterilization, storage. *** 263. Autoclave relative to 100f dry oven a)the same time b)slightly higher time c)considerable higher time d)less time. *** 264. What is the type of sterizliation applied on ligation/fixation wires A) Autoclave. *** 265. Why the moisture heat sterilization is better than dry heat sterilization A) makes the instruments less rusty and blunt B) needs more time and affects the proteins of the cell membrane C) needs less time and affects the proteins of the cell membrane. ***

266. AUTOCLAVE PRINCIPLE: a. Breaks the protein cell membrane at moderately low temp. *** b. breaks the protein cell membrane at very high temp )) (453( 267. Which one of the following is a disadvantage of autoclaving endodontics instruments: It can dull the sharp edges of instruments. *** All forms of bacteria are not destroyed by it. Compared to other technique it takes too long to sterilize. None of the above. "Dental Secrets" : . 268. The radiographic criteria used for evaluating the successes of endodontic therapy Reduction of the size of the periapical lesion. *** No response to percussion and palpation test. Extension of the sealer cement through lateral canals. None of the above. 269. If the maxillary first molar is found to have four, the fourth canal is likely found: In the disto-buccal root. In the mesio-buccal root. *** In the palatal root. None of the above. 270. What is the basis for current endodontic therapy of a periapical lesion: Due to rich collateral circulation system, the perpical area usually heals despite the condition of the root canal. If the source of periapical irritation is removed, the potential for periapical healing is good. *** Strong intracanal medications are required to sterilized the canal and periapical area to promote healing. Periapical lesions, especially apical cyst, must be treated by surgical intervention. 271. To enhance strength properties of ceramo metal restoration, it is important to: Avoid sharp or acute angles in the metal structure. Build up thick layer of porcelain.

a. b. c. d.

a. b. c. d.

a. b. c. d.

a. b. c. d.

a. b.

c. Porcelain should be of uniform thickness and any defect of the preparation should be compensated by the metal substructure. d. Compensate any defect in the preparation equally by porcelain and metal substructure. e. A and b are correct. f. A and c are correct. *** g. B and d are correct. shillingburg 3rd ed - page 457 sharp angels and under cuts should be avoided. thin porcelain with uniform thickness supported by rigid thickness is the strongest. porcelain should be kept at a minimum thickness that is still compatible with good esthetic deficiencies of the tooth preparation.should be compensated by with extra thickness of the coping in those areas. 272. Endodontically treated 2nd maxillary premolar with moderate M & D caries is best restored by: Amalgam. 3/4 crown. Full crown. *** Onlay. 273. HBV can be transmitted by transplacental: True. *** False. 274. Bleeding of the socket following tooth extraction: 1. Is always a capillary bleeding in nature. 2. Takes not less than half day in normal individual. 3. Is always favorable if it is primary type. 4. Can be due to the presence of a nutrient vessel. 1 and 2 are correct. 1, 2 and 3 are correct. 1, 3 and 4 are correct. *** All are correct. 275. In sickle cell anemia, O2 is decreased in oral mucosa: True. *** False. 276. Destruction of RBC may cause anemia and it is due to defect in cell membrane: True. ***

a. b. c. d.

a. b.

a. b. c. d. a. b.

a.

b. False. 277. Immunofluorecent test and biopsy are used to diagnosis pemphigus: a. True. *** b. False. 278. Selection of shade for composite is done: a. Under light. b. After drying tooth and isolation with rubber dam. c. None of the above. *** . . 279. Measuring blood pressure is one of vital signs important in medical compromised: a. True. *** b. False. 280. Most commonly, after placement of amalgam restoration PT. Complain from pain with: a. Hot. b. Cold. *** c. Occlusal pressure. d. Galvanic shock. e. Sweet. 281. Management of tuberosity fracture during extraction of maxillary molar is: a. Replace and suture. *** b. Remove and suture with primary heal. c. Replace and suture intra alveolar by wire. d. Remove and leave to heal. If the tuberosity is fractured but intact, it should be manually repositioned and stabilized by sutures. Dental decks 1954 282. During maxillary 3rd molar extraction the tuberosity fractured. It was firmly attached to the tooth and cannot be separated. What is the management: a) Remove it with the tooth. *** B) Splint the tooth to the 2nd molar then re-extracted after 6 weeks. C) Suture x . 283. After extraction amolar you found a hard tissue at the furcation like pearl . what is

a. Enamel pearl b. Enostosis c. Hypercementosis 284. Tooth # 36 planned to extraction on x-ray no pdl after extraction u found lesion like pearl on furcation wt the lesion a. Enamel pearl b. Enostosis c. Hypercementosis 285. Microbial virulent produced by root bacteria is collagenase from spirochete: a. True. *** b. False. 286. Bacteria in endodontic pathosis mostly is: a. Porphyromonas endodontalis obligate anaerobic: *** b. Streptococcus mutans. c. Streptococcus anaerobic. 287. Bacteria in root canal: a) mixed**** b) anaerobes obligatory c) aerobes only 288. Bacteria in root canal pathosis: a. Mixed anaerobe and aerobe. *** b. Single obligate anaerobe. c. Aerobic. d. None of the above. Dental pulp 2002 page 294 When intact teeth with necrotic pulps were cultured, over 90% of the bacteria were strict anaerobes. Because bacteria isolated from root canals are usually a subgroup of the bacteria found in the sulcus or periodontal pockets, it is believed that the sulcus is the source of bacteria in root canal infections. 289. Calcium hydroxide is used in deep cavity because it is: a. Simulate formation of 2nd dentine. *** b. Not irritant to the pulp. c. For thermal isolation.

. 290. Use of miswak and toothbrush: a. Toothbrush after meals and miswak at prayer time and when out of home. *** b. Miswak and toothbrush must be used together. c. Use the miswak only when they can not afford to buy the toothbrush and toothpaste. d. Not use the miswak and use the toothbrush instead. 291. Oral diaphragm consist mainly of: ) ( a. Tongue. b. Geniohyoid muscle. c. Digastric muscle. d. Mylohyoid muscle. *** 292. Occlusal rest function: a. To resist lateral chewing movement. b. To resist vertical forces. *** c. Stability. d. Retention. . 293. In placement of rupper dam: a. 4 jaw contact in teeth. b. Only 4 contacts 2 lingual surface and 2 buccal surface. *** c. Only 4 contacts 2 mesial and 2 distal. 294. Chronic suppurative periodontitis: a. PT complains from moderate pain. b. Fistula with drain. *** c. Pulp polyp in open coronal carious lesion. . 295. Acute periodontal abscess: a. Fistula present. b. Swelling enlargement in tooth site. *** c. None of the above. 296. Masseter muscle extends from lower of border zygomatic arch to lateral border of ramus and angel mandible. a. True. *** b. False.

Dental Decks - page 416 297. Extend of temporalis behind infratemporal fossa of temporal bone insert in coronoid process: a. True. *** b. False. 298. Main arterial supply in face is facial artery and superficial temporal artery: a. True. *** b. False. 299. Mandible is the 1st bone calcified in skull but clavicle start first but in same embryological time: a. True. *** b. False. 300. Mandible formed before frontal bone: a. True.*** b. False. 301. Maxilla is formed a. before mandible b. same with mandible c. slightly after mandible. *** d. none of the above 302. Development of maxillary process and medial frontal process in medial elongation of central portion: a. True. b. False. *** 303. Some bone are formed by endochondral ossification like long bone, flat bone by intramembranous ossification and some bone by endochondral and intramembranous ossification: a. True. *** b. False. 304. Facial nerve supply: a. Masseter muscle. b. Temporal muscle. c. Buccinator muscle. *** d. Mylohyoid muscle. 305. Upon giving a lower mandible anaesthesia, you notice the patients eye, cheek corner of the lip are uncontrolled , whats the reason : A) paresthesia of the Facial Nerve. ***

306. Muscle of facial expression are all innervated by facial nerve: a. True. *** b. False. 307. While performing cranial nerve examination you notice that the patient is unable to raise his eyebrows, hold eyelids closed, symmetrically smile or evert his lower lip..this may indicate: a. Trigeminal nerve problem. B. Facial nerve problem. C. Oculomotor nerve problem. D. Trochlear nerve problem. E. All of the above. 308. Dentin permeability 1- decreases with the increase of cavity preparation. 2- Increase when sclerotic dentin develops under a carious lesion. 3- Increase with smear layer. 4- Bacterial toxins can pass through before the actual penetration of bacteria. *** 309. Dentin permeability 1- decreases with the increase of cavity preparation. 2- Increase when sclerotic dentin develops under a carious lesion. 3- Increase with smear layer. 4- Increase with presence of sclerotic dentin under caries lesion. Art & Science Dentin permeability: increase with the increase of cavity preparation decrease when sclerotic dentin develops under a carious lesion decrease with smear layer 310. Tooth brushing and dental floss help in community prevention of periodontal disease: a. True. *** b. False. 311. Cell of chronic inflammation: a. Lymphocytics. *** b. PMN. c. Neutrophils. 312. Dentist must: a. Treat PT medically. b. Prescribe medication to Pt with medical problem.

c. Do clinical examination, take medical history and evaluate the medical state. *** 313. After patient came to your clinic and gave an extended history and complain, whats your next step in treatment : A) Clinical examination. *** B) Start the treatment C) Radiographic examination 314. Reparative dentine: a. Same like secondary dentine. b. Happen as site if irritation. *** 315. Reparative dentine: nd a. 2 dentine. b. Formed as dentine Bridge above the pulp. *** c. Highly tubular dentine and it is detective from 1st dentine. d. Sclerosing dentine with less permeability. . ------- --------- 316. Physiological reaction of edema on vital pulp: a. Decrease tissue fluid by decompression of blood vessel. b. Increase blood preasure. *** c. Necrosis of pulp due to hyperoxia and anaryxia. 317. Microabscess on vital pulp: start necrosis of small part and sequela of destruction cycle and full repair: a. True. b. False. *** " Dental pulp 2002" : . 318. Amalgam tattoo is an oral pigmentation lesion: a. True. *** b. False. 319. Oral and perioral cyst formed from epithelial rest of serres: a. True. b. False. *** 320. Cementum contain cell like bone. It is yellow in color in vital, extracted or avulsed tooth. But in non vital tooth, its color is dark:

a. True. *** b. False. 321. Dentine composition: a. 60-65 inorganic by wgt . (70% inorganic by volume). *** b. 25% water by wgt. (13% water by volume). c. 43% organic by wgt. (20% organic by volume). .%43 %53 %:3 322. The primary direction for spread of infection in the mandible is to submental lymph node: a. True. b. False. *** 323. 7 days after amalgam restoration Pt came complaining of pain during putting spoon on the restored tooth because: a. Irreversible pulpitis. b. Reversible pulpitis. c. Broken amalgam. d. Galvanic action. *** 324. Filling amalgam in the first madibular molar when touch the spoon there is a pain the reason is: 1. Galvanic action. *** 325. DNA only infect human but RNA doesn't infect human: a. True. b. False. *** . 326. Artificial teeth best to be selected by: a. Preextraction cord. *** b. Postextraction cord. Dental Decks - page 408 327. In full gold crown, to prevent future gingival recession: a. Make the tooth form gold at gingival one third. *** b. Make the tooth form gold at gingival one fifth. c. Make the tooth form gold at gingival one half. Dental Decks - page 466 328. Subgingival scaling and root planning is done by: a. Gracey Curette. *** b. Hoe. c. Chisel.

329. Paraesthesia of lower lip after surgical removal of lower 8 is due to the irritation of inferior alveolar nerve: a. True. *** b. False. 330. The aim of conditioning agent on dentine before GI cement is to remove smear layer: a. True. *** b. False. 331. Compomer release fluoride as GI: a. True. b. False. *** . 332. Barbed broach in endodontic is used for pulp examination in straight canals: a. True. *** b. False. 333. Fixed partial prosthesis is more successful in: a. Single tooth missing. *** b. Multiple missing teeth. 334. Best pontic is: a. Ridge lap. b. Hygiene. *** c. Saddle. 335. PT feel pain of short duration after class II restoration. Diagnosis is: a. Reversible pulpitis (hyperemia). *** b. Irreversible pulpitis. c. Periodontitis. 336. Radiotherapy increase caries by decreasing salivary secration: a. True. *** b. False. 337. In the preparation of cavity class II, for restoration with composite resin all cavosurface angles should be a. Well rounded. *** b. Right angles. c. Acute angles. d. Obtuse angles. 338. A class IV composite resin restoration should be finished with a: a. No. 330 Tungsten carbide bur. b. Mounted stone. c. 12- fluted carbide bur. *** d. Coarse diamond point (stone). The 12-fluted carbide burs (#7901, #7804 ET series) have traditionally been used to perform gross finishing of resin composites.

http://www.apad.cc/e-journal/issue2007/daniel.pdf 339. In class 5 composite restorations a layer of bonding agent is applied: 1. Following removal of cement then cured. *** 2. Following removal of cement and not cured. 3. Cured then remove cement. 340. After class V GI restoration removal of a thin flush of GI is done by: a. Scaller or knife immediately. b. Finishing stone immediately. c. Scale or knife later. d. Finishing stone later. e. A+B. f. A+D. *** g. B + C h. D+ C 341. After class V GI restoration removal of a thin flush of GI is done by : a. Scaller or knife in the same visit. b. Finishing stone in the same visit. c. Scale or knife next visit. d. Finishing stone next visit. e. A+B. f. A+D . B+C D+C 342. After finish class v glass ionomer cement we do finishing with: 1. Pumice slurry. 2. Aluminum-oxide disc. "Art & Science of Operative Dentistry" Micron finishing diamonds used with a petroleum lubricant to prevent desiccation are ideal for contouring and finishing conventional glass ionomers. Also, flexible abrasive discs used with a lubricant can be very effective. A fine grit aluminum oxide polishing paste applied with a prophy cup is used to impart a smooth surface. Dental Decks - page 2098 6 343. Indirect composite inlay has the following advantages over the direct composite EXCEPT: a. Efficient polymerization.

b. Good contact proximally. c. Gingival seal. d. Good retention. ***

344. Indirect composite inlay has the following advantages over the direct composite EXCEPT: a. Efficient polymerization. b. Good contact proximally. c. Gingival seal. d. Price *** 345. Indirect composite inlay over come the direct composite by 1/ insusffition polymerization 2/good contact proximaly 3/ gingival seal 4/ good retention a/ 1-2-4 b/ 1-2-3. *** c/ 4-3 346. A glossy finish is best retained on a: a. Microfilled composite resin restoration. *** b. Macrofilled resin restoration. c. Hybrid composite resin restoration. d. Fiber reinforced composite resin restoration Dental decks 2 page 2100 Microfill(fine particle composite) 0.01-0.1 *** develop smoothest finish. Fundamentals of operative dentistry, a contemporary approach, 2nd edition, Page 237 Microfilled resin composite can be polished to the highest luster and smoothest surface of all the resin composites, 347. Composite for posterior teeth: a. Microfilled + fine filler. b. Macroflled + rough filler. c. Hybrid + rough filler. *** Clinical Aspects of Dental Materials Theory, Practice, and Cases (3rd Ed 2009) The strength and other physical properties, EXCEPT wear resistance and surface roughness, of macrofilled composites are adequate for Class III, IV, and V restorations. Excessive wear when used for Class I and II restorations

limited their posterior use. Macrofills were used before dentinal bonding systems were developed; placing them in posterior teeth resulted in postoperative sensitivity, leakage, and recurrent decay. The problem with microfilled composites is the low percentage filler (40 50%). The surface area of the very small filler particles requires much more resin to wet the surface of the filler particles. This high resin content results in an increased coefficient of thermal expansion and lower strength. Microfilled composites were used when esthetics are the dominant concern. Large composite restorations, such as an extensive Class IV restoration, are built in layers of several different shades and translucencies. The first layers to be placed are a hybrid composite selected for strength. The final layer, a veneer of sorts, is a microfilled composite selected for surface luster. Microfilled composites are also used in Class V restorations at the cemento enamel junction. Microfills have a lower modulus of elasticity and flex with the tooth better than the strongest composite materials. Clinical research has shown Class V microfill composite restorations are more likely to be retained than other composite materials. Hybrid composites are very popular; their strength and abrasion resistance are acceptable for small to medium Class I and II restorations. Their surface finish is nearly as good as that of microfills; thus, they are also used for Class III and IV restorations. 348. Check bite of retainer by: a. Paste. *** b. Impression. .

349. Mastoid process is a part of: a. Temporal bone. *** b. Parietal bone. c. Occipital bone. 350. Parotid duct opens opposite in 2nd Mandibular molars: a. True. b. False. *** .

351. parotid DUCT is opposite to maxilary premolar maxilary 1st molar maxilary 2nd molar *** mandibular 1st molar The parotid duct, also known as Stensen's duct, is the route that saliva takes from the parotid gland into the mouth. It passes through the buccal fat, buccopharyngeal fascia, and buccinator muscle then opens into the vestibule of the mouth next to the maxillary second molar tooth. The buccinator acts as a valve that prevents inflation of the duct during blowing. Running along with the duct superiorly is the transverse facial artery and upper buccal nerve; running along with the duct inferiorly is the lower buccal nerve 352. Palate consists of: a. Palatine and sphenoid bone. b. Palatine and maxillary bone. *** c. Palatine and zygomatic bone. 353. Hard palate consists of the following: A. Palatal maxillray process & Ethmoid bone B. Palatal maxillary process & Sphenoid bone C. Palatal maxillary process & Palatine bone. *** D. Palatal maxillary process & Temporal bone 354. The most important microorganism in dental caries is: a. Streptococcus mutant. *** b. Streptococcus salivarius. c. Spirochaeta. 355. Emergency endodontic should not be started before: a. Establishing the pain. b. Check restorability of the tooth. c. Establishing the diagnosis. *** . 356. Selection of type of major connector in partial denture is determined: a. During examination. b. During diagnosis and planning. *** c. During bite registration. 357. White polycarbonate are temporary crowns used for anterior teeth: a. True. *** b. False.

358. For etching 15 sec, for composite restoration use: a. 37% phosphoric acid. *** b. 15% fluoric acid. c. 3% sulfuric acid. 359. Nerve impulse stops when injection local anesthesia: a. True. *** b. False. 360. The most common benign tumor in oral cavity is: a. Fibroma. *** b. Papilloma. c. Lipoma. Dental Decks - page 1030-1039 Fibroma: reactive, the most common tumor seen in oral cavity. 361. The most prominent cell in acute inflammation is: a. Lymphocyst. b. Plasma cell. c. PMN. *** 362. Flat bone grow by endochondral ossification: a. True. b. False. *** . 363. Pulp chamber in lower 1st molar is mesially located: a. True. *** b. False. " Endodontics Problem solving in clinical practice 2002" : 364. Radiopacity at the apex of a tooth with chronic pulpitis: a. Condensing osteitis (a focal sclerosing osteomyelitis). *** b. Cemental dysplasia. c. Perapical granuloma. Dental Decks - page 1050 365. Radiopacity at the apex of a tooth with deep carious lesion related to lateral surface of root : a. Condensing osteitis (a focal sclerosing osteomyelitis). *** b. Cemental dysplasia. c. Perapical granuloma. 366. Radiopacity attached to root of mandibular molar:

a. Ossifying fibroma. b. Hypercementosis. *** c. Periapical cemental dysplasia. 367. Extra canal if present in mandibular incisor will be: a. Lingual. *** b. Distal. " Endodontics Problem solving in clinical practice 2002" : %78 .%93 . 368. The access opening in lower incisor: a. Round. b. Oval. *** c. Triangular. Dental Decks - page 152 369. The accesses opening for a maxillary premolar is most frequently: a. Oval. *** b. Square. c. Triangular. d. None of the above. Dental Decks - page 152 370. The correct access cavity preparation for the mandibular second molar is: a. Oval. b. Quadrilateral. c. Round. d. Triangular. : .. NBDE part II :Upper central : triangular lateral: ovoid canine: ovoid first premolar: ovoid second premolar: ovoid first molar: triangle second molar triangle :Lower central: ovoid

lateral ; ovoid canine: ovoid first premolar: ovoid second premolar : ovoid first molar: trapezoid second molar: trapezoid 371. Acute periapical cyst and acute periodontal cyst are differentiated by: a. Vitality test. *** b. Radiograph. c. Clinical examination. 372. Acute periapical abscess associated with a. Swelling. *** b. Widening of PDL. c. Pus discharge. 373. The most common cause of endodontic pathosis is bacteria: a. True. *** b. False. . 374. Palatal canal in upper molars is curved: a. Buccally. *** b. Palatally. c. Distally. 375. If tooth or root is pushed during surgical extraction into max sinus: a. Leave it and inform the Pt. b. Remove it as soon as possible. *** c. Follow the Pt for 3 months. d. None of the above. 376. Difference between Gracey and universal curette: a. Section of gracey is hemicircular and in universal triangular. b. Gracey has one cutting edge while universal has two. c. Gracey Used for cutting in specific area while universal is in any area. d. Universal 90 not offset, gracey 60 offset. e. A and d f. A, b and c. g. B, c and d. 377. Cause of fracture of occlusal rest: a. Shallow preparation in marginal ridge. *** b. Extension of rest to central fossa. c. Improper centric relation. 378. Child came to the clinic with amalgam restoration fracture at isthmus portion, this fracture due to:

a) Wide preparation at isthmus. b) High occlusal. C) shallow preparation *** D) constricted isthmus Oxford Handbook of Clinical Dentistry - 4th Ed. (2005) page 58 Isthmus joins the occlusal key with the interproximal box. It is the part of the filling most prone to fracture. :4<6 " " 379. after class II amalgum fill , broken is happen in isthmus area why A. over high of filling virtically B. over flair cavosurface angle or edge C. unproper mixed fill 380. The cause of fracture in amalgam class II restoration is: a. Thin thickness at the marginal ridge. *** b. Wide flared cavity c. Deep cavity. 381. Bridge return to dentist from lab with different degree of color although the shade is the same, the cause: a. Thin metal framework. b. Different thickness of porcelain. *** c. Thick opaque. 382. Complete denture poorly fit and inadequate interocclusal relation: a. Relining. b. Rebasing. c. New denture. *** d. None of the above. 383. Small caries confined to enamel: a. Preventive measure. *** b. Amalgam feeling. c. Keep under observation. 384. In enamel caries passing half of enamel: a. Leave it. b. Restoration. ***

385. Currently the only effective preventive measure for periodontal disesse (apart from limited use of antiseptic solutions) is: a. Regular and rough removal of dental plaque. *** b. Salt flouridation c. Dental health education. 386. Dental caries: a. Is a transmissible disease b. Is world wide in distribution but uneven in intensity. c. Can be prevented d. All of the above. *** e. None of the above. Dental caries is a transmissible and multi-factorial disease. " It suggests an impact on populations of entire countries, continents, or much of the world. The term therefore implies two elements: global distribution and severe consequence Dental caries is a disease that usually can be successfully prevented or controlled 387. At which location in enamel is the density of enamel crystals is lowest: a. Prismless enamel. b. DEJ. *** c. Center of enamel Prisms. d. Edge of enamel Prisms. e. Facial enamel. sturdenvant 4th edition - page 17 The density of enamel also decrase from the surface to the DEJ. 388. Rampant caries in adult in anterior teeth restored by: a. Glass ionomer. *** b. ZOE. c. Amalgam. 389. The 1st cervical vertebrae is: a. Atlas. *** b. Axis. -: -9 -8 -7 Longus colli -6 Axis or Epistropheus,-5 Atlas-4 : prominens. 390. Most of dentine bonding material need conditioning time: a. 15 sec. b. 30 sec. c. 45 sec.

d. 60 sec. 391. Time of curing of dentine: a. 10 sec. b. 15 sec. c. 30 sec. d. 60 sec. 392. Light curing time for simple shallow class III composite: a. 10 sec. b. 15 sec. c. 20 sec. 393. Cartilaginous joints in the body affect bone growth: a. True. *** b. False. 394. Cavity varnish should be applied at least in: a. One layer. b. Two layer. *** c. Three layer. d. Four layer. 395. Geographic tongue is seen in Pt with: a. Diabetes. *** b. Iron deficiency anemia. c. Pemphigus. Burket- Oral medicine page 116 Benign migratory glossitis is seen with a fourfold increase in frequency in patients with juvenile diabetes, possibly due to an increased frequency of elevated amounts of the HLA-B15 tissue type. . a. b. c. d. 396. Geographic tongue is always accompanied in patient with: Diabetes. Erythema multiform. Iron deficiency. Psoriasis. *** Cawson Essintials of Oral Pathology and Oral Medicine 7th ed Geographical tongue: . .

. . . psoriasis Burket- Oral medicine page 115 There may be an association between certain types of psoriasis (especially pustular psoriasis) and geographic tongue. 397. A 21 years old patient who has iron deficiency anemia, difficulty in swallowing, with examination of barium sulphate, you found: A. Geographical tongue. B. Burning mouth syndrome. *** C. .. Syndrome D. Diabetic patient Iron deficiency result in: Candida Leukoplakia - Aphthous Stomatitis - Sore tongue (glossodynia glossitis Burning mouth syndrome) - Patterson-Brown-Kelly syndrome (GASTROINTESTINAL DISEASE) 398. a 21 years old patient who has iron deficiency anaemia, difficulty in swallowing, with examination of barium sulphate, you found >>>>>>>missing part>>>>>>> a. geographical tongue b. burning mouth syndrome c. (plummer vinson syndrome) patterson kelley d. diabetec patient ( plummer vinson syndrome) patterson kelley burning mouth ) . (63 . )( http://emedicine.medscape.com/article/187341-overview http://emedicine.medscape.com/article/187341-diagnosis

399. Pt came to the clinic complaining from soreness in the tongue sore throat the diagnosis is: a) burning mouth syndrome. *** b) geographical tongue c) fissure tongue Burket- Oral medicine page 96 Acute atrophic candidiasis presents as a red patch of atrophic or erythematous raw and painful mucosa, with minimal evidence of the white pseudomembranous lesions observed in thrush. Antibiotic sore mouth, a common form of atrophic candidiasis, should be suspected in a patient who develops symptoms of oral burning, bad taste, or sore throat during or after therapy with broad-spectrum antibiotics. Patients with chronic iron deficiency anemia may also develop atrophic candidiasis. simplex herpes 400. Burning mouth syndrome is a chronic disorder typically characterized by each of the following EXCEPT: a. Mucosal lesion. *** b. Burning pain in multiple oral sites. c. Pain similar in intensity to toothache pain. d. Persistent altered taste perception. 401. Which of the following is the most likely potential cause of BMS (Burning mouth syndrome): a. Denture allergy. b. Salivary dysfunction. *** c. Neural dysfunction. d. Menopausal changes.
There are a number of possible causes of burning mouth syndrome, including:

damage to nerves that control pain and taste hormonal changes dry mouth, which can be caused by many medicines and disorders such as Sjgrens syndrome or diabetes nutritional deficiencies oral candidiasis, a fungal infection in the mouth acid reflux poorly-fitting dentures or allergies to denture materials anxiety and depression.

In some people, burning mouth syndrome may have more than one cause. But for many, the exact cause of their symptoms cannot be found.

http://www.nidcr.nih.gov/OralHealth/Topics/Burning/BurningM outhSyndrome.htm#2
BMS ( . ) ) ( lichen planus .xerostomia ) ( . ACE Candida albicans . thrush ) ( . reflux

http://www.aawsat.com/details.asp?issueno=10626&article=48 1372 402. Which of the following represents the best pharmacologic therapy for BMS: a. Antidepressant agents. b. Corticosteroids. c. Anxiolytic agents. d. There is no therapy of proven general efficacy. *** http://jada.ada.org/cgi/reprint/126/7/842.pdf If an underlying cause of BMS can be identified (Table 2), then treatment should be directed toward the source."41"6 Unfortunately, treatment based on any of these possible etiologic factors is frequently ineffective,' and no therapy for BMS has proven generally effective.

403. Diabetic Pt with ill fit denture, examination of residential ridge help to: a. Determine the need for tissue conditioning and surgery. *** b. Determine occlusal height. c. Determine vertical dimension of occlusion. 404. Handicapped Pt with lesion in central nervous system appears to have different type of disorder in movement and procedure: a. Seizure. b. Cerebral palsy. *** c. Learning disability. McDonald, 7th ed, page 585 Cerebral palsy.the disability might involve .and uncoordinated or involuntary movements

405. To obturate the canal the most important step is: a. Cleaning and shaping of the canal. *** b. Irrigation of the canal. 406. aim to shape apical 3rd of the root: a) widening apex b) permit irrigation to reach apical 3rd. c) permit GP to fill d) for good sealer achieve .) ( . 407. The most important in RCT is seal: a. Apical 1/3. *** b. Middle 1/3. c. Cervical 1/3. 408. During placement of amalgam pins, the number of pins per cusp is: a. 1 pin. *** b. 2 pins. c. 3 pins. d. 4 pins. 409. The amount of L.A in 2% lidocaine with 1/100000 adrenaline is: a. 0.01 b. 0.02 c. 36 mg. *** Dental Decks - page 1870 410. How many mg of adrenalin in cartridge 1.8 cc of 2 % Xylocine: a- 1.8 mg b- 0.18 mg c- .036 mg d- 3.6 mg e- 0.018 mg. *** 411. What is the amount in mg in 1.8 ml of xylocain a. 20 mg/ml b. 1.8 mg/ml c. 36 mg. *** 4.; 69 %5 3.34; 433333 412. The most common endodontic cyst is: a. Radicular cyst. *** b. Keratocyst. c. Acute apical periodontitis.

413. Pt complains from severe spontaneous pain related to upper 6. It respond to vitality test no pain on percussion, diagnosis is: a. Irreversible pulpitis. *** b. Reversible pulpitis. c. Acute apical periodotitis. 414. The most common complication after extraction for diabetic Pt is: a. Infection. *** b. Severe bleeding. c. Oedema. d. All of the above. 415. Lateral canal is detected by: a. Radiograph. *** b. Tactile sensation. c. By clinical examination. 416. Contact area is in incisal/occlusal 1/3 in which tooth: a. Mandibular incisors. *** b. Mandibular molars. c. Maxillary molars. 417. Incipient caries is diagnosed by: a. Fiber optic light. *** b. Tactile examination. c. X-ray film. (And dye) 418. Disinfection of GP is done by: a. Autoclave. b. Dry heat. c. Sodium hypochlorite. *** 419. Periodontal ligament fibers in the middle third of the root is: a. Oblique. *** b. Horizontal. c. Transeptal. 420. To detect interproximal caries in primary teeth, the best film is: a. Periapical. b. Bitewing. *** c. Occlusal. 421. Pt with missing lower right 1st molar for long time you'll find: a. Mesial drifting of lower right 2nd molar. *** b. Intrusion of upper right 1st molar. c. Over eruption of lower right 2nd molar. 422. Over erupted upper right 1st molar will be managed by: EXCEPT: a. Intruded easily orthodontically. *** b. Crowing. c. Adjustment of occlusion.

423. Over erupted upper right 1st molar will be managed by:: a. Intruded easily orthodontically. b. Crowing. c. Adjustment of occlusion. d. In sever cases may be extracted e. 1 2 3 f. 2 3 4**** g. 1 2 4 h. 1 3 4 424. Over erupting tooth can be treated by: 1. Crowning after endo. 2. Ortho intrusion. 3. Extraction. 4. A and B. 5. All the above. 425. Broken instrument during RCT, best prognosis if broken at: a. Apical 1/3. *** b. Middle 2/3. c. Cervical 3/3. 426. Pulp stone: a. Cause discomfort and pain. b. Free in pulp chamber. *** c. None of the above. 427. Pulp stone can be the following EXCEPT: a) present freely in the pulp. b)cause discomfort & pain to the patient. *** c)In radiographs,Small spheroidal radiopaque. d)False stone occurs due to dystrophic dentin. Pathway of the pulp 9ed 1st part page 219 The morphology of the apical root varies tremendously; it includes numerous accessory canals; areas of resorption and repaired resorption; attached, embedded, and free pulp stones; and varying amounts of irregular secondary dentin. : : : -

. -6- -5- -4 : : 428. The amount of facial reduction in PFM crown: a. 1.3. b. 1.7. c. 0.8. d. 2.2. 429. A tooth with 25 degree inclination could be used as abutment: a. True. *** b. False. 430. Intercellular movement of PMN leukocytes is called migration: a. True. *** b. False. 431. In onlay, stopping of cusp is 1.5-2 m.m: a. True. *** b. False. 432. Among the reasons that molar teeth are more difficult to treat endodontically than anterior teeth: a. Molar have more complex canal configuration. b. Molar tend to have greater canal curvature. c. A and b. *** d. None of the above. 433. Gingival hyperplasia related to phenytoin therapy is: a. Most common on lingual surface. b. Most common in older Pt. c. Strongly related to phenytoin dosage. d. Strongly related to poor oral hygiene. *** 434. Type I diabetes mellitus can be characterized as: a. Non-insulin-dependent. b. Adult- onset. c. Ketosis-prone. *** d. Accompanied by normal cell activity. http://en.wikipedia.org/wiki/Diabetes_mellitus_type_1 435. Which of the following statement is true for the reported relationship of periodontal disease and diabetes mellitus: a. The reported incidence of periodontal disease in the diabetes is less than that for nondiabetic. b. Pts with history of diabetes of less than 10 years have more periodontal disease destruction than those with history of longer than 10 years.

c. The prevalence of periodontal disease increase with the advancing age of the diabetic. *** d. The prevalence of periodontal disease increase with the better metabolic coronal of the diabetic state.. 436. The spontaneous production of an electric current resulting from two dissimilar metal in the oral cavity is called: a. Nuclear reaction. b. Galvanic action. *** c. Precipitation reaction. d. Thermodynamics. e. Fission. Dental Decks - page 2268 437. The first step in diagnostic work, up is obtaining the: a. Medical history. b. Present complaint. *** c. Biographical data. d. Restorative history. e. Traumatic history. 438. The basic difference between K files and reamers is: a. The number of spirals or flutes per unit length. b. The geometric cross section. *** c. The depth of flutes. d. The direction of the spirals. 439. In case of traumatic intrusion of young permanent incisor, the treatment of choice is: a. Surgical repositioning of intruded tooth and splinting. b. To wait for re eruption of the intruded tooth. *** c. Slow orthodontic extrusion using light force. d. Only antibiotic prescription and wait for eruption. "Dental Secrets" 43 . 440. Best treatment of choice for carious exposure of a primary molar in a 3 year old child who complain of toothache during and after food taking: a. Direct pulp capping with caoh. b. Direct pulp capping with zao paste. c. Formocresol pulpotomy. *** d. Caoh pulpotomy. 441. Which of the following statement about the mechanism of action for denture adhesive is not correct:

It depends in part on physical force and viscosity. *** Carboxyl group provide bio adhesion. Greater water solubility increase duration of adhesion. Zinc salts have been associated with stronger longer adhesion. . . 442. One week after filling of class II restoration, the Pt present with a complain of tenderness on mastication and bleeding from the gingival. The dentist should initially: a. Check the occlusion. b. Check the contract area. *** c. Consider the probability of hyperemia. d. Explain to the Pt that the retainer irritated the surrounding soft tissue and prescribe an analgesic and warm oral rinse. 443. It is recommended to avoid an intraligamental injection when the planned dental treatment is: a. Pulp extirpation. b. Pulpotomy. *** c. Full crown preparation. d. A and b. . a. b. c. d. 444. The root canal treated teeth has the best prognosis when the root canal is instrumented and obturated: a. To the radiograph apex. b. 1 mm beyond the radiograph apex. c. 1-2 mm short of the radiograph apex. *** d. 3-4 mm short of the radiograph apex. " Clinical Endodontics TRONSTAD" : 445. Which of the following would be clinically un acceptable as a primary of isolating a tooth for sealant placement: a. Cotton roll. b. Rubber dam. c. Vac-ejector moisture control system. d. None of the above. *** 446. Which one of the following is least likely to contribute to oral bad breath: a. Periodontal disease. b. Denture. c. Faulty restoration. d. Carious lesions. 447. Which one of the following is not a characteristic of dentinal hypersensitivity:

a. b. c. d.

It is one of the most successfully treated chronic dental problems. *** Its prevalence range from 8 to 30%. The majority of the Pts who experience it are from 20 to 40 years of age. One source of the irritation that leads to hypersensitivity is improper tooth brushing. :4 . . Art and science of operative dentistry 2000 :5 Dentin hypersensitivity is a common clinical condition that is difficult to treat because the treatment outcome is not consistently successful. 448. Hypersensitivity is due to: A- Exposed dentine with opened dentinal tubules. *** B- Obliterated dentinal tubule 449. The most common form of oral ulcerative disease is: a. HSV. b. Major aphthous ulcer. c. Bahjet disease. d. Minor aphthous ulcer. *** 450. The majority of primary herpetic infections are: a. Symptomatic. b. Asymptomatic. *** c. Proceeded by fever. d. Accompanied by gingival erythema. e. A, c and d. 451. The function of the anterior teeth is: a. Disarticulate the posterior teeth. b. Incise food. *** c. Prevent attrition. d. Prevent food impaction. 452. In geriatrle Pt, Cementum on the root end will: a. Become thinned and almost nonexistent. b. Become thicker and irregular. *** c. Render apex to locater useless. d. Often not be seen on the radiograph. e. Indicate pathosis. 453. Tobacco should be considered a risk factor when planning treatment for Pt who require: a. Implants. b. Periodontal surgery. c. Oral surgery.

d. Esthetic treatment. e. All of the above. *** 454. Pulpal pain may NOT be referred from: a. The right maxilla to the left maxilla. *** b. The third molar to the ear. c. A max molar to the sinus. d. An incompletely fractured tooth. e. A max cuspid to ear. 455. Spontaneous pulpal pain is indicative of: a. Reversible pulpitis. b. Irreversible pulpitis. *** c. Neurotic pulp. d. Hyperplastic pulp. e. Atrophic pulp. 456. Internal Resorption: a. Painful. b. Seldom differentiated external resorption. c. Can occur in primary teeth. *** .

a. b. c. d.

457. Primary teeth had trauma, tooth change in color become white yellowish ,what should you tell the parents: Pulp is dead Inflammation of pulp. Calcification of dentin. B& c. *** 73: 69" Principles and Practice of Endodontics WALTON " : .

458. Teeth that are discolored as a result of internal resorption of the pulp may turn: a. Yellow. b. Dark brown. c. Pink. *** d. Green. Dental Decks - page 244

a. b. c. d. e.

459. Treatment of internal resorption involves: Complete extirpation of the pulp to arrest the resorption process. *** Enlarging the canal apical to the resorbed area for better access. Utilizing a silver cone and sealer to fill the irregularities in the resorbed area. Filling the canal and defect with amalgam. Sealing sodium hypochlorite in the canal to remove the inflammatory tissue necrotic in the area of the resorption. Clinical Endodontics textbook TRONSTAD page 150 Irrigation with copious amounts of 5 % sodium hypochlorite may have some effect, but the treatment of choice is to pack the canal and the resorption lacuna with calcium hydroxide paste. By the next visit, the calcium hydroxide will have necrotized any remaining tissue in the lacuna, and the necrotic remnants are readily removed by irrigation with sodium hypochlorite. 460. Sensitivity to palpation and percussion indicates: a. Reversible pulpitis. b. Irreversible pulpitis. c. Neurotic pulp. d. Hyperplastic pulpitis. e. Inflammation of the periradicular tissues. *** 461. Trauma lead to fracture in the root between middle cervical and apical third: a) poor prognosis b) good prognosis *** . : 462. Tooth with a fracture between the apical and the middle third, what's your management a) RCT for the coronal part only. b) RCT for both. c) Splint the two parts together :448 . :459

: ; 45- : . . . Text book of Pediatric Dentistry :446 PAEDIATRIC DENTISTRY - 3rd Ed. (2005) page 343 . . . . 6 : . . . : . 463. Transverse fracture of developing teeth in the mixed dentition can be managed by: *** a. Forced eruption. b. Extraction and placement of a removable partial denture. c. Placement of single tooth. d. All of the above. Root Fractures The noncommunicating fracture occurs in the apical or middle third of the root. Perform a vitality test, check for color change in the crown, and record the degree of mobility of each traumatized tooth. If the pulp is vital, then immobilize the tooth by splinting it to the adjacent teeth................... .......If the fracture of any part of the root is coronal to the periodontal attachment,The fractured part should be removed during the emergency visit, and endodontic treatment should be done in one visit. Once the emergency has been taken care of, plans must be made for restoring the tooth .

Text book of Pediatric Dentistry 449 464. pt presented to u with trauma of the central incisor with open apex tooth clinical examination revealed cut of blood supply to the tooth what is the next step: 1-extraction 2-endo 3- observe over time. *** 465. child came to u with gray discolouration of the deciduous incisor also on radiographic exam. There is dilation of follicle of the permenant successor what will u do: 1-extract the decidous tooth. *** 2-start endo 3-observe over time 466. pt came with class IV he had tooth trauma & he brought the fracture segment & on examination u found that the pulp is not exposed & only u can see dentine, how u manage: - to get rid of the fragment & fill with composite. - to reattach the fragment with composite and latter cover with veneer. *** - others 467. adult 20 years male with soft tissue & dental trauma reveals severe pain in soft tissues with loss of epithelial layers and anterior upper centrals are intruded the diagnosis is: a-abrasion with luxation b-errosion with sub luxation c- traumatic ulceration with luxation. d-ulceration with subluxation 468. adult 20 years male with soft tissue & dental trauma reveals severe pain in soft tissues with loss of epithelial layers and anterior upper centrals are intruded the diagnosis is: a-abrasion with luxation b-errosion with sub luxation c- Laceration with luxation.. d-laceration with subluxation 469. Apexification is procedure that: a. Finds the most apical stop of the guttpercha in RCT. b. Induce the formation of a mineral barrier in the apical region of incompletely root. *** c. Is new in the endodontic field.

d. Involves the surgical removal of the apical region of the root and placement of a retrograde filling material: 470. The preferred material used in apexification is: a. Zinc phosphate cement. b. Zinc polycarboxylate cement. c. Calcium hydroxide. *** d. Dycal. 471. what is the time between the first onset of HIV virus and the appearance of acute symptoms : a)1-5 years. b)9-11 years. *** c- No specific time is known. 472. What is the estimated incubation period of HIV infection: a. 4 weeks. b. 6 months. c. 3 years. d. 6 years. e. 10 years. *** http://en.wikipedia.org/wiki/Aids In the absence of antiretroviral therapy, the median time of progression from HIV infection to AIDS is nine to ten years, and the median survival time after developing AIDS is only 9.2 months. However, the rate of clinical disease progression varies widely between individuals, from two weeks up to 20 years. 473. Hydrogen peroxide is the ideal bleaching agent because: a. It bleaches effectively at natural ph. b. It bleaches faster than carbamide peroxide. c. Protection for sensitive tissues can be incorporated into the hydrogen gel. d. All of the above. *** 474. The most common cause of the angina is: a. Stress. b. Renal disease. c. Arteriosclerotic plaques of the coronary vessels. *** d. Hypoglycemia. e. Hypertension. 475. Which of the following drugs is completely effective in eliminating angina episode:

a. Propranolol. b. Nifedipine. c. Diltiazem. d. Transdermal nitroglycerin. *** e. None of the above. 476. CPR a. Is best performed in the dental chair. b. Should be performed on all patients experiencing chest pain. c. Is more efficient when using a full mask, delivering 100% oxygen, than with the mouth to mouth technique. *** d. Is beyond the medico legal responsibility of the practicing dentist. 477. Which statement concerning sensitive teeth is false: a. Small dentin exposure can result in sensitivity. b. The extent of dental hard tissue loss always correlates with sensitivity. c. A wide variety of clinical condition can cause teeth to become sensitive. d. Oral hygiene habits and diet can contribute to clinical sensitivity problems. . 478. Droplet nuclei containing mycobacterium tuberculosis: a. Do not cause infection. b. Settle out of room air quickly. c. Do not spread widely in the building. d. Remain airborn for prolonged period. *** 479. The most common activity associated with percutaneous injury of the dentist is: a. Suturing. b. Anesthesia injection. *** c. Handpiece dig. d. Trimming impressions. 480. The most common location of percutaneous injury among dentist is: a. Hand. *** b. Face. c. Elbow. d. Arm. 481. The normal response of a vital pulp to the thermal testing is: a. No response. b. Lingering painful response. c. Hypersensitive painful response. d. Painful response that disappears soon after stimulus is removed. ***

482. The normal response of a inflamed pulp to the thermal testing is: a. No response. b. Lingering painful response. c. Hypersensitive painful response. d. Painful response that disappears soon after stimulus is removed. http://www.hindawi.com/journals/ijd/2009/365785/
The three types of responses can be summarised as follows. (i) The pulp is deemed normal when there is a response to the stimulus provided by the sensibility test and this response is not pronounced or exaggerated, and it does not linger. (ii) Pulpitis is present when there is an exaggerated response that produces pain. Pulpitis can be considered as reversible or irreversible, depending on the severity of pain and whether the pain lingers or not. Typically mild pain of short duration is considered to indicate reversible pulpitis while severe pain that lingers indicates irreversible pulpitis [7, 8]. (iii) The absence of responses to sensibility tests is usually associated with the likelihood of pulp necrosis, the tooth is pulpless, or has had previous root canal therapy

483. The normal response of a vital pulp to the electric pulp testing is: a. No response. b. Higher than that of the control teeth. c. Lower than that of the control teeth. d. In a range similar to that of the control teeth. *** 484. Asymptomatic tooth has a necrotic pulp, a broken lamina dura, and circumscribed radiolucency of long duration. The periradicular diagnosis: - a. Acute apical periodontitis. b. Chronic apical periodontitis. *** c. Acute exacerbation of chronic apical periodontitis. d. Abscess. 485. A Pt with severe periradicular pain has a necrotic pulp, a broken lamina dura, and circumscribed radiolucency of long duration. The periradicular diagnosis: a. Acute apical periodontitis. b. Chronic apical periodontitis. c. Acute exacerbation of chronic apical periodontitis. *** d. Abscess. 486. A Pt present in severe pain. The periapical area over the involved tooth is inflamed and swollen. The tooth is mobile and depressible in its socket with a diffused radiolucency. The diagnosis is: a. Acute apical periodontitis. b. Chronic apical periodontitis. c. Acute exacerbation of chronic apical periodontitis. d. Abscess. ***

487. Reduction of mandibular fracture is defined as: a. Nonalignment and separation of the fracture segment. b. Realignment of fracture segments. c. Holding of the fracture segments in place. *** d. Screw and bone places. e. Internal fixation. 488. Wiring the upper and lower teeth together is called: a. Internal fixation. b. An open reduction. c. Intermaxillary fixation. *** d. Displacement. e. External fixation. http://nyp.org/health/intermaxillary-rigid-fixation.html
Fixation is an important step in treating fractures. It is also a crucial part of orthognathic surgery, used to correct mandibular and maxillary deformities. Orthognathic surgery on the mandible and maxilla generally involves breaking the bones in a controlled way and then resetting them into correct positions. After the bone is set (a process called "reduction") a period of fixation ensures proper healing. Oral and maxillofacial surgeons use two basic fixation techniques. One of these, intermaxillary fixation, involves binding the jaw shut with wires or elastic bands. The other, called rigid fixation, is a newer technique in which tiny screws or plates are attached directly onto the fractured sections of the jaw bone; it does not require physically binding the jaws shut.

489. The incidence of nerve damage after third molar surgery is estimated to be: a. 5% or less. *** b. 10% to 15%. c. 15% to 20% d. 20% to 25%. 490. The least likely mechanism for the nerve damage is: a. Direct needle trauma. b. Intraneural haematoma formation. c. Local anesthetic toxicity. *** d. Stretching and binding of the nerve. 491. Which of the following is the cause of immediat type allergic reaction to latex products: a. Accelerator. b. Antioxidants.

a. b. c. d. e. a. b. c. d. a. b. c. d. e.

c. Latex protein. *** d. Nickel. 492. Which of the following is the longest in the dental arch: a. Maxiliary central incisor. b. Maxiliary second premolar. c. Mandibular canine. d. Maxiliary canine. *** 493. Chlorhexidine is used as mouth wash in the concentration of: a. 0.1-0.2% *** b. 1-2% c. 5-10% d. 20% 494. Traumatically fractured crown of central incisor in an 8-years-old child with pulp exposure (more than 1 mm) half hour ago, medical history is non- contributory and the tooth is not displaced. What is your management: a. Endodontics-pulpectomy and obturation. b. Direct pulpcap with caoh and composite. c. Caoh pulpotomy. *** d. Total extirpation of pulp and caoh. 495. The use of the rubber dam in endodontics is: Frequently required. An established rule. *** Not required. Time consuming. Dictated by Pt comfort. 496. Dentine hypersensitivity is best relieved or controlled by: Using efficient cooling system. Blacking exposed tubules on the dentin surface. *** Opening tubules to permit release of intrapulpal pressure. Applying anti inflammatory agent to exposed dentin. 497. Which of the following is a benign epithelial neoplasm: Rhabdomyoma. Fibroma. Lipoma. Granular cell tumor. Keratoacanthoma. *** Journal of Applied Oral Science And MCQs in Dentistry Cawson page 79 And http://famona.erbak.com/OTOHNS/Cummings/cumm072.pdf

A rhabdomyoma is a benign neoplasm of striated muscle. Almost all lesions in the oral cavity that are called fibromas are not true neoplasms A lipoma is a benign neoplasm composed of fat cells. A granular cell tumor, also called granular cell myoblastoma or Abrikosov's tumor in the past, is a benign lesion of the soft tissues whose origin and nature are not fully understood. For a long time the lesion was considered a benign neoplasm related to muscles. Currently a neurogenic origin seems to be more likely. A keratoacanthoma, also called molluscum sebaceum, is a benign cutaneous lesion that is believed to arise from hair follicles Histologic examination of a keratoacanthoma shows hyperplastic epithelium with carcinoma-like features And Journal of Applied Oral Science The keratoacanthoma: It is defined as a benign epithelial neoplasm, originated from the superior portion of the sebaceous gland of the hair follicle 498. The function of the periodontal ligament include a. Mechanical function b. Formative function c. Nutritive function d. Sensory function e. All of the above. *** clinical periodontology 9th edition page 39 499. Ankylosis: a. No PDL b. Caused by trauma c. Extracted surgically d. All of the above. *** 500. The following are types of hamartoma EXCEPT: ) ( a. Cementoblastoma ***. b. Compound odontoma. c. Complex odontoma.

501. A child came to the clinic with continuous involuntary movement of his head and extremities and difficulty in vocal communication; The condition is described as: a. Epilepsy. b. Cerebral palsy ***. 502. The movement of water across a selectively permeable membrane is called: a. Osmosis. *** b. Active transport. c. Filtration. d. Diffusion. Osmosis Osmosis is the net movement of water across a selectively permeable membrane driven by a difference in solute concentrations on the two sides of the membrane. 503. Cell that can give more than one type: a. Fibroblast. b. Odontoblast c. Mesenchymal cell. *** Leiomyosarcoma Terms Leiomyoma through Neurofibromatoses Mesenchymal cell : An undifferentiated cell found in mesenchyme and capable of differentiating into various specialized connective tissues. 504. High rate of fractures at canine area in the mandible due to: a. Change direction of forces occruing here b. Long canine root *** c. Lower border is thin in this area d. Alveolus is thin in this area Surgery: Basic Science and Clinical Evidence By Jeffrey A. Norton - Page 2013 The mental foramen, and the long roots of the canine teeth as well as impacted 3rd molars create points of weakness that are particularly prone to fracture. 505. Lesion similar to Endo Lesion: a. Hyperparathyroidism b. Initial stage of cemental dysplasia *** c. Ossifying Fibroma d. Dentigerous cyst. e. Ameloblastoma. f. Lateral periodontal cyst. g. Myxoma & hemangioma. 506. Infection is more dangerous in children than adult because: a. Marrow spaces are wide ***

b. Affect growth centre. c. Hypo calcification in enamel. 507. The Common disease affecting the submandibular salivary gland is: a. Salivary calculi *** b. Pleomorphic adenomas. c. Viral sialoadenitis. d. Infected sialoadenitis. 508. Which most common salivary gland neoplasm: Pleomorphic adenoma. *** Oral pathology clinical pathologic correlation 3rd edition - Page 239 Pleomorphic adenoma is the most common tumor of the major and minor salivary glands . %;3 509. Ranula is associated with which salivary gland: a-submandiuLar gland. b-sublingual gland. *** EAOM - OBSTRUCTIVE SALIVARY DISEASE [Notizia-89] A ranula is a similar cyst arising in the floor of mouth from the sublingual gland 510. Ranula Can be treated by: a. Excision. - b. Cauterization. c. Incision. d. Marsupialization. *** Oral pathology clinical pathologic correlation,3rd edition, Page 222 "Marsupialization can be performed before a definitive excision" And Contemporary oral and maxillofacial surgery, peterson, 4th edition, Page 447 The usual treatment of ranula is marsupialization. The preferred tx for recurrent or persistent ranula is excision of the ranula and sublingual gland. 511. solitary bone cyst management a. anti inflammatory and follow up b. curettage and close c. marsupialization*** d.no active management

512. solitary bone cyst management a. anti inflammatory and follow up b. curettage and close c. marsupialization and antibiotic *** d.no active management 513. For the ceramometal restorations, the type of finish line is: a. Chamfer b. Beveled shoulder *** shillingburg 3rd ed - page 151 Beveled shoulder: According to the results of this study as the shoulder bevel had better fit than shoulder and deep chamfer designs and also there was significant difference between shoulder bevel and shoulder, so it is recommended to use shoulder bevel finish line in the metal ceramic restoration. http://scialert.net/fulltext/?doi=jms.2008.665.668&org=11 514. Benefits of opaque porcelain layer: a. Bonding the metal structure. b. Initiating the color. c. A & b. *** Dental Decks - page 604 515. Radiographic examination in impacted teeth is useful to demonstrate: a. Proximity of the roots to the adjacent anatomical structures. b. Associated pathology. c. All of the above. *** The criteria for imaging impacted teeth include identifying the impactions within the confines of the surrounding alveolar bone, then to determine their location relative to adjacent teeth and vital structures in the dento-alveolar complex and then finally to evaluate whether there is pathology Use of Tomography For Evaluating Impacted Posterior Teeth http://www.aadmrt.org/currents/abramovitch_spring03_print.htm 516. Epidemiology can be defined as: a. A study of special areas of the skin. b. The study of the distribution and determinant of disease in man. *** c. Study of biological animals. d. Study of disease in research laboratory. 517. Most common Benign Tumer in oral cavity is:

a. Fibroma (Ameloblastic fibroma). *** 518. Which of the following spaces are bilaterally involved in Ludwig's angina? a. Submandibular + masticatory spaces b. Sublingual+Lat.Pha.space c. Submandibular+sublingual+submental. *** Dental Decks - page 1554 And Ludwig angina remains a potentially lethal disease, rapidly spreading bilateral cellulitis of the submental, sublingual, and submandibular spaces. 519. When you do amalgam finishing..... a. Immediately. b. 24 hours later. *** 520. When polishing amalgam restoration . a. Avoid heat generation by using wet polishing paste b. Wait for 24 hours c. A & b. *** d. A only e. B only 521. The roof of mandibular fossa consist of: a. Thin compact bone. *** b. Spongy bone. c. Cancellous bone. 522. Neoplasm that spread by Lymphatic from the angle of the mouth reach the: a. Preauricular Lymph nodes. b. Mental Lymph nodes. c. Submandibular Lymph nodes. *** d. Pterygoid plexus. e. Jugulo-digastric nodes. - Cancer Medicine A standard rule of thumb is that the lymphatic drainage for any particular region is predicted by the arterial supply of that region. The lip, cheek, and anterior gingiva drain to submandibular and submental lymph node groups. In addition, the cheek and upper lip also drain to inferior parotid nodes. 523. A plastic anemia is caused by: a. Tetracycline. b. Penicillin. c. Erythromycin.

d. Sulfonamide. *** Aplastic Anemia Roughly half of all aplastic anemias occur as a result of drugs (such antibiotics as chloramphenicol, sulfonamides, phenylbutazone [Butazolidin], and such anticonvulsant agents as mephenytoin) Also: chloramphenicol, phenylbutazone [Butazolidin], and such anticonvulsant agents as mephenytoin. 524. Odontogenic infection can cause least complication: a. Pulmonary abscess. b. Peritonitis. c. Prosthetic valve infection. d. Cavernous sinus thrombosis. Dental Secrets page 260 What are the significant complications of untreated Odontogenic infection? Tooth loss Spread to the cavernous sinus and brain Spread to the neck with large vein complications Spread to potential fascial spaces with compromise of the airway Septic shock 525. Cavernous sinus thrombosis not manifested as: infra orbital syndrome. Syncope due to atrial obliteration. *** eye exophthalmos. MCQs in Dentistry Cawson page 131 Cavernous sinus thrombosis: may follow infection from the ptyrogid venous plexus. Dental Secrets page 263 Cavernous sinus thrombosis: Patients present with proptosis (eye exophthalmos), orbital swelling, neurologic signs, and fever. 526. After u inject L.A for 2nd max molar pt become colorless with external swelling its due to : 1/facial artery. 2/ plexus vein. *** 3/ Posterior alveolar Nerve. Hand Book Local Anasthesia page 168

Hematoma: This is commonly produced by inserting the needle too far posteriorly into the pterygoid plexus of veins. Additionly, the maxillary artery me be perforated. 527. Pass throw parotid gland: a. Facial nerve. *** b. Facial arteries. c. External carotid veins.
If needle is positioned too posteriorly, anaesthetic may be put into parotid gland (dangerous systemic effects), or paralyse Cranial Nerve VII (7), medially the medial pterygoid muscle can be injected, resulting in trismus. alveolar nerve block resulting in Bells Palsy-like symptoms. Also if the needle is placed too The sphenomandibular ligament is most often damaged in an inferior

http://en.wikipedia.org/wiki/Inferior_alveolar_nerve_anaesthesi a#Injection_techniques 528. Endomethasone is a root canal sealer that: a. Dissolve in fluid so it weaken the root canal filling. b. Very toxic contain formaldehyde. c. Contain corticosteroids. d. All the above. *** 529. Cause that master G.P not reach working length although it is the same size of last file: a. Dentin debris. b. Ledge formation. c. A & b. *** d. None of the above Endodontics Problem solving in clinical practice 2002 page 135 Master Cone Will Not Fit to Length: Dentine chips packed into the apical extent of the root canal preparation will lead to a decrease in working length, and consequently the master cone will appear to be short. This can be avoided by using copious amounts of irrigant during preparation. A ledge in the root canal wall can prevent correct placement of the cone. If the cone hits an obstruction during placement then the end may appear crinkled. It may be possible to remove or smooth a ledge by refining the preparation with a greater taper instrument. If the canal is insufficiently tapered, the master cone may not fit correctly because it is binding against the canal walls coronally or in the mid-third. The

completed root canal preparation should follow a gradual taper along its entire length. Further preparation may be required with Gates-Glidden burs, orifice shapers or a greater taper instrument. 530. Small access opening in upper centeral incisor lead to: a. Complete removal of the pulp. b. Incomplete removal of the pulp. *** c. Conservative restoration. 531. In sickle cell anemia O2 decreased in oral mucosa: a. True. *** b. False 532. We distinguish between periapical and periodontal abscess: a. X-ray examination b. Clinical examination c. Vitality of the pulp. *** 533. How can you prevent dental hyper sensitivity: a. Restoration by adhesion. *** b. Controlled by alcohol c. Put sedative medication 534. A U- shaped radiopaque structure in the upper 1st molar x-ray is: a. The zygomatic process. *** b. Maxillary sinus wall "Dental Radiographic Diagnosis by Dr. Thunthy - page 44" And Dental Decks page 150 535. Loss of sensation in the anterior 2/3 of the tongue is related to paralysis of: a. Lingual nerve. *** b. Hypoglossal nerve. c. Chorda tympani nerve. Dental Decks for tha ant. 2/3: lingual n. for the sensation & chorda tympani n. for the taste for the post. 1/3: both taste & sensation by glossopharngeal n ( ) ( ) . 536. The choice of local anesthesia depend on: a. Diameter of the nerve b. Structure of the bone c. Number of branches

d. Type of L.A agent chemistry. *** . 537. Choice of local anesthesia technique influenced by: a) Chemical composition of anesthesia. B) The location of the nerve. C) Bone structure. *** Dental decks - page 1904 The bone of the maxilla is more porous than that of the mandible,therefore it can be infiltrated anywhere. 538. Mandibular foramen in young children is: a. At level of occlusal plane. b. Above the level of occlusal plane. c. Anterior the level of occlusal plane. d. Below the level of occlusal plane. *** PAEDIATRIC DENTISTRY - 3rd Ed. (2005) page 98 In children, the mandibular foramen is low in relation to the occlusal plane 539. When you give inferior dental block for pedo Pt the angulations for the needle a. 7 mm below the occlusal plane. b. 5 mm below the occlusal plane. c. 7 mm above the occlusal plane. d. At the occlusal plane. *** pedo PAEDIATRIC DENTISTRY - 3rd Ed. (2005) page 98 Inferior alveolar and lingual nerve blocks: The height of insertion is about 5 mm above the mandibular occlusal plane, although in young children entry at the height of the occlusal plane should also be successful. And Restorative Techniques in Pediatric Dentistry page 25 The needle enters the tissues at a point midway between the external oblique ridge and the pterygomandibular raphe at the level of the occlusal plane. And Pediatric dentistry infancy through adolescence, pinkham, 3rd edition, page 416

Needle must be inserted at the level of occlusal plane And McDnald 7th ed page 285 The mandibular foramen is situated at a level lower than the occlusal plane of the primary teeth, therefore the injection must be made slightly lower and more posteriorly than for an adult patient. 540. When you want to give inferior alveolar block for a child you have to take attention that the mandibular foramine is : a. At level of occlusal plane. b.Above the level of occlusal plane. c. Anterior the level of occlusal plane. d. Below the level of occlusal plane .*** ) ( The mandibular foramen was located 4.12 mm below the occlusal plane at the age of 3. It subsequently moved upward with age. By the age of 9, it had reached approximately the same level as the occlusal plane. The foramen continued to move upward to 4.16 mm above the occlusal plane in the adult group. The height percentage averages ranged from the lower 1/3 of the ramus height in the 3 year-old group to the middle of the ramus height in adults. The depth percentage averages ranged from 67.8% in 3 year-old children to 61.7% in adults. For greater accuracy in anesthetic procedures, dentists should relate the locational changes in the mandibular foramen with .age when performing block anesthesia for the inferior alveolar nerve

http://forums.studentdoctor.net/showthread.php?t=550408 mandibular foramen is below the occlusal plane in children but in adults it is above the occlusal plane and post to molars

541. The cell primary site of ATP production is: a. Mitochondria. *** b. Lysosomes. c. Nucleus. d. Nucleolus.

e. Vacuoles. 542. The organelle most closely associated with the manufacture of proteins within the cell: a. Ribosome. *** b. Lysosome. c. Nucleolus. d. Cell wall. e. Cell membrane. 543. The packing and sorting of protein is the function of: a. Endoplasmic reticulum. b. Golgi apparatus *** c. Mitochondria d. Nucleus 544. The process of attraction of neutrophils to a site of Local tissue injury is called: a. Phagocytosis. b. Diapedesis. c. Chemotaxis. *** d. Epistaxis. 545. Action of Histamine: a. Vasodilatation. b. Permeability. c. Chemokinesis. d. Broncho. ALL OF THE ABOVE 546. Cholesterol crystals are found in: a. Keratocyst. *** b. Periodontal cyst MASTER DENTISTRY- Oral and Maxillofacial, Surgery, Radilolgy, Pathology and Oral Medicine radicular cysts contain brown shimmering fluid because of the presence of the cholesterol crystals, whereas odontogenic keratocysts contain pale greasy fluid, which may include keratotic squames. http://ses.library.usyd.edu.au/bitst...z%20Iqubal.pdf Cholesterol crystals are found in many odontogenic cysts including Radicular cysts, dentigerous cysts, and odontogenic keratocysts. And Probable keratocyst in a mandible from the late Roman era -- Chimenos-K

The cyst wall shows nicks produced by cholesterol crystals, inflammatory cells, calcifications and satellite microcysts. Apical periodontal cyst or Radicular cyst Odontogenic keratocyst . 547. The process of cell engulfing particle is called: a. Endocytosis. b. Exocytosis. c. Phagocytosis. *** d. Pinocytosis.
Phagocytosis (from Ancient Greek (phagein) , meaning "to is the cellular process of engulfing solid particles by the cell devour", , (kytos) , meaning " cell", and -osis, meaning "process") membrane to form an internal phagosome by phagocytes and protists. Phagocytosis is a specific form of endocytosis involving the vesicular internalization of solids such as bacteria, and is, therefore, distinct of various liquids. Phagocytosis is involved in the acquisition of from other forms of endocytosis such as the vesicular internalization nutrients for some cells, and, in the immune system, it is a major mechanism used to remove pathogens and cell debris. Bacteria, dead tissue cells, and small mineral particles are all examples of objects that may be phagocytosed.

http://en.wikipedia.org/wiki/Phagocytosis

548.

The Most common odontogenic cysts in the jaws are:

a. Radicular cyst. *** b. Keratocyst. 549. Most commonly dentigerous cysts are associated with: a. Unerupted permanent maxillary canines b. Unerupted mandibular third molars. *** Spontaneous regression of bilateral dentigerous cysts associated with impacted mandibular third molars : Article : British Dental Journal It is most prevalent in the third molar region and is therefore a common cause of a radiolucency associated with the crown of an impacted third molar Bilateral Dentigerous Cysts - Report of an Unusual Case and Review of the Literature although it may involve any tooth, the mandibular third molars are the most commonly affected. 550. Histopathologically, dentigerous cyst Lining epithelium may be: a. Cuboidal in type. b. Stratified squamous in type. *** c. Reduced enamel epithelium. d. All of the above. Oral pathology clinical pathologic correlation,3rd edition, Page : 294 551. Thyroglossal duct cysts: a. Are only found in the posterior tongue. b. Clinically present in the Lateral neck tissue. c. May be found anywhere along the pathway of the embryonic thyroglossal duct. *** d. Are sometimes called Lympho-epithelial cysts. Oral pathology clinical pathologic correlation,3rd edition, Page 316 Most cyst occur in the midline ,with 60% over the thyroid membrane and only 2% within the tongue it self. Residual epithelial elements that do not completely atrophy may give rise to cysts in later life. 552. Unilateral swelling + slowly progressing Lesion on the Left side of the mandible. This could be: a. Osteoma. b. Cementoblastoma. c. Ossifying Fibroma. *** d. Osteo-sarcom. Oral pathology clinical pathologic correlation,3rd edition, Page 357

a. b. c. d.

The ossifying fibroma is a slow growing ,expansile lesion that is usually a symptomatic 553. Toothgerm of primary teeth arise from: Dental lamina. *** Dental follicle. Enamel organ. Epithelial cell of malassez. http://en.wikipedia.org/wiki/Dental_lamina The dental lamina is a band of epithelial tissue seen in histologic sections of a developing tooth. The dental lamina is first evidence of tooth development and begins at the sixth week in utero or three weeks after the rupture of the buccopharyngeal membrane.

554. Apical periodontal cyst arise from: a. Hertwig sheath. b. Epithelial cell rest of malassez. *** CLINICAL PATHOLOGICAL CORRELATIONS, 4TH EDITION, PAGE 241 Apical periodontal cyst= periapical cyst = radicular cyst: these inflammatory cysts derive their epithelial lining from the proliferation of small odontogenic epithelial residues (rests of Malassez) within the PDL. .malassaz . 555. Formation of periodontal cyst due to: a. Nasolacrimal cyst. b. Hertiwigs. *** c. Epithelial rest of malassaz. d. Peals of serres. Oral Pathology - 4th Ed. (2005) J. V. Soames Professor of Oral Pathology University of Newcastle upon Tyne UK And J. C. Southam Emeritus Professor of Oral Medicine and Oral Pathology University of Edinburgh UK http://obm.quintessenz.de/index.php?doc=html&abstractID=9558 http://en.wikipedia.org/wiki/Epithelial_cell_rests_of_Malassez In dentistry, the epithelial cell rests of Malassez or epithelial rests of Malassez (frequently abbreviated as ERM) are part of the periodontal ligament cells around a tooth. They are discrete clusters of residual cells

from Hertwig's epithelial root sheath (HERS) that didn't completely disappear. It is considered that these cell rests proliferate to form epithelial lining of various odontogenic cysts such as radicular cyst under the influence of various stimuli. They are named after Louis-Charles Malassez (18421909) who described them. Some rests become calcified in the periodontal ligament (cementicles) . 556. Which is the most Likely cause of periodontal cyst? a. Cell Rest of Malassez. b. Cell rest of serss. c. Cell of Hertwig sheath. *** Dental secrets page 66 19. What is the difference between a lateral radicular cyst and a lateral periodontal cyst? A lateral radicular cyst is an inflammatory cyst in which the epithelium is derived from rests of Malassez (like a periapical or apical radicular cyst). It is in a lateral rather than an apical location because the inflammatory stimulus is emanating from a lateral canal. The associated tooth is always nonvital. The lateral periondontal cyst is a developmental cyst in which the epithelium probably is derived from rests of dental lamina. It is usually located between the mandibular premolars, which are vital. malasseze >>>> apical serris >>>> lateral hertwing >>>> peridontal

557. Primary malignant melanoma of the oral mucosa: a. Always originates within the surface epithelium. b. Mostly originates within the surface epithelium. c. Always originates from nevus cells in the connective tissue. *** d. Always originates from Langerhans cells within epithelium. ) ( 558. Histopathologically adenoid cystic carcinoma in characterized by islands of:

a. Basophilic islands of tumor cells that are intermingled with areas of pseudocartilage. b. Basophilic islands of tumor cells having a "Swiss cheese" appearance. *** c. Basophilic islands of tumor cells having a "Swiss cheese" appearance and evidence of serous acini. d. Basophilic islands of tumor cells that contain mucin and normal acini. 559. The risk of malignant change being present in epithelium is greatest in: a. Homogenous Leukoplakia b. Erythroplakia. *** c. Chronic hyperplasic candidiasis d. Speckled Leukoplakia Erythroplakia should be viewed as a more serious lesion because of a significantly higher percentage of malignancies associated with it 560. The term acanthosis refers to: a. A decreased production of keratin. b. An increased production of keratin. c. An increased thickness of the prickle cell zone (stratum spinosum). *** d. None of the above. WordNet Search - 3.0 Acanthosis: Thickening of the epidermis and elongation of the rete ridges due to thickening of the spinous layer. May be associated with enlargement of rete pegs. an abnormal but benign thickening of the prickle-cell layer of the skin (as in psoriasis) 561. The most common malignant tumors of the minor salivary glands are: a. Adenoid cystic carcinoma and adenocarcinoma b. Adenoid cystic carcinoma and acinic cell carcinoma c. Mucoepidermoid carcinoma and adenoid cystic carcinoma. *** d. Mucoepidermoid carcinoma and polymorphous Low grade adenocarcinoma Arch Otolaryngol Head Neck Surg -- Malignant Minor Salivary Gland Tumors of the Larynx, July 2006, Ganly et al. 132 (7): 767 The most common malignant minor salivary gland tumors are adenoid cystic and mucoepidermoid carcinomas. 562. Mandibular branch of trigeminal nerve leaves the skull through: a. Foramen rotundum. ) ( b. Foramen ovale. *** ) ( c. Superior orbital fissure. d. Inferior orbital fissure. e. Jugular foramen.

563.

Foramen oval is in the following bone: a. Temporal. b. Occipital. *** c. Sphenoid. 564. The inferior alveolar nerve is branch of: ***1. Mandibular nerve not divided 2. Posterior mandibular alveolar nerve. 3. Anterior mandibular alveolar nerve. () ()

565. The following structures open into the middle meatus: a. Nasolacrimal duct. b. Posterior ethmoidal sinus. c. Maxillary sinus. d. Sphenoid sinus. e. Anterior ethmoidal sinus. f. A, b & d. g. A & b. *** h. C & e. i. All of the above : . : . : . : . 566. Ligaments associated with TMJ: a. Tempromandibular. b. Sphenomandibular.

c. Stylomandibular. d. All of the above. *** Ref *: The lateral temporamandibular ligament: limits the movement of the mandible in a posterior direction. The sphenomandibular ligament lies on the medial side of the joint. The stylomandibular ligament lies behind and medial to the joint. ) ( . 567. Location to give inferior alveolar nerve block the landmarks are: 1/ pterygomandibular raphe 2/ cronoid notch 3/ all of the above. *** 568. The optic foramen canal is a part of: A)Frontal bone B)Sphenoid bone. *** C)Esthmoid bone 569. Optic nerve coming from which bone: - sphenoid bone - zygomatic - palatal ) ( 570. The following cavity bases are moisture sensitive: a. Polycarboxylate b. Zinc phosphate c. GI cem. *** d. ZOE e. A, c. Dental Decks - page 2060 . . 539; Restorative dental materials 2002 page 615 Glass ionomer cements are very sensitive to contact with water during setting. The field must be isolated completely. Once the cement has achieved its initial set (about 7 minutes), coat the cement margins with the coating agent supplied with the cement. 571. Which of the following types of base materials can be placed in contact with polymethyl methaacrylate & not inhibit the polymerization of the resin:

a. b. c. d. e.

ZOE GI cement Zn phosphate cement Varnish B, c. *** Dental Decks - page 2076 - 2102

572. Cement which contains fluoride: a. GI. *** b. ZOE. c. Reinforced ZOE. d. Polycarboxylate cement. 573. Marginal deterioration of amalgam restoration should be due to: a. No enough bulk of dentine. b. Corrosion. c. Over carving. d. Improper manipulation of amalgam. e. A and b. f. C and d. g. All the above. *** h. B, c and d. Art and science of operative dentistry 2000 page 157 Amalgams that are corroded or have inadequate bulk to distribute stresses may fracture. At margins, where amalgams are thinner, extrusion may have occurred, and corrosion may have compromised the integrity of the amalgam, fracture is even more likely. <3 ( ) . ( ) . These objectives help to conserve the dentinal support and strength of the tooth, and they aid in establishing an enamel cavosurface angle as close as possible to 90 degrees . They also help to minimize marginal deterioration of the restoration by locating the margins away from enamel eminencies where occlusal forces may be concentrated. 574. A restoration of anterior teeth with RCT, abraded incisal edge & small M&D caries is by: a. Ceramometal crown. *** b. Composite laminated.

c. Veneer. d. None of the above.

575. The powder for GI cement contain: a. Sio2, Al2o3, caf. *** b. Sio2,zno, barium sulphate c. None of the above. GIC Powder: Silica 41.9% - Alumina 28.6% - Calcium Fluoride 15.7% - Sodium Fluoride 9.3% - Aluminium Phosphate 3.8% - Aluminium Fluoride 1.6%

a. b. a. b.
a. b. c. d. e.

576. The body secret antibody against antigen using which cells: T lymphocyte B lymphocyte 577. In diabetic patient, periodontium affected by which cells: Neutrophil. pmns Macrophages 578. When take an x-ray to pregnant lady, we use all of this method EXCEPT: Digital x-ray. High sensitive film. Paralleling tech (Long cone) 16 inch. Bisecting algle (short cone) 8 inch. *** Lead apron with thyroid collar. Dental decks page 4 the 8 inch technique exposes more tissue by producing divergent beam. : Dental Decks - page 48 The intensity of the radiation is inversely proportional to the square of the distance. .

a. a. b. c. d.

579. When take x-ray we should stand: 6 feet away in 90-135 angle. 580. Proximal caries should be opened when: Confined within enamel. Pass DE junction. *** Dentin laterally All of the above "Sturdevant's Art & Science of Operative Dentistry"


a. b. c. a. b.

581. In a study, it should? Protect you against role of the statistician Protect you against legal risks Protect against physical risks. 582. Cause of angular cheilitis: Loss vertical dimension Pt have complete denture. *** Autoimmune factors. "Dental secrets" . "Atlas Of Oral Medicine"

. " " ( ) 583. Ugly duckling stage: a. 9-11 years old. b. 13-15 years old. c. 7-9 years old. . 1. 2. 3.
a. b.

a. b. c.

584. Eruption of primary dentition starts from: 6-7 months.*** 1 year. 9 months. 585. Component of Gutta percha: 50% Gp & 50% ZOE 20% Gp & 70% ZOE. *** Elsevier: Article Locator + Endodontics : : inorganic )Gutta percha & barium sulfate( %:8 organic (gutta percha) %56 transpolyisoprene 586. All are irrigation for canals EXCEPT: Saline. Hydrogen beroxide. Naocl

d. RC prep. ***

EDTA 587. Most common bacteria causing caries: Streptococcal mutans. ***
a. b. c.

a. b. a. b. c.

a. b. c. d.

a. b. c. d.

2.

a.

588. Proximal caries confined to enamel: Prevention. *** Observation Restore with G I 589. In community diagnosis and treatment program: Water flouridation Diagnose, prevent, treat. *** 590. Porcelain shrinkage after firing: 1-5% 5-10% 10-20% At a firing temperature of 1400 C, they shrink by 16 percent %73 63 :;6 591. The cement under MOD amalgam have this character: High modulus of elasticity(stiff)*** Low modulus of elasticity ( stiffness) The high modulus of elasticity prevent of bonding and decrease tensile strength. Both a &c :86 Flexibility Stiffness "Sturdevant's Art & Science of Operative Dentistry" Page 479 592. Examination of Pt health by the dentist: To know the patients health. To know what medications to give. To know general health data. All of the above. *** 593. 2 statement true or false: 1. RCT abutment of FPD has higher risk for fracture. Abutment which has RCT in cantilever FPD have higher susceptibility to fracture. a. 1st is true,2nd is false b. 1st is false,2nd is true. c. Both are false d. Both are true. *** 594. Both glass ionomer & polycarpoxylate cement contain: Polyacrylic acid. ***

b. ZOE powder. 595. Most common cyst oral cavity: a. Radicular cyst. b. Peridontal cyst. ) ( . (. ) http: //www.lsusd.lsuhsc.edu/faculty/...the%20Jaws.pdf :5 a. b. c. d. a. b. 596. Factors delay healing of wound: Infection. Torn wound edges. Strain. All of the above. *** 597. Factor interfere with healing: Poor suturing Infection. *** Dental Decks - page 1792 Healing occur more rapidly with a lower risk of infection. 598. Dry socket happen after: 24 h 3-5days. *** 1week 2weeks Pigmentum ( Whitehead Varnish . ) Iodoform Compositum) (B.P.C

a. b. c. d.

599. Compared to herpetic ulcers...aphthous ulcers are: a. Small size. b. In mucosa lining. *** c. Leave scars. . 600. Avulsion more important factor that affect reimplantation: a. Contaminated roots. b. Time since the avulsion. Dental secrets 601. Nicotine stomatitis:

a. Palate.hyper?? b. Hyperplasia - c. Prickle cell like shape prominent base***

Dental decks 2004 a. b. c. d. 602. Dentinogenesis imperfecta have all EXCEPT: Broken enamel. Blue sclera. Broken bone. Supernumerary teeth. *** Symptoms of Dentinogenesis imperfecta, type I The list of signs and symptoms mentioned in various sources for Dentinogenesis imperfecta, type I includes the 14 symptoms listed below: Bluish-gray teeth - Amber-colored teeth - Bulbous teeth crowns Absent tooth roots canals - pulp chambers Too small tooth roots - canals - pulp chambers Enamel separation from the ivory (dentin) Misaligned teeth - Recurring dental abscess - Brittle bones - Blue sclera 603. Generalized gray discoloration in a 28 years old patients teeth, with blue sclera and an enlarged pulp chambers and short roots, and multiple fractures in Enamel the diagnosis is : A) Dentinogenesis Imperfecta. *** B) Amelogenesis Imperfecta . Dental Decks - page 998 6 . http://en.wikipedia.org/wiki/Dentinogenesis_imperfecta Type I and II show total obliteration of the pulp chamber. Type III shows thin dentin and extremely enormous pulp chamber.These teeth are usually known as Shell Teeth. http://en.wikipedia.org/wiki/Dentin_dysplasia Type 1: Roots are short, blunt and conical. In deciduous teeth, pulp chambers and root canals are completely obliterated in permanent they may be crescent shaped. Type 2: The pulp chamber of the deciduous teeth become obliterated in deciduous teeth. While in permanent teeth, large pulp chamber is seen in coronal portion of the tooth - referred to as thistle tube appearance.Pulp stones may be found.

604. 30 years old pt came to the clinic with brownish discoloration of all his teeth (intrinsic discoloration) & yellowish in U/V light the most likely cause is: 1/ flourosis 2/ tetracycline. *** 3/ amelogensis imperfecta 4/ dentogensis imperfectea 605. Most sign of fracture of mandible: a. Nose bleeding. b. Malocclusion. *** c. Parasthesia. . mcqs In Dentistry 606. What supply the gingival buccal tissue of premolars, canines and incisors: Long buccal. Inferior alveolar nerve. *** Superior alveolar nerve. : . . : : . 607. Drainage of tip of the tongue: Submandibular lymph nodesSubmental Oral pathology Regezi 3rd edition - page 72 Metastases from the tongue cancer.....the 1st nodes to become involved are the submandibular or jugulodigastric. 608. Cementum in cervical 2/3 have: a. Acellular intrinsic fiber.

a. b. c.

a. b.

b. Acellular extrinsic fiber. *** c. Cellular mixed fibers. d. Intermediate cementumDental Decks - page 836 PAGE 15Periodontology a. b. c. d. 609. Pins are insert into: Enamel. Dentin. *** Enamel and dentin (DEJ). Any of the above .

a. b. c. d.

a. b. c. a. b. c.

Dental secrets : 4 5 610. After etch enamel and bond it with 5th generation the strength of? 5-10Mp. 25Mp. 30Mp. 100Mp. 68 611. Composite restoration that was matching in shade, after one week it became much light... The reason could be: light started photoinitation. Absorption water. Shade selected after rubber dam. *** 612. Disadvantage of digital x-ray EXCEPT: Large disk space Storage Clarity and resolution. *** Expensive .

613. Treatment of fungal infections: a. Penicillin b. Tetracyclin c. Nystatin. *** Oral thrush (fungal infection in the mouth) Dental Decks - page 2454 : a. b. a. b. 614. Properties of ideal endo obturation material are all EXCEPT: Biocompatible. Radiolucent. *** 615. Most difficult of extract: Mand. 3rd molar with mesioangular fused roots Mand 3rd molar with distoangular angulation with divergent curve roots *** . 616. Pt have hyperventilation in clinic. Most cause:

a. Reduced of CO2 b. Increase CO2 c. Anxiety. *** 617. Very important part in endo treatment: a. Complete debridement of the canal 618. Perio endo lesion the primary treatment: a. Endo treatment. *** b. Deep scaling and root planning. Dental decks - page 216 619. Contraindication to extraction: a. Cardiac pt. b. Previous recent radio therapy. *** Dental secrets : . - 620. Base of the flap should be wide for: a. Healing b. Better blood supply to the wound. "flap design should ensure adequate blood supply; the base of the flap should be larger than the apex" Dental decks 1754 621. Supra calculus all true EXCEPT: a. Hard and rough *** b. Easy to detach c. Has component of saliva . 622. Thickness of luting cement: ?? a. 100 micrometer b. 40micro meter c. 1mm "http:/tdc.thailis.or.th/tdc/browse.php?Option=show&browse_type=title&titl eid=160951&display=list_subject&q=Adhesives" http: //tdc.thailis.or.th/tdc/browse.php?Option=show&browse_type=title&titleid= 160951&display=list_subject&q=Adhesives . 73 . 5-4 58-48 .

-Cements for luting have desired film thickness of 15: 25 microns. -Cavity Liners(either solution or suspension liners) have a desired film thickness of 5 microns -Bases have a final application thickness of 1-2mm(they may be thicker depending on the amount of dentin that may be destroyed) :Source Dental Decks Part 2,2007-2008 page2072 The maximum allowable thickness is 25 m (ADA specification No. 96) Dental Cements The low 12-micron film thickness is ideally suited for luting applications a. b. c. a. b. a. b. c. 623. Formacresol used in: Full concentration 5th concentration One fifth concentration. *** 624. Zinc phosphate cement: Mechanical attachment *** Chemical 625. Traditional Glass ionomer: Mechanical bonding. Acid-base reaction *** Mechanical chemical bonding. Dental Decks - page 2060 . 626. Pontic design of an FPD: Same size buccolingually of the missing tooth Smaller than missing buccolingually. *** Wider buccolingually None of the above .

a. b. c. d.

627. False negative response of an electric pulp test given: a. After trauma b. Periodontal disease c. In teenager. ( ) : - .

Attachment apparatus . 628. Young with open apex examination test: a. Reliable. b. Non reliable. *** c. None of the above. 629. Best media for the avulsed tooth: a. Tap water. b. HBSS (Hank's balanced salt solution). *** c. Saliva. d. Milk. Dental secrets . Viaspan 630. Rigid palatal strap major connector. The material of construction is .co-cr *** Gold ti .gold .. .wrought wire 631. the use of low speed hand piece in removal of soft caries in children is better than high speed because a. .less vibration b. .less pulp exposure. *** c. .better than high speed ) ( 632. Progression of initial caries to cavitations takes 18 month this based on a .strepto.mutans initiate caries. *** b .lactobacillia progress caries . 4; 633. The vertical fracture of the tooth detected by .periodontal pocket. *** .radiographically .vertical percussion Dental secrets page 161 Dental Decks - page 135 1. Transillumination with fiberoptic light

2. Persistent periodontal defects in otherwise healthy teeth 3. Wedging and staining of defects 4. Radiographs rarely show vertical fractures but do show a radiolucent Defect laterally from sulcus to apex (which can be probed). And: "Clinical Endodontics textbook TRONSTAD page 57" Vertical Fracture A vertical fracture of a tooth may result in communication between the gingival sulcus (oral cavity) and the apical periodontium. The fracture line will be a portal of entry for bacteria from the mouth into the tissues, causing inflammation and bone destruction. A periapical lesion often forms which may have the appearance radiographically of a pulpally related lesion. Since a vertical fracture may be incomplete, its diagnosis in many instances is extremely difficult. Sometimes it is recognized clinically because a periodontal pocket forms along the fracture line and in other instances a simple exploratory surgical procedure may aid in establishing the correct diagnosis. And: http://www.doctorspiller.com/Cracked_Teeth.htm 634. Principle of elevator use of all the following EXCEPT 1) wheel and axle. 2) wedging the socket wall. *** 3) wedging. 4)lever. Textbook for General and Oral Surgery page 193 Dental elevators work either on the principle of 'block and wedge' or 'wheel and axle', and should never be used as crow-bars (Fig. 23.4). Hence, a dental luxator with its sharp edge is pushed between the root of a tooth and its alveolar bone via the periodontal space. This wedging effect should cause the root to be moved from its socket Oral Surgery- pgs 119-155 Mechanical principles involved in extraction- Lever, wedge, & wheel and axle 1- Lever- Elevators are used primarily as levers -transmit modest force- w/ mechanical adv. of long lever arm and short effector arm- into sm mvmt against greatest resistance -use a purchase point and crane pick to elevate tooth from socket 2- Wedge-

-force tips of foceps into PDL space to expand bone and force the tooth out of the socket -useful when a str8 elevator is used to luxate a tooth from its socket, sm elevator forced into space, displaces the root toward the occlusal and out of the socket 3- Wheel and Axle-triangular or pennent shaped elevator -when one root is left, pennent elevator put into socket and turned -handle is an axle, tip of triangular elevator is whell and engages & elevates the root from the socket 635. To kill HIV use all of the following EXCEPT .naocl .ultraviolet chamber. *** .autoclave .chimoclave 636. Patient with amalgam usually complain of pain with .cold. *** .galvanic . Hot 637. Radiographic diagnosis of bilateral expansile radioopaque areas in the canine premolar area region of the mandible is a) Hematoma b) Remaining roots c) Torus mandibularis *** d) Internal oblique ridge e) Genial tubercle 638. Pain of short duration with hot and cold .dentin sensitivity. *** .irriversible pulpitis. .chronic pulpitis .apical periodontitis 639. shade guide: Under light Dry tooth None of above. *** 640. When do we do incision and drainage? A. Indurated diffuse swelling. b. Sinus tract c. Chronic apical periodontitis

641. Pregnant lady needs oral surgery: a. Needs prophylactic antibiotic. B. Needs under GA c. Needs steroid cover d. None of the above. *** 642. When do we give antibiotic: a. Widespread, rapid infection b. Compromised host defence c. . D. A&b 643. Tooth requires RCT with bone resorption. Terminate RCT at: a. Radiographic apex b. 0.5-1 mm short of radiographic apex. c. 0.5-1 mm beyond radiographic apex d. "ENDODONTICS Fifth Edition page 515" Weines recommendations for determining working length based on radiographic evidence of root/bone resorption. A, If no root or bone resorption is evident, preparation should terminate 1.0 mm from the apical foramen. B, If bone resorption is apparent but there is no root resorption, shorten the length by 1.5 mm. C, If both root and bone resorption are apparent, shorten the length by 2.0 mm. Color atlas of endodontics page 54 Some researchers suggest calculating the working length 1 mm short of the radiographic apex with normal apical anatomy, 1.5 mm short with bone but no root resorption, and 2 mm short with bone and root resorption. 644. Muscle that form floor of the mouth: a. Mylohyoid. *** b. 645. Organism that initiates caries: a. S. Mutants. *** b. c. 646. Incipient caries a. Surface zone is relatively unaffected. *** b. The surface zone is the largest portion with the highest pore volume c. Tooth preparation and composite is the best treatment. D. Pulpal reaction is not possible. E. Caries progress in enamel faster than dentin. Dental decks 2080 Surface zone relatively unaffected by the carious attack

647. Important factor in long term success of perio treatment: a. Skill of the operator b. Perio maintenance c. 648. Which causes gingival enlargement: a. b. Cyclosporines c. d. 649. Pedo use rubber dam for a. Improve visibility and access b. Lowers risk of swallowing c. Sterile field d. A & b. *** 650. Root most commonly pushed in max sinus a. Buccal of 7 b. Palatal of 6 *** c. Palatal of 7 d. Buccal of 6 e. Distal of 6 & 7 Dental decks 1816 The palatal root of the maxillary first molar is most often dislodged into the maxillary sinus during an extraction procedure. 651. If tooth or root is pushed during surgical extraction into maxillary sinus a) Leave it and inform the patient b) Remove it as soon as possible c) Follow the patient for 3 months d) None of the above 652. Cementum is formed from a. Cementoblasts *** b. Fibroblasts c. Cementicles d. .. 653. Teeth have convexity in buccal and lingual a. Upper premolars. *** b. .. 654. Body defends itself by antibodies from a. B lymphocytes *** b. T lymphocytes c.Plasma cell

655. cell that produce antibodies a. B lymphocytes *** b. T lymphocytes c.Plasma cell 656. Hyperventilation in dental office: a. Anxiety *** b. Low CO2 c. High CO2 657. Avulsed teeth stored in a. Milk *** b. Water c. 658. The best midea for keeping the avulsed teeth is: a. Water in the same temperature of room b. Milk in the same temperature of room c. Cold water d. Cold milk 659. The depth of cavity prep for composite in posterior: a. Limited to enamel b. 0.5 mm in dentin c. Depends on caries extension *** d. Depends on tooth discoloration e. 0.2 mm in dentin 660. Factors that make impaction surgery more difficult: a. Mesioangular position, large follicle, wide periodontal ligament and fused conical roots. B. Mesioangular position, large follicle, wide periodontal ligament and curved roots. C. Distoangular position, large follicle, wide periodontal ligament and fused conical rooths d. Distoangular position, thin follicle, narrow periodontal ligament and divergent curved roots. *** E. Soft tissue impaction, separated from second molar and inferior alveolar nerve. 661. Which scalpel below is universally used for oral surgical procedures? A. Number 2 blade. B. Number 6 blade. C. Number 10 blade. D. Number 12 blade. E. Number 15 blade. *** 662. Main disadvantage of chlorhexidine: a. Staining. ***

b. Burning sensation. c. Altered taste.

chlorohexidine has several disadvantages. One of the most important disadvantages is that if a patient rinses his mouth with chlorohexidine compositions regularly, his teeth and tongue obtain a brownish colour. This is off course a major disadvantage. Another disadvantage of chlorohexidine is that it has no significant anti-bacterial effect on gram positive bacteria at relatively low concentrations. This means that gram positive bacteria will not be effected by the rinse composition and may thus still cause periodontitis or produce the volatile sulphur compounds that cause the malodour. 663. The radiograph shows condylar head orientation and facial symmetry a. Submentovertex b. Reverse town *** c. Opg d. Transorbital. "US Army medical course - Dental Radiography page 376"


664. The best way of radiograph shows displacement of mandibular conyle a. Reverse town *** b. Oplaqe horizontal 30 665. what kinds of radiographs which we do not use for TMJ movements? A- transcranial b-computerized t c-conventional t d-arthrography 666. To check TMJ range of movement: a) cranial imagery B) arthrography *** c) traditional tomography d) computerized tomography " oxford handbook of clinical dentistry 4ed 2005 oxford up - mitchell david Mitchell"

...
667. To check a perforation in the desk of the tmj we need:

A) cranial imagery B) arthrography. *** (CT after injection of a high contrast fluid) C) traditional tomography D) computerized tomography. 668. Zinc phosphate cement and polycarboxylic cement both have a. Zinc oxide particles. *** b. Silica quartz particles c. Polyarcyilic acid d. Phosphoric acid 669. Pedo, has trauma in 11 , half an hour ago , with slight apical exposure , open apex, treatment is: a. Pulpotomy with formacresol b. Apexification c. DPC (direct pulp capping). *** d. Extraction 670. Which intracanal medicament causes protein coagulation: a. Formocresol. *** b. Naocl c. Wad.... D. Hydrogen peroxide 671. GIC compared to composite: a. Increase linear coefficient of Thermal Expansion B. More wear resistant c. Less soluble d. Stiff e. Polymerization shrinkage *** ( ) : . 672. Pt came with pain awaken her from sleep 20 a.m. And could'nt sleep later: reversible pulpitis a. Irreversible pulpitis. *** b. Periodontal pain 673. Pt with severe pain in lower left mandibular molar, examination positive pulp test , percussion test, no radiographic abnormality, rt side have recent fpd upper: a. Chronic apical periodontits b. Actue apical periodontitis ***

a. b. c. d.

a. b. c. d.

c. Apical abcess d. None of the above. 674. Most common cyst a. Apical radicular cyst *** b. Keratocyst c. .... 675. 6 years old child have 74 and 84 extracted best space maintainer is: Lingual arch. Bilateral band and loop. *** Bilateral distal shoe. No need for space maintainer. 676. 6 years old child lost his upper right 1st molar, arrangement: a. Lingual bar. b. Crown and loop. c. Band and loop. *** 677. Band and loop space maintainers is most suitable for the maintenance of space after premature loss of: A single primary molar *** Two primary molars A canine and a lateral incisor All of the above 678. Pedo ( forgot the age ) , lost 75 , sm a. Band and loop *** b. Nance appliance c. Crown and loop . 679. 5 years old patient lost his primary first maxillary molar the best retainer is: Band and loop. *** Crown and loop. Lingual arch. Nance appliance. . 680. (6 years) child with bilateral loss of deciduous molars &the anterior teeth not erupted yet ,the space maintainer for choice is: a-lingual arch B-bilateral band and loop c-bilateral band and loop with distal shoe d-removable partial denture

1. 2. 3. 4.

( ) . . . : . .

Children generally develop their teeth before birth. Eruption starts at about six months, usually with the appearance of the lower incisors, and is complete by about two-and-a-half years. The table below will show an estimate chronology of tooth development: Baby teeth Incisors Canines 1st Molar 2nd Molar Permanent teeth Lower incisors Upper incisors Lower Canines Upper Canines Premolars 1st Molars 2nd Molars 3rd Molars Calcification begins 4 mo (fetal life) 5 mo (fetal life) 6 mo (fetal life) 6 mo (fetal life) Calcification begins 3-4 mo 4-5 mo 4-5 mo 5-6 mo 1.5-2.5 yrs Birth 2.5-3 yrs 7-10 yrs Appearance 6-9 mo 16-18 mo 12-14 mo 20-30 mo Appearance 6-8 yrs 7-9 yrs 9-10 yrs 11-12 yrs 10-12 yrs 6-7 yrs 11-13 yrs 17-21 yrs

http://www.williamsdentalassociates.com/toothdevelopment.htm 681. Lower anterior teeth labial mucosa supplied by: a. Mental nerve. *** b. Inferior dental nerve. C. Buccal nerve. . 682. Buccal branch of trigeminal is: a. Sensory *** b. Motor c. Psychomotor d. Sensory and motor 683. Buccal branch of facial is: a. Sensory b. Motor *** c. Mixed Buccal nerve - Wikipedia, the free encyclopedia

.6 5 . 684. Dentine permeability increases a. Coronal less than root dentine. *** b. Permeability increase toward DEJ. C. Permeability increase toward bcj. . 685. Which material has best biocompatibility Intraorally: a. Cobalt chromium b. Titanium c. Nickle chromium d. Gold .... Palladium 686. Porcelain teeth in complete denture opposing natural teeth are not preferred due to: a. Increase occ load on natural teeth b. Wear of natural teeth *** c. Clicking during mastication 687. Which of following resto more likely to cause wear to opposing: a. Composite b. Gold c. Porcelain *** d. Amalgam 688. In restoring lost tooth, which is least important: a. Esthetic b. Pt demand *** c. Function d. Arch integrity and occlusal stability 689. Enamel tufts are a. Extensions of odontoblasts in the DEJ b. Enamel rods change their direction. C. Enamel rods get crowded *** " Enamel tufts - Wikipedia, the free encyclopedia" 690. In clinical research: a. Blind or double blind b. Needs control

c. Includes intervention d. Offers no benefits to subject at risk 691. One of the main cause of malocclusion: a. Premature loss of primary teeth 692. To disinfect gutta percha: a. Chemical agents b. .. Squamous cell carcinoma : Cawson Essintials of Oral Pathology and Oral .. : Medicine 7th ed : . . . %:3 ( ) . %<8 693. Squamous cell carcinoma is derived from: a. Epithelial tissue. *** b. Connective tissue. 694. Most common site of squamous cell carcinoma: a. Postero-lateral border of tongue. b. Floor of the mouth. c. Buccal mucosa. d. Lip. *** e. Skin. 695. Most common site of oral squamous cell carcinoma: a. Postero-lateral border of tongue. *** b. Floor of the mouth. c. Buccal mucosa. d. Lip. e. Skin. Oral pathology clinical pathologic correlation,3rd edition, Page 71-72 Dental Secrets - page 35 : The posterior lateral and ventral surfaces of the tongue are the most common sites of intraoral cancer. 696. The majority of introral squamous cell carcinomas are histologically: a. Poorly differentiated.

b. Well moderately differentiated. *** c. Spindle cell in type. d. Carcinoma in situation.

Background
Verrucous carcinoma (VC) refers to a clinicopathologic concept implying a locally aggressive, clinically exophytic, low-grade, slow-growing, well-differentiated squamous cell carcinoma with minimal metastatic potential.

Verrucous carcinoma typically involves the oral cavity, larynx, genitalia, skin, and esophagus.

In 1948, Ackerman first described verrucous carcinoma in the oral cavity as a lowcarcinoma.[1] Aird et al first described cutaneous verrucous carcinoma (carcinoma cuniculatum) in 1954, and it was named as such because of its characteristic cryptlike spaces on histologic appearance.[2]

grade tumor that generally is considered a clinicopathologic variant of squamous cell

http://emedicine.medscape.com/article/1101695-overview 697. Squamous cell carcinoma is multifactorial: a. True. *** b. False. 698. Early squamous cell carcinoma of oral cavity present as: a. Vesicle. b. Sessile mass. c. An ulcer. d. Red plaque. *** e. A white cauliflower like lesion Oxford Handbook of Clinical Dentistry - 4th Ed. (2005) page 247 SCC: Clinical appearance Most often seen as a painless ulcer, although may present as a swelling, an area of leukoplakia, erythroleukoplakia or erythroplakia (A reddened patch), or as malignant change of long-standing benign tumours or rarely in cyst linings. Pain is usually a late feature when the lesion becomes superinfected or during eating of spicy foods. Referred otalgia is a common manifestation of pain from oral cancer. The ulcer is described as firm with raised edges, with an indurated, inflamed, granular base and is fixed to surrounding tissues.

SCC . . Burket- SCC A white " SCP Oral medicine page 553 : "cauliflower like lesion Squamous cell papillomas may present as exophytic pedunculated papules with a cauliflower-like appearance.

699. Firm, fixed neck nodes are most to be detected in association with: a. An ameloblastoma b. A basal cell carcinoma c. An odontogenic fibroma d. A squamous cell carcinoma. *** 700. Stage Ib disease of squamous cell carcinoma: A-T1 NO MO b-T3 NO MO c- T2 NO MO. *** d-T4 NO MO http://en.wikipedia.org/wiki/Non-small_cell_lung_carcinoma Grouping TNM staging Occult carcinoma TX N0 M0 Stage 0 Tis N0 M0 Stage IA T1 N0 M0 Stage IB T2 N0 M0 Stage IIA T1 N1 M0 Stage IIB T2 N1 M0 T3 N0 M0 Stage IIIA T1 N2 M0 T2 N2 M0 T3 N1 M0 T3 N2 M0 Stage IIIB Any T N3 M0

T4 Any N M0 Stage IV Any T Any N M1 Grouping Survival rate (percents) One year Two years Three years Four years Five years IA 82% 79% 71% 67% 61% IB 72% 54% 46% 41% 38% IIA 79% 49% 38% 34% 34% IIB 59% 41% 33% 26% 24% IIIA 50% 25% 18% 14% 13% IIIB 34% 13% 7% 6% 5% IV 19% 6% 2% 2% 1% 701. File #40 means: a. 0.40 is the diameter at d1 *** b. 0.40 is from d1 to d16 702. Cause of radicular cyst: a. Non vital tooth *** b. Vital tooth 703. Most difficult of extract: mand. 3rd molar with mesioangular fused roots mand 3rd molar with distoangular angulation with divergent curve roots. *** Dental decks - page 1846 For imapacted mandibular molars, order from the least difficult to most difficult to remove: Mesio angular ---- Horizontal ---- Vertical ---- Distoangular (The opposite in maxilla) Wisdom tooth - Wikipedia, the free encyclopedia Typically distoangular impactions are the easiest to extract in the maxilla and most difficult to extract in the mandible, while mesioangular impactions are the most difficult to extract in the maxilla and easiest to extract in the mandible. 704. Minimum thickness of noble metal crown a-.1 mm b-0.5 mm *** c-1 mm d- 2 mm Dental deck page 441 The necessary thickness of the metal substucture is 0.5 mm the minimal porcelain thickness is 1.0 - 1.5 mm

705. To locate the canal orifice use a-barite probe b-endo spreader c-endo file with curved tip d-round bur Color atlas of endodontics page 14 - Conventional stainless steel files can be precurved and "hooked" into canals. Pathway of the pulp 9ed 1st part page 197-215-227 - examining the pulp chamber floor with a sharp explorer. (maxillary molar) - An oval orifice must be explored with apically curved small instruments. The clinician should place the file tip in the orifice with the tip to buccal when trying to locate the buccal canal. A curved file tip is placed toward the palate to explore for the palatal canal.

- Micro-Openers (Dentsply Maillefer, Tulsa, OK) are excellent instruments for locating canal orifices when a dental dam has not been placed.

- An endodontic explorer is used to search for canal orifices. 706. healing by secondary intention cause ab-there is space between the edges filled by fibrous tissue c-leading to scar formation d- b and c. *** 707. Contraindication for endo treatment EXCEPT: -non strategic tooth -non restorable teeth -vertical fracture teeth -tooth with large periapical lesion ***

708. Arrange the steps a. [ca(oh)2 placing varnish-base amalgam "Art & Science - page 171" Dental Decks - page 709. Soft palate falls abruptly facilitate recording post dam, falls gradually make recording post dam difficult .two statement true .two false *** .first true, second false .first false, second true 5339 " " : 710. caries progress in children more than adult due to .difference in ph .generalized dentin sclerosing by age *** .increasing in organic content of tubular dentin by age Dental Decks - page 2188 711. osteogensis during endodontic surgery aimed to prevent .fibrous in growth *** .growth factor .formation of blood 712. 60 YEARS old patient need to make complete denture with thick labial frenum with wide base. The operation .vestibuloplasty. *** .z-plasty .subperiostum incision .deepmucoperiosteum incision "Peterson's Principles of Oral and Maxillofacial Surgery 2nd Ed 2004 page 173" Z-plasty are effective for narrow frenum attachments. Vestibuloplasty is often indicated for frenum attachments with a wide base. 713. In 6 week intra uterine life the development start. The oral epithelium is stratified squamous epithelium will thickened and give dental lamina a: true *** b: false Http: //www.emro.who.int/publications/emhj/0503/08.htm 714. Retention of amalgam depends on a-amalgam bond b- convergency of walls oclusally ***

c- divergency of walls oclusally d- retentive pins Bhatia's Dentogist: mcqs in Dentistry 715. Energy absorbed by the point of fracture called a- ultimate strength b- elastic limit c- toughness. *** d- brittleness Toughness It is defined as the amount of energy per volume that a material can absorb before rupturing. Toughness - Wikipedia, the free encyclopedia The ability of a metal to deform plastically and to absorb energy in the process before fracture is termed toughness. Toughness 716. Mix in walking non vital bleatching a-H2O2 with phosph........... B-superexol with sod parporate c-superexol with ca hydroxide d- H2o2 with Sodium perborate: *** Oxford Handbook of Clinical Dentistry - 4th Ed. (2005) 717. Local contraindication of extraction abc-pt recent recive radiotheraby d-tooth in the malignant tumar e-both c and d *** 718. Since in composite tooth prep should be conservative so the design a-amalgam in moderate and large cavities b_beveled amalgam margines...... C-conservative restorations. *** : . ) ( 719. preparation of all incipient cavity within enamel Acquired pellicle a- structures layer protect tooth. *** b- aid in remineralization 720. The most affect tooth in nurse bottle feeding a-lower molars b-upper molars

c-max incisor *** d-mand incisor 721. Most impacted tooth is a-mand 8 *** b-max 2 c-upper canine d-premolars 722. Most common tooth which needs surgical extraction a)mandibualr third molar. *** 723. Patient is diagnosed for ceramometal full veneer. You plan to use epoxy Resin >>>>>>missing text>>>>>>>, what's the best impression material to be used : A. Poly ether. B. Poly sulfide. C. Agar agar. D. Irreversible hydrocolloid. 724. the impression material of choice when we want to take impression for epoxy resin pin is: a) Polysulfide. *** B) Polyether. C. Agar agar. D. Irreversible hydrocolloid. 725. Impression material cause bad taste to patient a-poly sulfide. *** b-polyether. c-additional silicon d- alginate Dental terminology 726. what is the most unfavorable imp material by the patient due to bad taste a. agar b. silicone c. polyether d. polysulfide*** . 727. The impression used for preliminary impressions or study casts is: 1. Agar agar.

2. Silicon. 3. Alginate. *** 4. None. 728. What is the least accurate imp material: a.Alginate*** b.Plysufide c.Polyether 729. Irrigant that kills e-foecalis 1-naoh 2-mtad. *** 2-saline 3-chlorohexidine Pathway to the pulp New irrigants are being developed in an attempt to address some of the shortcomings of past and current materials. MTAD is a mixture of a tetracycline isomer (i.e., doxycycline), an acid, and a detergent. In an in vitro study, MTAD was found to be an effective solution for killing E. Faecalis. 730. Irrigant that kills e-foecalis 1-naoh 2-mta. *** 2-saline 3-chlorohexidine 731. During the orthodontist removes orthodontic braces he noticed white hypocalcific lesion around the bracket what to do: 1- microabration and application of pumice then fluoride application. *** 2-composite resin 4-leave and observe 732. daily wear of amalgam: 1-3 microgram /DAY *** 733. Amount of daily wear of amalgam ingested in the body: a. 1 3 gs /day of mercury. b. 10 15 gs /day of mercury. c. 25 gs /day of mercury. 734. weeping canal we use 1-g.p 2-caoh *** 3- Formocresol 4-

Clinical Endodontics textbook TRONSTAD page 224 This situation is often referred to as a weeping canal and is annoying in that the apical part of the canal cannot be dried properly. The right therapy is a discontinuation of the use of the tissue-irritating antiseptics followed by 23 weeks of calcium hydroxide in the root canal. The chemically-induced exudation will then have stopped and the root canal can be dried and obturated. 735. The easiest endo retreatment in: a- Over obturation w GP. b- Under obturation w GP. c- Weeping canals. *** d- Obturated w silver cone. 736. Tug back refers to: 1. Retention of GP inside the canal. *** 2. Fluibilty of GP. http://www.endomail.com/articles/asd28fitting.html
Next we place it into the canal until it stops. We then begin to pull it out of the canal to see if there is tug back. Tug back is that feeling of the GP point sticking in the canal as you try to remove it, causing you to apply a little extra force to remove the point from the canal. When you do achieve tug back, it means that the GP point is binding somewhere in the canal. Figure 3 demonstrates the medium GP being placed into the canal.

737. dental student using thermoplastized g.p. What is the main problem he may face; 1-extrusion of g.p. From the canal *** 2-inability to fill the proper length 3- failure to use maser cone at proper length 4- ledge Oxford Handbook of Clinical Dentistry - 4th Ed. (2005)- page 177 It is difficult to control the apical extent of the root filling and in addition some contraction of the GP occurs on cooling. Useful for irregular canal defects . 738. cracked enamel best Dx by Dye***

739. how can test crack tooth??? Xray elictric test ethyle dye test. *** vitality test 740. Atropine : A- Dries secretion such saliva. *** B- depresses the pulse rate. c -cause central nervous system depression. 741. Drug used to increase saliva is: 1- anticholinergic. 2- cholinergic. *** 3- antidiabetic 4- anticorticosteroid 742. In order to decrease the gastric secretion: a. histamine A antigen equivalent b. histamine B antigen equivalent c. anticholenergic. *** d. adrenal steroids 743. Pt with complete denture come to your clinic, complaint from his dry mouth, the proper medicine is: 1. Anti diabetic medicine. 2. Anticordial. *** 3. Steroid. 744. Drug used to decrease saliva during impression taking is: 1. Cholinergic. 2. AntiCholinergic. *** 3. Antidiabetic. 4. Anticorticosteroid. http://en.wikipedia.org/wiki/Atropine Atropine: It is classified as an anticholinergic drug Injections of atropine are used in the treatment of bradycardia (an extremely low heart rate), asystole and pulseless electrical activity (PEA) in cardiac arrest. This works because the main action of the vagus nerve of the parasympathetic system on the heart is to decrease heart rate. Atropine blocks this action and, therefore, may speed up the heart rate. Atropine's actions on the parasympathetic nervous system inhibits salivary, sweat, and mucus glands. Atropine induces mydriasis by blocking contraction of the circular pupillary sphincter muscle, which is normally stimulated by acetylcholine release

And Dental Decks - page 2012 - 2192 Scopolamine, atropine and benztropine are anticolinergic drugs. They decrease the flow of And salive.Cholinesterase inhibition is associated with a variety of acute symptoms such as nausea, vomiting, blurred vision, stomach cramps, and rapid heart rate. : Cholinergic anticholinergic . 745. probe used to detect furcation: 1-nabers probe. ***
http://www.dentalproductshopper.com/nabers-probe

Nabers Probe
Hu-Friedys 2N Nabers color-coded probes are excellent for assessing furcation areas and measuring bone-loss. The probes are color-coded (3 mm, 6 mm, 9 mm, 12 mm) with Qulix technology that results in smoothly finished, vivid black markings that will not chip, flake, or fade. The probe is available in a Satin Steel handle, a great ergonomic choice for clinicians

746. Least effective to kill aids is: ultra voilet source: dentogist naocl is very effective Also cetrimide and dry heat at 100 degree are inffective in kill aids 747. during endo pt is complaining of pain with percussion what is the cause 1-apical periodontitis 2-secondery apical periodontitis. 3-over instrumentation. *** 4-over medication 748. during endo pt is complaining of pain with percussion what u suspect? 1-apical periodontitis 2-secondery apical periodontitis. 34Oxford Handbook of Clinical Dentistry - 4th Ed. (2005) page 178 Pain following instrumentation: This is usually due to instruments or irrigants, or to debris being forced into the apical tissues. "Pathway of the pulp 9ed 1st part page 217" Postoperative discomfort generally is greater when this area (apical constriction) is violated by instruments or filling materials, and the healing process may be compromised.

: 749. During doing Rct, pt complains from pain during percussion due to: A- Apical infection. B- Impact debris fragment c- Over instrumentation "Pathway of the pulp 9ed 1st part page 406" Shaping to the radiographic apex is likely to produce overinstrumentation past the apical foramen, with possible clinical sequelae of postoperative pain and inoculation of microorganisms into periapical spaces. .Over instrumentation 750. pt on long term antibiotic came with systemic Candida: 1-amphotrecin 2-fluconazol *** 3-nystatin Oxford Handbook of Clinical Dentistry - 4th Ed. (2005) page 240 Fluconazole 50 mg od is the systemic drug of choice. C. Glabrata, C. Tropicalis, and C. Knusel are fluconazole resistant, therefore, candida subtyping should be performed for resistant cases. . 751. systemic Candida what is the best medicine : a-amphotrecin B b- fluconazol*** 752. Candida infection is a frequent cause of: Burning mouth 753. Which one of the following was the most frequently reason for replacement of a molar restoration with larger restoration: a. New caries. b. Recurrent caries. c. Faulty restoration. d. All of the above. *** 754. Lesion at junction between hard and soft palate and surrounded with psudoepithelium and hyperplasia in salivary gland. A . necrotizing sialometaplasia. ***

B . squamus cell carcinoma http://www.sciencedirect.com/science/article/pii/S1741940905001019


Summary
Necrotizing sialometaplasia is an uncommon inflammatory condition that affects salivary glands. A 38-year-old man with bilateral ulcerative painful lesions at the junction of the hard and soft palate was presented. An incisional biopsy was performed. Histopathologically, pseudoepithelomatous hyperplasia, lobular necrosis with through the maintenance of the architecture of salivary glands and squamous metaplasia of residual acinar and ductal elements with a bland appearance were observed. The complete self-healing of the lesions occurred in 3 weeks. Since this entity presents clinical and histopathological findings resembling either mucoepidermoid carcinoma or squamous carcinoma diagnostic failure may culminate in unnecessary mutilating surgery.

755. A child at dentation age is suffering from: a) diarrhea b) sleep disorders c) increased salivation *** : 7< . 756. When restoring asymptomatic healthy tooth with amalgam, the normal physiologic symptom after that is: a. Pain on hot b. Pain on cold *** c. Pain on biting d. Pain on sweet 757. Sharp pain is due to which type of fibers? A. A fibers. *** B.B fibers C. C fibers

A-delta fibers small, myelinated fibers that transmit sharp pain C-fibers small unmyelinated nerve fibers that transmit dull or aching pain. 758. Minimal facial reduction when preparing for veneers: a. 0.3 mm b.0.3-0.5 mm *** c. 1-1.5 mm 3.: 3.8 Dental secrets . "Operative Dentistry A Practical Guide to Recent Innovations - page 83" The facial enamel is usually reduced by 0.30.5mm, but where the underlying tooth is severely discolored, reduction should be 0.7mm. 759. After u did RCT to your pt he came back to the clinic after few days with sever pain on biting, you did x-ray and it revealed that the RCT filling is very good, but u saw radiopaque, thin (film like) spot on the lateral border of the root what is the most probable diagnosis? A- Accessory canal b) vertical root canal fracture. *** 760. Patient suffering from a cracked enamel, his chief complain is pain on : A) Hot stimuli B) Cold stimuli C) A & B. *** D) Electric test. 761. patient came complaining of severe pain on biting, related to a certain tooth. upon examination no pulpal or periodontal findings, and pulpal vitality is positive, your Dx: 1) cracked tooth syndrome*** 762. Method of Detection of Cracked teeth : A) Horizontal percussion B) Vertical percussion C) Electric pulp test D) Transillumination / visible light test.. *** 763. Cracked tooth syndrome is best diagnosed by? A. Radiograph b. Subjective symptoms and horizontal percussion c. Palpation and vertical percussion d. Pulp testing Pickard's Manual of Operative Dentistry Eighth edition OXFORD - PAGE 213 The diagnosis of cusp fracture is easy when the cusp has fallen off. Before this actually happens, however, the patient may experience pain but often finds it

remarkably difficult to locate this to a particular tooth. The patient will frequently complain of sensitivity to hot and cold and discomfort on biting. Even on clinical examination it is often difficult to pinpoint which tooth is causing the pain, but a fiber-optic light or disclosing solution may assist the diagnosis by making the crack easier to see. Lateral pressure on the suspect cusp may also help by producing a sensitivity that mirrors the patients symptoms. Often the pain occurs when the pressure is released. A crack usually does not show up on an x-ray, a physical examination of the tooth will have to be performed. A sharp instrument will be used to allow us to explore the tooth for cracks. We will also place pressure on the tooth to see if we can expand the crack until it is seen. You may have X-rays taken but X-rays often do not reveal the crack. Your dentist may use a special tool to test the tooth. There are different kinds of tools. One looks like a toothbrush without bristles. It fits over one part of the tooth at a time as you bite down. If you feel pain, the part of the tooth being tested most likely has a crack in it. Diagnostic tests of cracked tooth visual examination of cracks: aided by staining with dye such as methylene blue. Tactile examination crutch the tooth surface with a sharp explorer widening a gap of the crack may elicit extremely painful response. Tooth sloth : bite tests each cusp tip must be tested individually pain on release often indicates crack tooth. Transillumination: fiberoptic light source held perpendicular to the suspected crack that mean the prober exam for crack in the tooth(b) subjective symptoms and horizontal percussion the book is NBDE II 764. The best method for tooth brush is Bass method because: a. It enter to interproximal area b. Can be used by patient with gingival recession and it rotainary advice to all types of patients. 1. The both sentences are correct. *** 2. The first sentence is correct and the second is wrong. http://www.oralcareshop.com/bass-method.htm
The Bass Method or Sulcular Vibration Brushing or 45 Degree Angle the Bass Tooth Brushing Technique is a very effective method for germs or plaque removal next to and directly below the gum or gingival margin. The area at the gum-

tooth margin is the most significant in the prevention of tooth decay and gum disease.

765. Bass brushing has the advantage of the bristles enters in the cervical area , and it is recommended for all patients: a)both statements are true. *** b)both statements are false c)first is true ,second is wrong d)first is wrong , second is true Caranza periodontology page 658 Bass method advantages: - It concentrates the cleaning action on the cervical and interproximal portions of the teeth. - The Bass technique is efficient and can be recommended for any patient with or without periodontal involvement. 766. Patient comes to you with edematous gingiva, inflamed, loss of gingival contour and recession, what's the best tooth brushing technique? A. Modified bass b. Modified stillman. *** c. Charter d. Scrub Dentogist MCQs in Dentistry The brushing technique which is recommended after periodontal surgery is Charte.r The brushing technique which is recommended for areas with progression gingival recession is modified stilman. Caranza clinical periodontology page 659

.
Modified bass . 468 Charter .
It's a good idea to use the tooth-brushing method recommended by your dentist. The modified Stillman method is often used in patients with progressing gingival recession. This means that the patient's gums are receding toward the base of the tooth, leading to root exposure. This method is used to avoid damage to the delicate tissue, which would further exacerbate the condition http://www.ehow.com/way_5399734_modified-stillman-tooth-brushing-technique.html

767. The best method to protect teeth that underwent bicuspidization procedure from fracture? A. Full crown. *** b. Splint with composite c. Orthodontic splint Dental decks page 273 768. Pt have unilateral fracture of left the condyle, the mandible will a)deviate to the left side. *** b)deviate to the right side. c)no deviate. dental decks 2004 the mandible will always deviate to the side of injury. A patient who sustaind a subcondyler fracture on the left side would be unable to deviete the mandible to the right. 769. Pt came after 24 month of tooth replantation which had ankylosis with no root resorption. It most likely to develop root resorption in: 1/ reduce greatly *** 2/ increase 3/ after 2 years 4/ after 4 years 469 % 68-9 .% 49-46 % ;3 770. tracing of GP used for : 1/source of periapical pathosis *** 2/acute periapical periodontitis 3/ periodental abscess 4/ none ) ( 771. isolated pocket in: -vertical root fracture -palato gingival groove -endo origine lesion -all. *** http://www.mmcpub.com/pdf/1998ppa/19...av10n3p369.pdf And Vertical root fractures: Clinical and radiographic diagnosis -- COHEN et al. 134 (4): 434 -- The Journal of the American Dental Association 772. After bleaching a tooth, we want to restore the tooth with composite resin, we dont want to compromise the bonding, we wait for:

a)24 hours b)a week *** c) choose a different material " Clinical Aspects of Dental Materials Theory, Practice, and Cases (3rd Ed 2009)" Esthetic restoration of teeth should be delayed for 2 weeks after the completion of tooth whitening. . 773. Three year old pt, has anodontia (no teeth at all), what would you do: a) full denture *** b) implant c) space maitainer d)no intervention "PAEDIATRIC DENTISTRY - 3rd Ed. (2005)" page 294 In cases of anodontia, full dentures are required. These can be provided, albeit with likely limited success, from about 3 years of age, with the possibility of implant support for prostheses provided in adulthood. B)-autoimmune factors++ (one of the signs of Autoimm diss) dental deck - page 1334 774. Treacher collins syndrome characterized by -PROGNTHESIA OF MANDIBLE. -NO EAR LOSS -UPWARD SLUTING OF EYE -MALAR BONE NOT WELL FORMED OR ABSENCE. *** : - underdeveloped cheek bone . - - - : Treacher Collins syndrome - Wikipedia, the free encyclopedia Treacher-Collins Syndrome-A Challenge For Aaesthesiologists Goel L, Bennur SK, Jambhale S Indian J Anaesth 775. Patient present with deffieciency at the malar bone+open bite+normal mental abilities; 1-treacher cholins. *** 2-cleidocrenial dyspasia 3-eagle syndrome

776. when removing lower second molar: a- occlusal plane perpendicular To the floor b- buccolingual direction to dilate socket. *** c- mesial then lingual 777. The best material for taking impression for full veneer crowns: a) poly vinyl sialoxane (additional Silicone)*** 778. Stock trays compared to Custom trays for a removable partial denture impression A. Custom trays less effective than stock trays B. Custom trays can record an alginate impression as well as elastomeric impression C. Custom trays provide even thickness of impression material. *** D. All of the above 779. Which type of burs is the least in heat generation: a) diamond b) carbide *** c) titanium d) Steel bur " -4 -5 ") . (: more heat generated in diamond burs dental secrets: page 200 780. Pt. Wears complete denture for 10 years & now he has cancer in the floor of the mouth. What is the firs question that the dentist should ask: a- does your denture is ill fitted b- smoking. ***+ (80% of the cancer of the floor of the mouth is caused by smoking) c- Alcohol d- does your denture impinge the o.mucosa. (traumatic cause) Screening Oral Cancer - Prepared by Bruce F. Barker, D.D.S. and Gerry J. Barker, R.D.H., M.A. University of Missouri-Kansas City School of Dentistry

Ulceration on floor of mouth in edentulous patient, initially misinterpreted as denture irritation. 781. Secondary dentine occur due to a- occlusal trauma b- recurrent caries c- attrition dentine d- all of the above*** 782. All of these are ways to give L.A with less pain EXCEPT: a- give it slowly b- stretch the muscle. *** c- Topical anesthesia d- the needle size over than 25 gauge. . 783. How much subgingivally do you go with the band in class II restorations: A) 0.5 1 mm. *** B) 1 2 mm C) 2 3 mm 784. The matrix band should be above the adjacent tooth occlusal surface by a. 1-2mm. *** b. 2-3mm. c. 2.5-3.5mm. d. below to it. Textbook of operative dentistry- with MCQs page 139 Matrix band should extend 2 mm above the marginal ridge height and 1 mm below gingival margin of the cavity. The matrix ban should not extend more than 2 mm beyond the occlusogingival height of the crown of the tooth. this facitilates vision and speed up working.

Thickness of band is 0.05 mm = 0.002 inch Concepts in nonsurgical periodontal therapy page 100 For adequate closure of the margin, a minimum 0.5 mm of matrix band beyond the margin is necessary. 785. A female patient came to your clinic with dry lips and mouth and bilateral submandibular oedema and ocular dryness. Diagnosis is: a)Polymorphecadenoma b) sialotitis*** or: Sjgren's syndrome if present 786. The compression / relaxation cycle of external cardiac compression should be repeated a- 2 times / second b- 60 times / minute c- 76 times / second d- 100 times / minute. *** Oxford Handbook of Clinical Dentistry - 4th Ed. (2005) - page 304 Circulation Feel for a carotid pulse. If it is present, provide 10 breaths per minute, checking the pulse for 10 sec every 10 breaths. If no pulse commence chest compression, at the middle of the lower half of the sternum, depressing 45 cm 100 times per minute. And Dental Decks - page 1790 787. One of the primary considerations in the treatment of fractures of the jaw is a- to obtain and maintain proper occlusion*** b- test teeth mobility c- vitality d- embedded foreign bodies Dental Decks - page 1756 788. A child patient undergone pulpotomy in your clinic in1st primary molar. Next day the patient returned with ulcer on the right side of the lip your diagnosis is: a) Apthosis b) Zonal herpes c) traumatic ulcer*** 789. Bitewing exam is used to diagnose EXCEPT: 1. Proximal caries. 2. Secondary caries. 3. Gingival status. 4. Periapical abscess***

. 790. Which of the following types of base materials can be placed in contact with polymethyl methaacrylate & not inhibit the polymerization of the resin a) zoe b) GI cement c) Zn phosphat cement d) varnish e) b&c*** 791. We can use under the composite restoration: 1. Varnish. 2. Zinc oxide and eugenol. 3. Ca (OH)2. 4. Zinc phosphate cement. 1+2. 2+3. 3+4*** 2+4. . 792. A patient complaining from a severe oedema in the lower jaw that increases in size upon eating, Diagnosis is: a) salivary gland. *** (submandibular sal. Gl.) 793. a patient that wasnt anaesthetized well in his 1st visit, next day he returns with a limited mouth opening (trismus). He must be anaesthized, whats the technique to be used: a) Williams technique b) Berchers technique.*** . - Trismus ( ) Bercher . ) (

794. a patient that wasnt anaesthetized well in his 1st visit, next day he returns with a limited mouth opening (trismus). He must be anaesthized, whats the technique to be used -a Williams technique b- gow gates technique

-c vazirani-akinosi techniqu : Vazirani-Akinosi technique - a closed-mouth injection technique, the syringe ** is "advanced parallel to the maxillary occlusal plane at the level of the maxillary mucogingival junction http://webcache.googleusercontent.com/search?q=cache:TNfGYamBvFkJ:en. wikipedia.org/wiki/Inferior_alveolar_nerve_anaesthesia+vazirani+akinosi+tec hniques&cd=5&hl=ar&ct=clnk&gl=sa&client=firefox-a

VaziraniAkinosi Nerve Block


The Vazirani-Akinosi closed mouth mandibular block is a useful technique for patients with limited opening due trismus or ankylosis of the temporomandibular joint. A 25 gauge long needle is recommended for this technique. The right handed operator should be in the eight oclock position whereas the left handed operator should be in the four oclock position. The gingival margin above the maxillary 2nd and 3rd molars and the pterygomandibularraphae serve as landmarks for this technique. The patient should close gently on the posterior teeth. The needle is held parallel to the occlusal plane at the level of the gingival margin of the maxillary 2nd and 3rd molars. The bevel is directed away from the bone facing the midline. The needle is advanced through the mucous membrane and buccinator muscle to enter the pterygomandibular space. The needle is inserted to approximately one half to three quarters of its length. At this point the needle will be in the midsection of the ptyerygomandibular space. . Nerves anesthetized are the lingual and long buccal nerves in addition to the inferior alveolar nerve.

795. At which of the following locations on a mandibular molar do you complete the excavation of caries first: a- axial walls . B- pulpal floor over the mesial pulp horns c- peripheral caries. *** d- all of the above are correct. 634 . 796. CMCP contains phenol in concentration a-0.5 %. b- 35 %. *** c-65 %. d- 5 %. 797. Dentist provided bleaching which also knowr as (home bleaching) contain a- 35-50% hydrogen peroxide b- 5-22% carbamide peroxide ***

Oxford Handbook of Clinical Dentistry - 4th Ed. (2005) - page 316 a solution of 10% carbamide peroxide in a soft splint has been advocated for home bleaching 798. What is the disadvantages of Mcspadden technique in obturation requires much practice to perfect. *** 799. What are the disadvantages of mcspadden technique in obturation: a) Increase time. B) Increase steps. C) Difficult in curved canals. *** D) All the above. Oxford Handbook of Clinical Dentistry - 4th Ed. (2005) - page 177 And: Disadv's of mcspadden Tech: 1. Use of speed higher than recommended>> poor seal. 2. Extrusion of the filling material. 3. Fracture of thermocompactor. 4. Gouging of the canal wall. 5. Inability to use the technique in curved canals. 6. Heat generation may lead to PDL damage, resorption and ankylosis.. 7. Voids in final filling And: If the file is turning in reverse, it can screw itself into the canal and periapical tissues Endodontic Obturation 800. thermo mech. Tech of obturation is: A) thermafil B) obtura C) ultrafil D) mcspadden. *** (source : Endodontic obturation materials) 801. Fractured tooth to alveolar crest, what's the best way to produce ferrule effect? A) restore with amalgam core sub-gingivaly. *** b) crown lengthening c) extrusion with orthodontics :

1- Dental secrets 269 " If the fracture is subgingival, remove the coronal segment and perform appropriate pulp therapy, then reposition the remaining tooth structure coronally either orthodontically or surgically" And Oxford handbook. 2- http: //www.dental-update.co.uk/articles/35/3504222.pdf In the absence of a ferrule, Aykent et al19 found that in vitro use of a dentine bonding agent with an amalgam core and a direct stainless steel post provided a significant increase in fracture resistance in extracted premolars. Whilst dentine bonding of the amalgam core did not offer any significant improvement when a 1 mm ferrule was present, this study suggests that there may be a role for dentine bonding of amalgam cores when a ferrule cannot be achieved. Both crown lengthening and orthodontic extrusion may allow for an increased ferrule, but they add additional cost, discomfort and length of treatment times for the patient. Crown lengthening increases the crown to root ratio. Whilst Ichim et al16 used finite element analysis to predict that crown lengthening did not alter the levels or pattern of stress within the palatal dentine, Gegauff20 concluded that crown lengthening could be problematic. Gegauff20 investigated whether crown lengthening to achieve a ferrule would affect the static load failure. By placing the finish line further apically, Gegauff postulated that the tooth may be weakened as a result of the resultant decrease in cross-sectional area of the preparation and the increased crown to root ratio. Orthodontic extrusion may avoid this problem as it results in a smaller change in the crown to root ratio. ) ( 802. 65 years old black man wants to have very white teeth in his new denture what should the dentist do: a- Put the white teeth b- Show the patient the suitable color first then show him the white one. c- Convince him by showing him other patients photos. d- Tell him firmly that his teeth color are good. Or - Do not show white teeth Complete Denture 17th Ed page 73-74 Often a short informative talk using some of the tooth manufacturer's (or preferably your own) "before and after" photos can be very effective in motivating patients to accept a more natural tooth selection. SHADE SELECTION: The shade is selected with the Portrait shade guide. There are 24 shades; enough to fulfill the color requirement of any denture patient. The large

selection seems confusing but you will soon note that certain shades will be used the most. Shades A,B,C, & D (16) are the most useful and are called characterized shades as they are a subtle mix of many shades. The last 8 are the Bioform shades, B59-882, and are not characterized and do not look as natural. They are included as Bioform teeth and shades have been sold for many years and it's often necessary to match them. 1. For youthful patients, use lighter shades with a bluish incisal. 2. For older patients, use a darker shade with mostly body color. 3. If the patient has dark hair, brown eyes, and dark skin, darker shades with more yellow and brown will look more natural. 'This rule does not apply for Black patients as they often have very white teeth. 4. If the patient has blue eyes and fair skin, use lighter shades with more gray. 5. Consider the patient's desires: Many patients have very definite ideas on tooth color and will often not accept the above guidelines. Don't let the patient get a hold of the shade guide; many will select the pearly white A1 !! Select a shade that you think is suitable and let them make the final choice. 6. Consider the patient's old denture. Do they like the color of the teeth? If they think the color of the old dentures teeth were satisfactory, match them closely. Keep in mind that teeth discolor and other manufacture's shades may be somewhat different. : " " . . . 803. During clinical examination the patient had pain when the exposed root dentine is touched due to: a- Reversiple pulpitis b- Dentine hypersensitivity c- Irreversible pulpitis 804. Pt presented to u having root recession he has pain when putting probe gently on the root

what is the diagnosis: a. Dentin hypersensitivity b. Reversible pulpitis. c. Irreversible pulpitis. d. Apical Periodontitis. 805. The patient have dull pain and swelling and the PA shows apical radiolucency your diagnosis will be: a- Acute periodontal abscess b- Chronic periodontal abscess with swelling. *** 806. All these shows honey combed bone radiographically EXCEPT: a- Ameloblastoma b- Odontogenic myxoma cyst c- Odontogenic keratocyst d- Adenomenated tumor. *** Source : http://www.head-face-med.com/content/1/1/3 807. Flouride amount in water should be: a- 0.2-0.5 mg/liter b- 1-5 mg/liter c- 1-2 mg/liter. *** d- 0.1- 0.2 mg/liter 808. For children considered to be at high risk of caries and who live in areas with water supplies containing less than 0.3 ppm: a. 0.25 mg F per day age 6 months to 3 yrs b. 0.5 mg f per day From 3-6 yrs c. 1 mg per day more than 6 yrs d. All of above*** 809. 3 year old pt, water fluoridation 0.2ppm what is the preventive treatment a. 0.25 mg fluoride tablet*** b. 1 mg fluoride tablet c. Fluoridated mouth wash d. Sealant oxford handbook of clinical dentistry 4th 2005 oxford 810. the powered toothbrush invented in: a- 1929 b- 1939. *** c- 1959 d- 1969 http://www.6moo7.com/vb/showthread.php?t=16782

811. The most superior way to test the vitality of the tooth with: a- Ice pack. B- Chloro ethyl. C- Endo special ice. *** D- Cold water spray. http://www.realworldendo.com/eng/lib_diagnosis_procedures.html Once we have determined that a particular tooth is indeed percussive, the next step is to determine the pulp status of that tooth (vital or non-vital). Without question, the method most commonly employed by Endodontists is the thermal test. When doing a cold test, Endo Ice, (a refrigerant) can be sprayed on a cotton pellet and then placed on a dry tooth. This test, along with the percussion test are the two most important tests in diagnosis. If the patient feels nothing and there is no response from the tooth, we must assume this tooth to be necrotic (non vital). <7 . 55 812. The way to remove mucocele is a- Radiation b- Excision *** c- Chemotherapy d- Caterization Mucocele Infrequently, a mucocele goes away without treatment. But if some mucoceles remain untreated, they can scar over. Your dentist should examine any swelling in your mouth. A mucocele usually is removed by surgery. The dentist may use a scalpel or a laser to remove the mucocele. Afterward, the tissue will be sent to a laboratory for evaluation. There is a chance that after the mucocele is removed another one may develop. Some doctors use corticosteroid injections before trying surgery. These sometimes bring down the swelling. If these work, you may not need surgery 813. 8 years old come with fractured max incisor tooth with incipient exposed pulp after 30 min of the trauma, whats the suitable rx: a- Pulpatomy b- Direct pulp capping

c- Pulpectomy d- Apexification
Children generally develop their teeth before birth. Eruption starts at about six months, usually with the appearance of the lower incisors, and is complete by about two-and-a-half years. The table below will show an estimate chronology of tooth development: Baby teeth Incisors Canines 1st Molar 2nd Molar Permanent teeth Lower incisors Upper incisors Lower Canines Upper Canines Premolars 1st Molars 2nd Molars 3rd Molars Calcification begins 4 mo (fetal life) 5 mo (fetal life) 6 mo (fetal life) 6 mo (fetal life) Calcification begins 3-4 mo 4-5 mo 4-5 mo 5-6 mo 1.5-2.5 yrs Birth 2.5-3 yrs 7-10 yrs Appearance 6-9 mo 16-18 mo 12-14 mo 20-30 mo Appearance 6-8 yrs 7-9 yrs 9-10 yrs 11-12 yrs 10-12 yrs 6-7 yrs 11-13 yrs 17-21 yrs

http://www.williamsdentalassociates.com/toothdevelopment.htm 814. pt came to dental clinic having a heamological problem after lab test they found that factor VIII is less 10% whats the diagnosis: a- Heamophilia A. *** b- Hemophilia b (defect factor 9 : hemophilia B) 815. all these are contraindicated to RCT EXCEPT: a- Non restorable tooth b- Vertical root fracture c- Tooth with insufficient tooth support d- Pt who has diabetes or hypertension 816. sharpening the curette and sickle, the cutting edge should be at angle: a- 50-60 b- 70-80. *** c- 80-90 d- 60-70 817. avulsed tooth is washed with tap water , it should be replaced again: a- Immediately. *** b- After 2 hours. c- 24 hrs. 818. 10 yrs old child, who is an able to differentiate the colors, and cant tell his name, or address. He is acting like:

a. 3 years old. *** b. 4 years old. c. 10 years old. 819. Patient with complete denture pronouncing F as a V anterior teeth are upward from lip line. *** dental decks II - page 396 Placement of maxillary anterior teeth in complete dentures too far from superiorly and anteriorly might result in difficulty in pronouncing F and V sounds. 820. Patient with complete denture pronouncing F as a V maxillary anterior teeth had placed too far from superiorly and anteriorly 821. pt have a complete denture came to the clinic ,tell you no complaint in the talking ,or in the chewing ,but when you exam him ,you see the upper lip like too long ,deficient in the margins of the lip, reason is? A)deficiency in the vertical dimensional b)anterior upper teeth are short. *** c)deficient in vit B . 822. child has a habit of finger sucking and starts to show orodental changes, the child needs: a- Early appliance b- Psychological therapy c-rewarding therapy d- punishment 823. Knife ridge should be tx with: 1/relining soft material 2/ maximum coverage of flange. *** 3/ wide occ. Table 4/all Dental Decks - page 414 If you are fibrating mandibular complete denture for with a patient with knifeedge ridge, you need maximal extension of the denture to help distribute the forces of occlusion over a large area. 824. Child 3 years old came to clinic after falling on his chin, you found that the primary incisor entered the follicle of the permanent incisor what you will do: A) Surgical removal of the follicle B) Leave it C) Surgicall removal of the primary incisor. ***

Dental Decks - page 1626 If the intruded incisor is contanting the permenant tooth bud, the primary tooth should be extracted. 825. Tongue develope from: 1/mandibular arch & tuberculum impar. *** 2/1st branchial arch Anatomy of the Human Body - Henry Gray page 27 - The mandibular arch lies between the first branchial groove and the stomodeum; from it are developed the lower lip, the mandible, the muscles of mastication, and the anterior part of the tongue. - The ventral ends of the second and third arches unite with those of the opposite side, and form a transverse band, from which the body of the hyoid bone and the posterior part of the tongue are developed. Anatomy of the Human Body - Henry Gray page 693 During the third week there appears, immediately behind the ventral ends of the two halves of the mandibular arch, a rounded swelling named the tuberculum impar, which was described by His as undergoing enlargement to form the buccal part of the tongue. More recent researches, however, show that this part of the tongue is mainly, if not entirely, developed from a pair of lateral swellings which rise from the inner surface of the mandibular arch and meet in the middle line. 826. Perforation during endo space preparation what is the most surface of distal root of lower molar will have tendency of perforation: 1/ M SURFACE. *** 2/ Distal surface. 3/ Buccal surface. 4/ Lingual surface. Dental decks 144 827. Crown and root perforation: 1/ respond to MTA 2/ use matrix with hydroxyapatite and seal with G I 3/1&2. *** 4/root canal filling 828. acceptable theory for dental pain: 1/hydrodynamic. *** 2/fluid movement 3/ direct transduction Dental Decks - page 2234

The most accepted theory to explain the unusual sensitivity and response of exposed root surfaces to to various stimuli is the hydrodynamic theory. And "Art and science of operative dentistry 2000" PAGE 257 Most authorities agree that the hydrodynamic theory "best explains dentin hypersensitivity. The equivalency of various hydrodynamic stimuli has been evaluated from measurements of the fluid movement induced in vitro and relating this to the hydraulic conductance of the same dentin specimen."' 829. While u were preparing a canal u did a ledge, then u used EDTA with the file, this may lead to a. perforation of the strip 830. Removing of dentine in dangerous zone to cementum is: 1/ perforation. *** 2/ledge. 3/stripping. 4/zipping. 831. Follow up of RCT after 3 years , RC failed best treatment is to: a) Extraction of the tooth b) Redo the RCT . *** c) Apicectomy 832. Acute abscess is: a) Cavity lined by epithelium. B) Cavity containing blood cells. C) Cavity containing pus cells. *** d) Cavity containing fluid. MASTER DENTISTRY- Oral and Maxillofacial, Surgery, Radilolgy, Pathology and Oral Medicine An abscess is a pathological cavity filled with pus and lined by a pyogenic membrane. epithelium ( ) .48 Pathway of the pulp 9ed 1st 833. The most close tooth to the maxillary sinus: a)maxillary 1st molar. *** . Endodontics

834. mechanochemical prep'n during RCT main aim: 1) widening of the apex 2) master cone reaches the radiographic apex 3) proper debridement of the apical part of the canal****** 835. Master cone doesn't reach the apex 1)ledge 2)residual remenants (debris) 3)...... 4) 1&2 ****** 836. child patient with oblitration in the centeral permenant incisor. What will you do: RCT pulpotomy pulpectomy careful monitoring****** 837. At which temperature that gutta percha reach the alpha temp: a- 42-48 c ****** b- 50-60 c- 70-80 d- 100c 838. After completion of orthodontic treatment he came complaining of pain in 11 tooth radiograph show absorption in the middle third of the root of 11 wt is the proper management: a. Apply caoh at the site of resorption. ****** b. Do RCT in a single visit c. Extract the tooth & reimplant it d. Extract the tooth & do implantation Dental decks part 2 page236 Bowl -shaped area of resorpation in volving cementument dentin characterize external inflamatory root resorption, this type of external resorption is rapidly progressive and will continue if treatement is not instituted. the process can be arrested by immeditae root canal treatment with calcium hydroxide paste. remembre the etiology of external resorption : excessive orthodontie forces, periradicular inflammation, dental trauma, impacted teeth. ) ( 839. Patient that has a central incisor with severe resorption and who's going through an ortho treatment that is going to make him extract the premolars, which of the following won't be present in the treatment plan a. rpd b.implant

c. Maryland bridge d.auto implant of the premolars. ****** ) ( 840. The aim of treatment maintenance is: A) Prevent secondary infection.. *** B) Check tissue response. 841. The aim of maintenin therpy is a- Prevent recurrent dieases B- Check tissue response. 842. Provisional restoration for metal ceramic abutment is a) aluminum sheet b) stainless steel crown c) zno d) tooth colored polycarbonate crown*** 843. Dr.black (GV black) periodontal instrument classification: study what the number represent in the instrument formula. *** 844. For g.v black classification study what the number refers to angulation? a. Number 1 b. Number 2 c. Number 3 *** d. Number 4 "For g.v black classification study what the number represent in the instrument formula one for width one fo length one for angulation 1st: Width of blade -2nd: Length of the blade in millimeter-3rd: Angle of blade -4th: when cutting edge at ab angle other then right" 845. An adult had an accident, maxillary central incisors intruded, lip is painful with superficial wound what is the traumas classification: a) luxation. *** b) subluxation c) laceration d) abrasion e) contusion Handbook of dental trauma 2001- page 73 Intrusive luxations, or intrusions, result from an axial force applied to the incisal edge of the tooth that results in the tooth being driven into the socket 846. Schick test an intradermal test for determination of susceptibility to

a. Diphtheria hyperSensitivity b. Tuberculosis hypersensitivity c. ??? lepsron : The Schick test, invented between 1910 and 1911[1] is a test used to determine whether or not a person is susceptible to diphtheria.[2] It was named after its inventor, Bla Schick (18771967), a Hungarian-born American pediatrician. The test is a simple procedure. A small amount (0.1 ml) of diluted (1/50 MLD) diphtheria toxin is injected intradermally into the arm of the person. If a person does not have enough antibodies to fight it off, the skin around the injection will become red and swollen, indicating a positive result http://en.wikipedia.org/wiki/Schick_test Schick test - definition of Schick test in the Medical dictionary - by the Free Online Medical Dictionary, Thesaurus and Encyclopedia. 847. In a curved root u bent a file by. A. Put gauze on the file & bend it by hand *** b. Bend the file by pliers c by bare finger d. By twist . 848. Father for child 12 year pt <asked you about ,the age for the amalgam restoration of his child ,you tell him: a)2 years b)9 years c)2 decades. *** d)all life Art and science of operative dentistry 2000 page 766 Complex amalgam restoration with pins: Smales reported that 72% of amalgam restorations survived for 15 years, including those with cusp coverage. 53 :49;

849. Cleft lip is resulted from incomplete union of: 1. Tow maxillary arches. 2. Maxillary arches and nasal arch.*** 850. Arrange the steps of cleft palate management: 1. Measures to adjust speech. 2. Establish way for nursing and feeding. 3. Cosmetic closure. 4. Prevent collapse of two halves. 2 4 1 3. 851. 8 years old pt. Had trauma to 8 presented after 30 minute of injury He had crown fracture with incipient pulp exposure what u do: 1. Direct pulp capping. *** 2. Pulpotomy. 3. Pulpctomy. 4. Observe. 852. Time of PT, PTT: a)11-15 seconds , 25-40 seconds. *** PTT=30-40sec ---- PT=12-14 sec bleeding time within < 8 min 853. Young pt came without any complain. During routine X ray appear between the two lower molar lesion diameter about 2mm & extend laterally with irregular Shape. Whats the type of cyst a) dentigerous cyst

b) apical cyst c) radicular cyst 854. When extracting all max teeth the correct order is: a) 87654321 b) 87542163. *** c) 12345678 . : Dentogist MCQs in Dentistry Archer suggest that the first maxillary molar and canine are key pillars of maxilla and most firm teeth of the arch, once their adjacent teeth are removed they can be easily luxated and extracted rathar than when these are tried to be removed first. 855. For a patient that is on a corticosteroid therapy, upon oral surgery, the patient is given: A) 100 - 200 mg hydrocortisone. *** B) 400 - 600 mg prednisolone 856. Patient under corticosteroid therapy , he will undergo surgical extraction of third molar . what will you give to avoid adrenal crisis a-Dixamethasone ( 4 mg / I.V.) b- Methyl prednisolone ( 40 mg / I.V). c- Hydro cortisone sodium sulfide ( 40 50 mg.) d- Hydro cortisone sodium succinate ( 100 200 mg) 857. Patient with lupus erythematous and under cortisone, he needs to surgical extraction of a tooth. What should the surgeon instruct the patient: a) Take half of the cortisone dose at the day of operation. B) Double the cortisone dose at the day of operation.*** c) Take half of the cortisone dose day before and at the day of operation and day after. D) Double the cortisone dose day before and at the day of operation and day after. Instruct patient to double dose of steroids the morning of surgery up to 200mg. If taking greater than 100mg, then give only an additional 100mg. If on alternate day steroids, do surgery on day steroids are taken If patient has had 20 mg of steroid for more than two weeks in the past 2 weeks, but is not currently taking steroids, then give 40mg hydrocortisone prior to surgery And Dental secrets page 49

For multiple extractions or extensive mucogingival surgery, the dose of corticosteroids should be doubled on the day of surgery. If the patient is treated in the operating room under general anesthesia, stress level doses of cortisone, 100 mg intravenously or intramuscularly, should be given preoperatively. And Dental Decks - page 1834 . 858. The right corticosteroid daily dose for pemphigus vulgaris is: a- 1-2 g/kg/daily b- 1-2 mg c- 10 mg d- 50- 100 mg hydrocortisone. *** Tyldesley's Oral Medicine, 5th Edition page 132 Very high dosages are used initially to suppress bulla formation (of the order of 1 mg/kg prednisolone daily), but this may often be slowly reduced to a maintenance dose of 15 mg daily or thereabouts 859. The right corticosteroid daily dose for pemphigus vulgaris is: a- 1-2 g/kg/daily b- 1-2 mg/kg/daily*** c- 10 mg/kg/daily d- 50- 100 mg/kg/daily hydrocortisone 860. The following are indication of outpatient general anesthesia EXCEPT a) ASA categories 1 & 2 b) the very young child. c) cost increase. *** d) Patient admitted and discharge the same day 861. The primary source of retention of porcelain veneer 1_mechanical retention from under cut 2_mechanical retention from secondary retentive features 3_chemical bond by saline coupling agent 4_micromechanical bond from itching of enamel and porcelain 862. A removable partial denture patient, Class II Kennedy classification. The last tooth on the left side is the 2nd premolar which has a distal caries. Whats the type of the clasp you will use for this premolar: a) gingivally approaching clasp. *** b) ring clasp

863. Patient come to your clinic complaining that the denture become tight, during examination you notice nothing, but when the patient stand you notice that his legs bowing (curved). What you suspect: a) Pagets disease. *** b) .x c) .x 864. A 55 year old patient with multi-extraction teeth, after extraction what will you do first: a) Suturing. B) Primary closure should be obtained if there is no luntant tissue. C) Alveoplasty should be done in all cases. *** 865. Child with traumatized lip, no tooth mobility, what will you do first: a) Radiograph to check if there is foreign body. *** b) Refer to the physician for sensitivity test. C) .? 866. 2nd maxillary premolar contact area: a) Middle of the middle third with buccal embrasure wider than lingual embrasure. B) Middle of the middle third with lingual embrasure wider than buccal embrasure.*** c) Cervical to the incisal third .x d) .x 867. Patient come to the clinic with ill-fitting denture, during examination you notice white small elevation on the crest of the lower ridge, what will you tell the patient: a) This lesion needs no concern and he should not worry. B) The patient should not wear the denture for 2 weeks then follow up. *** c) x 868. How do you know if there are 2 canals in the same root: a) Radiographically with 2 files inside the root. *** b) The orifices are close to each other. C) x 869. Hunter Schreger bands are white and dark lines that appear in: a) Enamel when view in horizontal ground. B) Enamel when view in longitudinal ground. *** c) Dentin when view in horizontal ground. D) Dentin when view in longitudinal ground.

Hunter-Schreger band formation as it exists in enamel structure. When examined by reflected light, these bands appear as alternating light and dark areas in the enamel portion of a longitudinal ground tooth section 870. Patient complains from pain in TMJ. During examination you noticed that during opening of the mouth mandible is deviate the right side with left extruded. Diagnosis is: a) Condylar displacement with reduction. B) Condylar displacement without reduction. *** C) .x d) x 871. Pt. Presented to u complain of click during open and close. Thers is no facial asymmetry EXCEPT when opening What is the diagnosis: 1-internal derangement with reduction. *** 2-internal derangement without reduction 3-reumatoid arthritis 4-,,,,,,, 1 Disc dislocation with reduction ) click( Disc dislocation without reduction click 872. Unilateral fracture of left condyle the mandible will: a) Deviate to the left side. B) Deviate to the right side. C) ..x d) ..x 873. Streptococcus activity detected by: a) Fermentation. B) Catalase.*** C) ..x d) x ) ( District Laboratory Practice Tropical Countries page 64 Catalase test This test is used to differentiate those bacteria that prodce the enzyme catalase as staphylococci from non-catalase producing bacteria such as streptococci

874. cleidocranial dysostosis characteristic: a) supernumerary of teeth. b) Clevical problems. c) Delayed closure of fontanelles d) all of above *** http://health.nytimes.com/health/guides/disease/cleidocranialdysostosis/overview.html#Symptoms Symptoms
Patients with cleidocranial dysostosis have a jaw and brow area that sticks out. The middle of their nose (nasal bridge) is wide. The collar bones may be missing or abnormally developed. This pushes the shoulders together in front of the body. Primary teeth do not fall out at the expected time. Adult teeth may develop later than normal, and extra set of adult teeth grow in. This causes the normal teeth to become crooked. The condition does not affect one's intelligence. Other symptoms can include

Ability to touch shoulders together in front of body Delayed closure of fontanelles Loose joints Occipital, parietal, and frontal bossing Short forearms Short fingers

875. To hasten Zinc Oxide cement, you add: a) Zinc sulfide. B) Barium sulfide. C) Zinc acetate. *** D) Barium chloride. Oxford Handbook of Clinical Dentistry - 4th Ed. (2005) - page 770 "Journal of Dental Research" 876. In which tooth the contact is at the incisal edge: a) Lower anterior teeth. *** b) x 877. Child with mental disorder suffer from orofacial trauma, brought to the hospital by his parents, the child is panic and Irritable, the treatment should done under: a) Local anesthesia.

B) General anesthesia. C) Gas sedation. D) Intravenous sedation. 878. Fracture before 1 year of upper central incisor reach the pulp in 8 year old child. How will you manage this case a) RCT. B) Apexification. *** c) Direct pulp capping d) Indirect pulp capping. http://www.ncbi.nlm.nih.gov/pubmed/10409837
When there is pulpal involvement of permanent teeth with incompletely formed roots, techniques for the induction of apical closure should be completed before endodontic therapy is begun. Apexification is a method of inducing a calcified barrier at the apex of a nonvital tooth with incomplete root formation. Apexogenesis refers to a vital pulp therapy procedure performed to encourage physiological development and formation of the root end

879. 8 years old child came to your clinic with trauma to upper central incisor with pulp exposure and extencive pulp bleeding your treatment will be: a. direct pulp cappin b. pulpectomy with gutta percha filling c.apexification d. pulpotomy with calcium hydroxide*** http://www.sciencedirect.com/science/article/pii/S0099239978801538
The exposed pulps of 60 permanent incisors with a complicated crown fracture were treated with partial pulpotomy and calcium hydroxide dressing. The interval between accident and treatment varied from one to 2,160 hours and the size of the pulpal exposure varied from 0.5 to 4.0 mm. Of the teeth, 28 had immature and 32 had mature roots. The treatment was successful in 58 teeth or 96% according to the following criteria: no clinical symptoms, no radiographically observed intraradicular or periradicular pathologic changes, continued development of an immature root, radiographically observed and clinically verified hard tissue barrier, and sensitivity to electrical stimulation. The follow-up examination varied from 14 to 60 months, with an average of 31 months.

Children generally develop their teeth before birth. Eruption starts at about six months, usually with the appearance of the lower incisors, and is complete by about two-and-a-half years. The table below will show an estimate chronology of tooth development: Baby teeth Incisors Canines 1st Molar 2nd Molar Calcification begins 4 mo (fetal life) 5 mo (fetal life) 6 mo (fetal life) 6 mo (fetal life) Appearance 6-9 mo 16-18 mo 12-14 mo 20-30 mo

Permanent teeth Lower incisors Upper incisors Lower Canines Upper Canines Premolars 1st Molars 2nd Molars 3rd Molars

Calcification begins 3-4 mo 4-5 mo 4-5 mo 5-6 mo 1.5-2.5 yrs Birth 2.5-3 yrs 7-10 yrs

Appearance 6-8 yrs 7-9 yrs 9-10 yrs 11-12 yrs 10-12 yrs 6-7 yrs 11-13 yrs 17-21 yrs

http://www.williamsdentalassociates.com/toothdevelopment.htm 880. 10 years pt come with necrotic pulp in upper central with root apex not close yet best treatment a. calcium hydroxide. b. calcific barrier.*** c. apexfication with gutta percha filling d. gutta percha filling Dental Decks - page 176 A premixed syringe of calcium hydroxide-methylcellulose paste is injected into the canal until it is filled to cervical level. The paste must reach the apical portion of the canal to stimulate the tissue to form a calcific barrier. The action of calcium hydroxide in prompting formation of a hard substance at the apex is best explained by the fact that calcium hydroxide creates an alkaline environment that promotes hard tissue deposition. http://www.ncbi.nlm.nih.gov/pubmed/10409837
When there is pulpal involvement of permanent teeth with incompletely formed roots, techniques for the induction of apical closure should be completed before endodontic therapy is begun. Apexification is a method of inducing a calcified barrier at the apex of a nonvital tooth with incomplete root formation. Apexogenesis refers to a vital pulp therapy procedure performed to encourage physiological development and formation of the root end

) ( 881. child 10 years came with trauma on the center incisor from year ago ,and have discoloring on it , in the examination ,no vitality in this tooth ,and in the x ray there is fracture from the edge of the incisal to the pulp ,and wide open apex the best treatment ? A)calcification. *** B)RCT with gutta percha

C)extraction D)capping 882. To remove a broken periodontal instrument from the gingival sulcus: a) Schwartz Periotriever. *** B) x http://www.johnnysilva.com/periodontal-disease/abc-1.html
Schwartz Periotrievers. Hie Schwartz Periotrievers are a set ol two double-ended, highly magnetized instruments designed for the retrieval of broken instrument tips from the periodontal pocket (I igs. 41-31 and 41-32). I hey are indispensable when the clinician has broken a curette tip in a furcation or deep pocket.w

a. b. c. d.

883. An 18 years old Pt present complaining of pain, bad breath and bleeding gingival. This began over the weakened while studying for the final exam. The Pt may have which of the following conditions: Acute necrotizing ulcerative gingivitis *** Rapidly progressive periodontitis Desquamative gingivitis. Acute periodontal cyst. http://www.ncbi.nlm.nih.gov/entrez/u...supplip114.pdf Thus, the former term, acute necrotising ulcerative gingivitis (ANUG) is summed up in necrotising gingivitis (NG( NG is a relatively rare disease and is generally described as existing in young adults between the ages of 18 and 30 years. It tends to emerge more frequently when the patient is under conditions of both physical and psychological stress. The disease is characterized by pain, bleeding and papillary necrosis with tendency to relapse Abrupt onset. If the patient has had prior outbreaks, he/ she is capable of reporting prodromal symptoms; for instance, a burning sensation in the gums . Poor general health status and low-grade fever Halitosis. ; it varies in terms of both intensity and degree. 884. Gingival condition occur in young adult has good oral hygiene was weakened

.ANUG .desqumative gingivitis .periodontitis .gingivitis "Tyldesley's oral medicine" The influence of poor oral hygiene in the initiation of ANUG has been often stressed, but there is no doubt that there are some patients whose standard of hygiene must be considered by normal criteria to be good. ) ( 885. Differences between ANUG and AHGS is: a. ANUG occur in dental papilla while AHGS diffuse erythematous inflamed gingival. b. ANUG occur during young adult and AHGS in children. c. All of the above. ***
Signs and symptoms
Clinical features of necrotizing periodontal disease may include:[2]

necrosis and/or punched out ulceration of the interdental papillae ("punched-out papillae")[3] or gingival margin pseudomembranous formation painful, bright red marginal gingiva that bleed upon gentle manipulation halitosis

Coincident factors may include heavy smoking and poor nutrition,[2] especially for those presenting with necrotizing ulcerative periodontitis.[3]

Treatment
Treatment includes irrigation and debridement of necrotic areas (areas of dead and/or dying gum tissue), oral hygiene instruction and the uses of mouth rinses and pain medication. As these diseases are often associated with systemic medical issues, proper management of the systemic disorders is appropriate.[2]

Prognosis
Untreated, the infection may lead to rapid destruction of the periodontium and can lips or the bones of the jaw. As stated, the condition can occur and be especially

spread, as necrotizingstomatitis or noma, into neighbouring tissues in the cheeks, dangerous in people with weakened immune systems. This progression to noma is

possible in malnourished susceptible individuals, with severe disfigurement possible.

http://en.wikipedia.org/wiki/Acute_necrotizing_ulcerative_gingivitis#Signs_an d_symptoms 886. Student, came to clinic with severe pain, interdental papilla is inflamed, student has exams, heavy smoker, poor nutrition. A. Gingivitis b. ANUG c. Periodontitis 887. 1212 - mypicx.com

The rows show truth, the column show test resulto a) Cell A has true positive sample.*** B) Cell A has true negative sample. C) Cell A has false positive sample. D) Cell A has false negative sample. 888. Distinguishing between right & left canines can be determined: a. because distal concavities are larger. b. with a line bisecting the facial surface the tip lies distally. c. others . . : . : Dental Decks - page 1602 Primary canine: We can differentiate between the upper mesial & distal canine by the functional cusp tip is inclined distally if bisectioning crown the cervical line on lingual surface is inclined mesially root curved mesially

889. The best way to remove silver point a) Steiglitz pliers. *** c) Ultrasonic tips d) H files e) Hatchet 890. The best way to remove silver point a) pliers and hemostate*** . c) Ultrasonic tips d) H files A grasping tool such as the Stieglitz pliers (Henry Schein) can generally get a strong purchase on the coronal end of a silver point and then, utilizing the concept of fulcrum mechanics, elevate the silver point out of the canal. Indirect ultrasonics is another important method to remove silver points. It is not wise to place any ultrasonic instrument directly on the silver point because it will rapidly erode away this soft material. And Endodontics Problem solving in clinical practice 2002 page 142 Cement can be removed carefully from around the point using a Piezon ultrasonic unit and CT4 tip or sealer tip. Great care must be taken not to sever the point and damage the coronal end. The point is withdrawn using Stieglitz forceps or small-ended artery forceps

. 891. Mucoceles the best tx is: a) Excision*** b) leave it c) marsupialization d) cauterization 7:3 892. Isolation period of chickenbox should be: -after appear of rach by week -untill vesicle become crusted. *** -until carter stage is last ) ( Dental decks - page1308 Its most contagious one day before the onset of rash and until all vesicle have crusted. 893. In fixed p.d u use gic for cemntation what best to do -remove smear layer by acid to increase adhesion -do not varnish because it affect adhesion. *** -mixed slowly on small area untill become creamy -remove excess when it in dough stage "Contemporary Fixed Prosthodontics ROSENSTIEL page 772"


43 . For glass ionomer cement, the measured powder is divided into two equal parts and mixed with a plastic spatula. The first increment is rapidly incorporated in 10 seconds, and the second increment is incorporated and mixed for an additional 10 seconds. .

Avoid using cavity cleaners to aid in drying the preparation because they may adversely affect pulpal health. Remove excess cement with an explorer. Early cement removal may lead to early moisture exposure at the margins with increased solubility. 894. Child have tooth which have no moblity but have luxation best treatment: -acrylic splint. -non rigid fixation. *** -rigid fixation. Handbook of dental trauma 2001 page 71 Luxation with immobilization treatment: Non-rigid `physiological' splinting should be applied for a period of no more than two weeks. The various splinting methods are discussed. 895. To drain submandibular abscess: a) Intraorally through the mylohyoid muscles. B) Extraorally under the chin. C) Extraorally at the most purulent site. D) Extraorally at the lower border of the mandible. *** In case of abscess you incise in healthy tissue to be sure good blood flow to the incision line so good healing of incision line with no scar.. If you incise in most purulent area which is not healthy tissue, the result is a scar in incision line "Oral and maxillofacial surgery - Jonathan Pedlar page96" 896. Scale to measure marginal deterioration: 1. Mahler scale.*** 2. Color analogues scale. "Art & Science - page 158" Progression of the events to deeper or more extensive ditching has been used as visible clinical evidence of conventional amalgam deterioration and was the basis of the mahler scale 897. ST.coccus mutans cause caries & this disease is? 1/ epidemic 2/endemic. *** 3/isolated endemic disease: An infectious disease that is present in the community at all times but normally at low frequency GESAMP Glossary

Endemic: a disease that exists permanently in a particular region or population. Malariais a constant worry in parts of Africa. Epidemic: An outbreak of disease that attacks many peoples at about the same time and may spread through one or several communities. Pandemic: When an epidemic spreads throughout the world 898. One of these has no effect on the Life span of handpiece: a-Low Air in the compressor. *** b-Trauma to the head of the hand piece c-Pressure during operating 899. Most used Scalpel in oral surgery: a-Bard Parker blade # 15. *** 900. Advantage of Wrought Wire in RPD over Cast Wire: a-Less irritation to the abutment. 901. Why we use acrylic more than complete metal palate in complete denture: - Cant do relining for the metal. *** 902. Relining of denture: - remove all or part of fitting surface of the denture and add acrylic - add acrylic to the base of the denture to increase vertical dimension. *** 903. Rebasing of Complete Denture mean: a-Addition or change in the fitting surface b-Increasing the vertical dimension c-Change all the fitting surface. *** 734 : Rebasing is replacement of most or all of the denture base. . 904. When Do class I preparation of posterior tooth for Composite Restoration: a-remove caries only. *** b-extend 2mm in dentin 905. Color Stability is better in: a-Porcelain. *** b-Composite c-GIC 906. when all the teeth are missing EXCEPT the 2 canines , according to kennedy classification it is: a- Class I modification 1. *** ( )

907. Antibiotics are most used in cases of: a-Acute Localized lesion b-Diffuse , Highly progressing lesion. *** 908. Patient un-cooperation can result in fault of operation, Technical faults ONLY are related to patient factor TRUE. FALSE. *** 909. Bone graft material from site to another site in the same person a-allograft b-auto graft *** c-alloplast d-xenograft
Autograft: Tissue transplanted from one part of the body to another in the same individual. Also called an autotransplant. Allograft: The transplant of an organ or tissue from one individual to another of the same species with a different genotype. A transplant from one person to another, but not an identical twin, is an allograft. Allografts account for many human transplants, including those from cadaveric, living related, and living unrelated donors. Called also an allogeneic graft or a homograft. Xenograft: A surgical graft of tissue from one species to an unlike species (or genus or family). A graft from a baboon to a human is a xenograft. The prefix "xeno-" means foreign. It comes from the Greek word "xenos" meaning stranger, guest, or host. (Xeno- and xen- are variant forms of the same prefix.)

http://www.medterms.com/script/main/art.asp?articlekey=40486 910. ester type of local anathsesia secreted by a-liver only b-kidney c-lung d-plasma. *** Dental Decks - page 2376 ) ( 911. where does the breakdown of Lidocaine occurs : A) kidneys B) Liver. *** Dental Decks - page 2362 912. Energy absorbed by the point of fracture called a-ultimate strengh b-elastic limit c-toughness *** d-britlness Toughness It is defined as the amount of energy per volume that a material can absorb before rupturing.

Toughness - Wikipedia, the free encyclopedia The ability of a metal to deform plastically and to absorb energy in the process before fracture is termed toughness. Toughness 913. patency filling a-push the file apically to remove any block at the apex *** b-rotate the file circumferentially at the walls to remove any block of lateral canals. c-rotary files circumferentially at the walls to remove any block of lateral canals. D-file with bleaching agent. 914. best stress transfer under amalgam a-with thin base layer. b-with thick base layer. *** c-if put on sound dentin. 915. Tooth with full crown need RCT, you did the RCT through the crown, what is the best Restoration to maintain the resistance of the crown: A) Glass ionomer resin with definite restoration. B) amalgam 916. 5 years old pt had extraction of the lower primary molar & he had fracture of the apex of the tooth what is the best ttt: 1/ aggressive remove 2/ visualization & remove 3/ visualization & leave. *** 917. 7 y/o boy came to the clinic in the right maxillary central incisor with large pulp exposure: 1/ pulpectomy with Ca(OH)2 2/ pulptomy with Ca(OH)2. *** 3/ Direct pulp capping 4/ leave it

Children generally develop their teeth before birth. Eruption starts at about six months, usually with the appearance of the lower incisors, and is complete by about two-and-a-half years. The table below will show an estimate chronology of tooth development: Baby teeth Incisors Canines 1st Molar 2nd Molar Calcification begins 4 mo (fetal life) 5 mo (fetal life) 6 mo (fetal life) 6 mo (fetal life) Appearance 6-9 mo 16-18 mo 12-14 mo 20-30 mo

Permanent teeth Lower incisors Upper incisors Lower Canines Upper Canines Premolars 1st Molars 2nd Molars 3rd Molars

Calcification begins 3-4 mo 4-5 mo 4-5 mo 5-6 mo 1.5-2.5 yrs Birth 2.5-3 yrs 7-10 yrs

Appearance 6-8 yrs 7-9 yrs 9-10 yrs 11-12 yrs 10-12 yrs 6-7 yrs 11-13 yrs 17-21 yrs

918. old pt came to replase all old amalgam filling he had sever occlusal attriation the best replacement is: 1/ composite 2/ amalgam 3/ cast metal restoration 4/ full crowns. *** 8 673 919. Child has bruxism to be treated with a. sedative b. cusp capping c. vinyl plastic bite guard. *** "Dentistry for Child and Adolescent - Page 646" A vinyl plastic bite guard that covers the occlusal surfaces of all teeth plus 2 mm of the buccal and lingual surfaces can be worn at night to prevent continuing abrasion. The occlusal surface of the bite guard should be flat to avoid occlusal interference 920. avulsed tooth: 1/ splint (7-14) day. *** 2/ or (3- 14) weeks 921. proxy brush with which type of furcation: Furcation Grade 1-1 Furcation Grade 2-2 Furcation Grade 3-3. Furcation Grade 4-4 922. instrument which we use to make groove in the wax is Curver 923. instrument used for wax grooving for a die in FPD -NKT no

-NKT no. -Spoon excavator Burnisher. 924. After u did upper& lower complete denture for old pt. He came back to the clinic next day complaining of un comfort with the denture. After u re check ,no pain, good occlusion, good pronunciations , but u notice beginning of inflammation in the gum and outer margins of the lips , u will think this is due to: 1- xerostomia. 2-vit-B deficiency 3- sclero edema Xerostomia can cause difficulty in speech and eating. It also leads to halitosis and a dramatic rise in the number of cavities, as the protective effect of saliva's remineralizing the enamel is no longer present, and can make the mucosa and periodontal tissue of the mouth more vulnerable to infection 925. Patient comes to your clinic with complete denture for Routine visit no complenining .During speech or swallowing or opening the mouth just glossitis Angular Cheilitis and discomfort increasing while day a. Vitamin B deficiency b. Xerostomia c. sclero edema ) ( 926. Patient with leukemia absolute neutrophilic count is 1700 what oral surgeon should do go on the manager a. as usual pt-*** b. postpone another day c. work with prophylactic antibiotic. d. platelets transfusion 927. child came to your clinic have leukaemia number of neutrophils are (1400)want to extract his primary central incisor will you treat him : a. as usual ptb. give prophylactic (antibody ) c. give platlets before extractiond. do not extract

928. child came to your clinic have leukaemia number of neutrophils are less than (1500)want to extract his primary central incisor will you treat him: a. as usual ptb. give prophylactic (antibody ) c. give platlets before extractiond. do not extract http://www.fpnotebook.com/HemeOnc/Lab/NtrphlCnt.htm
1. Normal Count 1. Range: 50-70% of White Blood Cells 2. Bands: 2-6% of White Blood Cells 3. Absolute Neutrophil Count (ANC) >1500 Neutrophils/mm3

929. Galvanic shock a. Put separating medium. b. Wait. *** c. put varnish. : . ( ) And Dental Decks - page 2268 "Galvenic shock: generally it gradually subsides and disappears in a few days" 930. An 8 years old child, suffered a trauma at the TMJ region as enfant. Complaining now from limitation in movement of the mandible. Diagnosis is: a) Sub luxation b) Ankylosis. *** 931. Genralised lymphadenopathy seen in a- infection b- lymphocytic leukemia c- HIV d- perncious anemia a+b a+b+c. *** only d b+d

Causes of generalized lymphadenopathy Infection : Viral : Infectious mononucleosis, Infective hepatitis, AIDS Bacterial : Tuberculosis, Brucellosis, 2ry syphilis Protozoal : Toxoplasmosis Fungal : Histoplasmosis Malignant : Leukaemia Lymphoma - Metastatic carcinoma Immunological : Systemic lupus erythematosus - Felty's syndrome - Still's disease Drug hypersensitivity as Hydantoin, Hydralazine, Allopurinol Misc. : Sarcoidosis Amyloidosis - Lipid storage disease - Hyperthyroidism 932. Contraindication of gingivectomy a-periodontal abscess 933. surgical interference with edentulous ridge for a-good retention, stability and continuous uniform alveolar ridge. *** 934. The tip of size 20 endo file is: - 0.02 m.m - 0.2 m.m*** 935. Bonding agent for enamel we use: -unfilled resin. *** -primer & adhesive bonding agent. -resin dissolve in acetone or alcohol. -primer with resin modified glass ionomer. Dental secrets page 188 1- The etchant: phosphoric acid, nitric acid, or another agent that is used to etch enamel and/or precondition the dentin . 2- The primer: a hydrophylic monomer in solvent, such as hydroxymethalmethacrylate. It acts as a wetting agent and provides micromechanical and chemical bonding to dentin 3- The unfilled resin is then applied and light or dual-cured. This layer can now bond to composite, pretreated porcelain luted with composite, or amalgam in some products. 936. We redo high copper amalgam restoration when we have: a-amalgam with proximal marginal defect. *** >food accumulation b-open margin less than 0.5 mm 937. Thickness of amalgam in complex amalgam restoration in cusp tip area: - 0.5 mm - 1-1.5 mm - 1.5-2 mm -2-3 mm. ***

Dental Decks - page 2170 Working cusp reduction for amalgam 2.5-3 mm 938. For cavity class II amalgam restoration in a second maxillary premolar, the best matrix to be Used: A) Tofflemire matrix. *** B) Mylar matrix C) Gold matrix D) Celluloid strips Summery of Operative dentistry page 220 Types of matrices Metal Firm, used for amalgam restorations. Mylar Easily mouldable and can light-cure through; used for resin composite. Plastic Rigid, can light-cure through; used in Class V cavities. Difficult cases In deep subgingival cavities use of special matrices such as tofflemire or automatrix or copper bands often achieve better contact points and marginal adaptation. Occasionally electrosurgery required to permit matrix adaptation. 939. Cast with (+ve) bubble b/c of: - Mixing stone - Voids in impression when taken by the dentist*. -pouring -using warm water when mixing ston. )( 940. Non absorbable suture: -catgut -vicryl -silk. *** Dental Decks - page 1774 941. The following is a non- absorbable suture: A. Plain catgut. B. Chromic catgut. C. Silk.*** D. All the above. 942. most important criteria for full ceramic FPD: -high compressive strength. *** - High tensile strength.

Restorative dental materials 2002 page 5 Because ceramics are stronger in compression than in tension, this property is used to advantage to provide increased resistance to shattering. 943. Reciprocal arm in RPD help to resist the force applied by which parts: -retentive arm. *** - guide plane and 944. Pt come with pain tooth #.. When drink hot tea . Pain continuous for 10 minutes diagnosis: - irreversible pulpitis *** - necrotic 945. Pulp with age: - reduce collagen fiber - Increase cellular in pulp - decrease pulp chamber size.*** 946. Pt need complete denture u take impression with irreversible hydrocolloid & poured it after late more than 15 min the cast appear sort & chalky the reason is: a. Dehydration of the impression. *** b. Expansion of the impression c. Immerse the impression in a chemical solution Oxford 947. after insertion of immediate complete denture, pt remove denture at might next day he couldn`t wearing it .and came to why this is happened a. relife b. Swelling And inflammation after extraction c.lack of skills for the patient to put the denture 948. Over extended GP should remove using: a- ultrasonic vibrating. b- dissolving agent. c- rotary or round bur d- surgery 949. Sterilization mean killing: - Bacteria and virus -. Bacteria, virus, fungus and protozoa. *** - Bacteria and fungus 950. Killing Bacteria is: - Bacteriostatic - Bactericidal. *** 951. The most technique use with children: - TSD***(Tell- Show Do)

- hand over mouth -punishment ) ( 952. Chronic pericoronitis: - Difficult mouth opening - Halitosis -all of the above. *** 953. Safe months to treat pregnant ladies: - 1-3 -4-6. *** -7-9. 954. Mandibular 1st permanent molar look in morphology as: a- primary 1st mand molar. b- primary 2nd mand molar. *** c- primary 1st max molar. d- primary 2nd max molar. 955. Material which used for flasking complete denture: a- plaster. *** b- stone c-refractory a+bhttp://webcache.googleusercontent.com/search?hl=ar&site=webhp&gs_ sm=e&gs_upl=5765l18572l0l19897l1l1l0l0l0l0l0l0ll0l0&safe=active&q=ca che:o2F1_V_B6LAJ Flasking were done in four groups: Group 1: Plaster- Mixed, Group 2: Plaster-Stone- Mixed, Group 3: Plaster- Mixed- Stone, Group 4: Plaster)Stone- Plaster 956. Ideal properties of RC filling material is the following EXCEPT: a) Radiolucent in radiograph. *** b)Not irritate the surrounding tissue c) Easily removable when retreatment is necessary d)Stable and less dimensional change after insertion 957. The best method for core build up is: 1. Amalgam. *** 2. Compomer. 3. Glass ionomer. http://dfd.atauni.edu.tr/UploadsCild/files/2007-1/2007_1_4%20.pdf light-cure composite resin (President) and amalgam (Cavex avalloy) were statistically different than the other materials tested. They are stronger than

compomer followed by resin modified glass ionomer and conventional glass ionomer core materials. 958. best core material receiving a crown on molar: a)amalgam. *** b)reinforced glass ionomer. d)composite Oxford Handbook of Clinical Dentistry - 4th Ed. (2005) page 158 Direct method Pre-formed posts are cemented into one or more canals. Amalgam may also be Packed into the coronal aspect of the root canals (Nayyar core technique) and an amalgam core Built up, which is the preferred technique. Resin modified GI or resin composite may also be used. These materials have the advantage that the preparation can be completed at the same visit. A Dentine adhesive system should be used with resin composite to enhance retention. Fundamentals of fixed prosthodontics page 185 composite resin cores exhibit greater microleakage than do amalgam cores,29 and they are not as dimensionally stable as amalgam in an vitro study. crowns made for teeth with composite resin cores failed to seat by 226 um more than crowns made for teeth with amalgam cores after immersion in bodytemperature normal saline solution for 1 week. The surface of a composite resin core is affected adversely by exposure to zinc oxide-eugenol temporary cement,31 although that does not seem to have a negative effect on the tensile strength of the final crown. 959. most comon site which drain pus is: a) mandibular central incisors b)mandibular canines c) mandibular first molar. *** d).. 960. When removing moist carious dentin which exposes the pulp, dentist should: 1- do direct pulp cap 2- do indirect pulp cap 3- prepare for endo. *** "Dental secrets page 167" There is general agreement that carious exposure of a mature permanent tooth generally requires endodontic therapy. Carious exposure generally implies bacterial invasion of the pulp, with toxic products involving much of the pulp.

However, partial pulpotomy and pulp capping of a carious exposure in a tooth with an immature apex have a higher chance of working. "Dental pulp 2002 page 335" Cavity Cleansing, Disinfection, and Hemorrage Control A clinical review failed to support direct pulpcapping or pulpotomy procedures in teeth when a mechanical exposure pushes infected carious operative debris into the subjacent pulp. Because of the stigma of long-term failures, our profession generally selects traditional endodontic treatment. Only in the treatment of pulp exposures in fractured young anterior teeth with open apices does the literature discuss pulpotomy or direct pulp-capping with Ca(OH)2. 961. In prevention of dental caries, the promotion of a healthy diet is: 1- low effective measure 2- moderately effective measure 3- high effective measure 4- mandatory measure 962. Treatment of cervical caries in old patients with a temporary restoration is best done by: a)Glass ionomer. *** b)composite resitn c) d).. 963. Most used sugar substitute: a) Sorbitol b) Mannitol c) Insulin d) Xylitol *** Xylitol is a "tooth-friendly," nonfermentable sugar alcohol A systematic review study[13] on the efficacy of xylitol has indicated dental health benefits in caries prevention, showing superior performance to .)other polyols (polyalcohols Xylitol is specific in its inhibition of the mutans streptococci group, bacteria .that are significant contributors to tooth decay

XYLITOL GUM Xylitol is a naturally occurring, low-calorie sugar substitute with anticariogenic properties. It is a sugar alcohol, derived mainly from birch and other hardwood trees.

Xylitol contains 40% fewer calories than sucrose. Data from recent studies indicate that xylitol can reduce the occurrence of dental caries in young children.

964. What the influence of xylitol a. It cause caries b. Safe to the teeth*** c. Increase saliva d. Decrease saliva 965. the retainer of rubber dam a)four points of contact two buccally and two lingually without rocking. *** b) four points of contact two buccally and two lingually above the height of contour c) four points of contact two mesially and two distally d) 2 points Elastomeric cord for retaining a dental dam, cord dispenser and related combinations and methods - Patent 5104317 dental dam retainer clamp is that the four prongs must contact the tooth, if they do not, the clamp may need to be ground. a properly selected retainer should contact the tooth in four areas-two on the facial surface and two on the lingual surface. This four- point contact prevents rocking or tilting of the retainer. Dental Decks - page 2220 All four points of the jaws of the clamp must contact the tooth gingival to the height of contour. 966. Mandibular fracture other complications: 1_nasal bleeding 2- exophthalmos 3-numbness in the infraorbital nerve distribution. *** 967. Hypercementosis and ankylosis is seen in a: paget disease. *** b: monocytic fibrous dysplasia c: hyperparathirodism 968. The infection will spread cervically in infection from a: lower incisors b lower premolars c: lower 2nd and 3rd molars. *** d: upper incsisors 969. In hypertension patient the history is important to detect severity a: true. *** b: false 970. in inflamed muocosa due to wearing denture to when do new denture a: immediatly

b: after week c: put tissue conditioning material and wait until the tissue heal and take impression after 2 weeks. *** 971. forceps to hold flap when suturing adson's *** Oral Surgery page 46 Surgical forceps are used for suturing the wound, firmly grasping the tissues while the needle is passed. There are two types of forceps: the long standard surgical forceps, used in posterior areas, and the small, narrowAdsonforceps,usedinanterior areas.

972. Pulp oedema 1- has no effect on vascular system 2- fluid is compressed in the vessels limiting the intercellular pressure 3- interstitial pressure increased due to increased vascularity *** 4- cause necrosis of the pulp tissues 973. master cone doesn't reach the apex 1)ledge 2)residual remenants (debris) 3)...... 4) 1&2 *** 974. pterygomandibular raph. Insertion & origin muscles should be medial to the injection all of the above. *** 975. The divergence should be mesiodistally for an amalgam restoration a. no it should be convergent b. if the remaining proximal marginal ridge = 1.6 mm c. if the remaining proximal marginal ridge only > 1.6 d. if the remaining proximal marginal ridge only < 1.6*** 5 4.9 .

Dental Decks - page 2298

976. Patient with radiopacity in the periapical area of a 1st mandibular molar with a wide carious lesion and a bad periodontal condition is: a. condensing osteosis *** b. hypercementosis 977. Patient had anaphylactic shock due to penicillin injection , what's the most important in the emergency treatment to do: a. 200 mg hydrocortisone intravenous b. 0.5 mg epinephrine of 1/10000 intra venous c. adrenaline of 1/1000 intra muscular. *** Http: //http: //www.wrongdiagnosis.com...y/treatment... Anaphylaxis is always an emergency. It requires an immediate injection of 0.1 to 0.5 ml of epinephrine 1: 1,000 aqueous solution, repeated every 5 to 20 minutes as necessary. If the patient is in the early stages of anaphylaxis and hasnt yet lost consciousness and is still normotensive, give epinephrine I.M. or subcutaneously (S.C.), helping it move into the circulation faster by massaging the injection site. For severe reactions, when the patient has lost consciousness and is hypotensive, give epinephrine I.V. Management Place patient supine with legs raised, if possible. 0.5 ml of 1:1000 adrenaline IM or SC. Repeat after 15 min, then every 15 min until improved. Do not give IV in this concentration as it will induce ventricular fibrillation. Up to 500 mg of hydrocortisone IV. Up to 20 mg of chlorpheniramine slowly IV (if available(. O2 by mask : %5 1/1000 %3.8-3.4

978. bronchial asthma epinephrine concentration subcutaneously A) 1/1000. *** B) 1/10000 C) 1/100000 Dental secrets page 50 If the reaction is immediate (less than 1 hour) and limited to the skin, 50 mg of diphenhydramine should be given immediately either intravenously or intramuscularly. The patient should be monitored and emergency services contacted to transport the patient to the emergency department. If other symptoms of allergic reaction occur, such as conjunctivitis, rhinitis, bronchial constriction, or angioedema, 0.3 cc of aqueous 1/1000 epinephrine should be given by subcutaneous or intramuscular injection. The patient should be monitored until emergency services arrive. If the patient becomes hypotensive, an intravenous line should be started with either Ringers lactate or 5% dextrose/water. 979. which is contraindicated to the general anaesthia: a. patient with an advanced medical condition like cardiac .*** b. down's syndrome patient c. child with multiple carious lesion in most of his dentition d. child who needs dental care, but who's uncooperative, fearfuletc 980. continuous condensation technique in gp filling is: a. obtura I b. obtura II c. ultrafill d. System B. *** . 981. best material for major connector. a. Gold wrought wire b. chrome cobalt *** c. gold palladium d. titanium 982. on a central incisor receiving a full ceramic restoration, during finishing of the shoulder finish line subgingivally Diamond end cutting

983. in a class III composite with a liner underneath, what's the best to use a. light cured GI. *** b. zno Eug c. Reinforced znoeug 984. outline of 2nd molar Access Opening Triangular with the base mesially*** ) ( 985. The outline form of upper maxillary molar access opening is Triangular, The base of this triangle is directed toward : A) Buccal. *** B) Palatal C) Mesial D) Distal 986. after usage of sharp scalpels, needles, what's the best management 1) throw in a special container of sharp instrument. *** 2)sterilize and re use 3) through in ordinary plastic waste basket 987. sharping of hand instrument mounted air driven better than unmounted due to A) fine grift. *** B) sterilization C) ability to curve instrument The mounted-stone technique. The second technique for sharpening dental instruments is the mounted-stone technique. This technique is especially useful in sharpening instruments with curved or irregularly shaped nibs. Equipment consists of mandrel-mounted stones, a straight handpiece, lubricant, two-inch by two-inch gauge, and again, the instrument to be sharpened. Mounted stones are made of two materials, Arkansas stones and ruby stones (sometimes called sandstones). Ruby stones are primarily composed of aluminum oxide. The ruby stone is comparatively coarse, has a rapid cutting ability, and is used for sharpening instruments that are dull. Mounted stones are cylindrical in shape and appear in several sizes. They have a fine grit and are used with the straight handpiece. The stones permit rapid sharpening, but without extreme care, will remove too much metal and may overheat the instrument. Overheating the instrument will destroy the temper, thereby causing the instrument to no longer hold a sharp edge.

988. Unmounted sharpening instruments are better than mounted because: a. has finer grains b. don't alter the bevel of the instrument c. easier to sterilize d. less particles of the instruments are removed. *** = cut less of the plade. Veterinary dentistry: principles and &ved=0CA0Q6AewAQ&safe=active#v=onepage&q&f=fals e Unmounted stones are customarly preferred as they are kinder on 207nstrument by removing less metal in the sharpening process "Clinical Aspects of Dental Materials: Theory, Practice, and Cases, 3 rd Edition" 989. Pt came to the clinic complaining from pain related to swelling on maxillary central incisor area with vital to under percussion? 1/ periapical cyst 2/incisive cyst( nasopalatin cyst). *** 3/ globulomaxillary cyst 4/ aneurysmalbone cyst 990. Pt came to the clinic with a lesion confined to the middle of the hard palat, on the clinical examination the lesion is fluctuant & tender. On the X-ray radiolucent area btw the two central incisors roots. The diagnosis will be: a- Glubulomaxillary cyst. b- incisive canal cyst. c- Bohn's nodule. d- Aneurysmal bone cyst. http://radiopaedia.org/articles/incisive-canal-cyst 991. Y/o boy came to the clinic in the right maxillary central incisor with large pulp exposure: 1/ pulpectomy with Ca(OH)2 2/ pulptomy with Ca(OH)2. *** 3/ Direct pulp capping 4/ leave it

Children generally develop their teeth before birth. Eruption starts at about six months, usually with the appearance of the lower incisors, and is complete by about two-and-a-half years. The table below will show an estimate chronology of tooth development: Baby teeth Incisors Canines Calcification begins 4 mo (fetal life) 5 mo (fetal life) Appearance 6-9 mo 16-18 mo

1st Molar 2nd Molar Permanent teeth Lower incisors Upper incisors Lower Canines Upper Canines Premolars 1st Molars 2nd Molars 3rd Molars

6 mo (fetal life) 6 mo (fetal life) Calcification begins 3-4 mo 4-5 mo 4-5 mo 5-6 mo 1.5-2.5 yrs Birth 2.5-3 yrs 7-10 yrs

12-14 mo 20-30 mo Appearance 6-8 yrs 7-9 yrs 9-10 yrs 11-12 yrs 10-12 yrs 6-7 yrs 11-13 yrs 17-21 yrs

992. The percentage of simple caries located in the outer wall of the dentin (proximal sides of the tooth) which left without cavitations is around: 1-10% 2-30% 3-60% *** 4-90% Art & Science of Perative Dentistry Page 102 approximately 60% of teeth with radiographic proximal lesions in the outer half of dentin are likely to be noncavitated. 993. Irrigation solution for RCT ,when there is infection and draining from the canal is a) Sodium hypochlorite b) Iodine potassium c) sodium hypochlorite and iodine potassium. *** "Pocket Atlas of Endodontics page 154" Therefore, like citric-acid rinsing, EDTA solution is recommended before the placement of calcium hydroxide. At a 15% concentration, citric acid has been shown to be very effective against anaerobic bacteria. Solvidont, a bisdequalium acetate, exhibits good antibacterial properties, but also an unfavorable relationship between cytotoxicity and antibacterial efficiency. Physiologic saline (nacl) is by far the most tissue-friendly rinsing solution, but its antibacterial effect is quite low. Iodine and also potassium iodine are good antiseptics with equally good tissue biocompatibility. Betadine is the commercially available product.

With paraformaldehyde or phenol-containing solutions, on the other hand, the tissue toxicityis higher than the antibacterial efficacy. 994. Irrigation solution for RCT ,when there is infection and draining from the canal is a) Sodium hypochlorite b) Iodine potassium c) sodium hypochlorite and H2O2. *** 995. trigeminal neuralgia treated by carbomizapine, the max dose per day divided in doses is: a-200 mg B-500mg C-1000mg D-1200mg 437 : 4533-933 Usual Adult Dose for Trigeminal Neuralgia Initial dose: 100 mg orally twice a day (immediate or extended release) or 50 mg orally 4 times a day (suspension). May increase by up to 200 mg/day using increments of 100 mg every 12 hours (immediate or extended release), or 50 mg four times a day. (suspension), only as needed to achieve freedom from pain. Do not exceed 1200 mg/ day. Maintenance dose: 400 to 800 mg/day. Some patients may be maintained on as little as 200 mg/day while others may require as much as 1200 mg/day. At least once every 3 months throughout the treatment period, attempts should be made to reduce the dose to the minimum effective level or to discontinue the drug Read more: Carbamazepine Dosage Drugs.com 996. 10 years child with congenital heart disease came for extraction of his lower 1st molar, the antibiotic for choice for prevention of infective endocarditis is; a-ampicelline 30 mg /kg orally 1hour before procedure b-cephalixine 50mg/kg orally 1hour before procedure c-clindamicine 20mg/kg orally 1hour before procedure d-amoxicilline 50mg/kg orally 1hour before procedure Dental secrets "Amoxicillin, 2.0 gm orally 1 hr before procedure"

997. What is uses if microscop??? To see metaobolic. To see live cells. *** To see dead cells. 998. -best way to detect presence of 2 canals putting 2 files & take x-ray *** 999. Patient has a palatal torus b/w hard & soft palate, the major connector of choice a. anteroposterior palatal strapb. u shaped *** c. posterior palatal strap1000. White lesion bilaterally on cheek,& other member in the family has it a -leukoplakia b -white sponge nevus. *** others Dental Decks - page 1336 "Burket- Oral medicine" White sponge nevus presents as bilateral symmetric white, soft, spongy, or velvety thick plaques of the buccal mucosa. And White sponge nevus, also known as Cannon's disease, Hereditary leukokeratosis of mucosa, and White sponge nevus of Cannon[1] appears to follow a hereditary pattern as an autosomal dominant trait.[2]:807 Although it is congenital in most cases, it can occur in childhood or adolescence. It presents in the mouth, most frequently as a thick bilateral white plaque with a spongy texture, usually on the buccal mucosa, but sometimes on the labial mucosa, alveolar ridge or floor of the mouth 1001. Pt construct for him a complete denture after few days he came to u complaining from pain & white spots on the residual ridge do relief in that area & give him ointment & after few days he came again complaining the same but in another area the main cause is : a. Uneven pressure on the crest of alveolar ridge. *** b. Increase vertical dimension 1002. After final inlay cementation and before complete setting of cement we should: a-remove occlusal interferences b-burnishing of peripheries of restoration for more adaptation. *** c-lowering occlusal surface "Pickard's Manual of Operative Dentistry Eighth edition OXFORD page 186"

It is easier to remove excess cement before it finally sets. Once the cement is hard the rubber dam is removed and the occlusion checked with articulating paper and adjusted with fine diamond burs. 1003. pt with renal dialysis the best time of dental tx is: a-1 day before dialysis b-1 day after dialysis. *** c-1week after dialysis Dental secrets page 54 Patients typically receive dialysis 3 times/week. Dental treatment for a patient on dialysis should be done on the day between dialysis appointments to avoid bleeding difficulties. . 1004. Pt presented with vehicle accident u suspect presence of bilateral condylar fracture what is the best view to diagnose condylar Fracture: 1. Occiptomenatal. 2. Reverse towne. *** 3. Lat oblique 30 degree. Reverse towne for fracture of condylar neck &ramus areas (dental decks) Reverse Townes position, beam 30 up to horizontal. Used for condyles. (Oxford) 1005. Female come need to endodontic for central insical ,and have media composite restorations in the mesial and distal walls ,and have attrition in the insicial, edge the best restoration? 1. Jacket crown. 2. Full crown. *** 3. Metal crown. 1006. Child have dental caries in 3 or 4 surfaces of his first primary molar wewill replace them with: -preformed metal crown. -porcaline crown -amulgam crown -composite restoration 1007. The nerve which supplies the tongue and may be anesthetized during nerve block injection: 1. V. 2. VII. ***

3. IX. 4. XII. Dental decks 1904 (notean injection into parotid capsule during nerve block injection MAY CAUSE Bell`s palsy facial expretion) and The sensory portion of facial N supplies the taste to the anterior two-thirds of the tongue) Cranial Nerve VII: The Facial Nerve and Taste -- Clinical Methods -- NCBI Bookshelf
If needle is positioned too posteriorly, anaesthetic may be put into parotid gland (dangerous systemic effects), or paralyse Cranial Nerve VII (7), resulting in Bells Palsy-like symptoms. Also if the needle is placed too medially the medial pterygoid muscle can be injected, resulting in trismus.

The sphenomandibular ligament is most often damaged in an inferior alveolar nerve block http://en.wikipedia.org/wiki/Inferior_alveolar_nerve_anaesthesia#Injection_tec hniques

1008. Electro surgery rate: a. 1.5 7.5 million cycle per seconds. *** b. 7.5 10 million cycle per seconds. c. 10 25 million cycle per seconds. d. 30 million cycle per seconds. Caranza periodontology page 582 1009. Pt came to the clinic after he has an accident. X-ray revealed bilateral fracture of the condoyle. Mandible movements are normal in all direction. What is your treatment? 1. Inter maxillary mandibular fixation. 2. Fixed IMF for 6 weeks. 3. Inter mandibular fixation. 4. No treatment is performed only anti inflammatory drugs and observation. 1010. 9 year old Pt came to the clinic after he has an accident. X-ray revealed bilateral fracture of the condoyle. Mandible movements are normal in all direction. What is your treatment? 1. Inter maxillary mandibular fixation. 2. Fixed IMF for 6 weeks. 3. Inter mandibular fixation. 4. No treatment is performed only anti inflammatory drugs and observation.

1011. 6 years old patient received trauma in his maxillary primary incisor, the tooth is intruded. The permanent incisors are expected to have: 1. Yellowish or whitish discoloration. *** 2. Displacement. 3. Malformation. 4. Cracks in enamel. 1012. 6 years old patient received trauma in his maxillary primary incisor, the tooth is intruded. The permanent incisors are expected to have: a. Displacement. b. Malformation. c. Cracks in enamel. d. Yellowish or whitish discoloration with hypoplasia. *** Atlas Of Oral Medicine page 151 Enamel hypoplasia : Trauma or infections of developing teeth. 1013. Head and neck nevi with multi lesion is: 1/Eagle syndrome. 2/Albert syndrome. *** (Albright syndrome) Albright Apert Albert : ::<8 . %58 1014. Avulsed teeth with replantation, dentist evaluate prognosis with : 1/flexible wire 2/ridge wire 3/in follow-up pd wire ttp://www.doctorspiller.com/avulsed_teeth.htm Spilinting the tooth in position: The ideal splint for avulsed teeth is a flexible splint. These are typically made using Gortex or other synthetic cloth or metallic mesh strips made for this purpose. Other types of flexible splint may be made using thin orthodontic wire. Ideally, the splint should encompass several teeth on either side of the avulsed tooth. There are quite a few options depending on the comfort level of the practitioner. The recommendation for flexiblity involves theoretical considerations in the formation of the new periodontal ligament. However, since the splint is kept in place for no more than 7 to 10 days, the flexibility factor may be of little practical significance. This is my personal opinion. I'm sure others would argue the point.

1015. Pt need complete dt, when u did the examination u notice the max tubersity will be interfere with dt 1/need 12 no blade to be extention. *** 2/partial thickness flap extend buccal & palatal 3/suture under tension Peterson's Principles of Oral and Maxillofacial Surgery 2nd Ed 2004 page 169 Tuberosity Reduction: Excesses in the maxillary tuberosity may consist of soft tissue, bone, or both. Sounding, which is performed with a needle, can differentiate between the causes with a local anesthetic needle or by panoramic radiograph. Bony irregularities may be identified, and variations in anatomy as well as the level of the maxillary sinuses can be ascertained. Excesses in the area of the maxillary tuberosity may encroach on the interarch space and decrease the overall freeway space needed for proper prosthetic function. Access to the tuberosity area can be obtained easily using a crestal incision beginning in the area of the posterior tuberosity and progressing forward to the edge of the defect using a no. 12 scalpel blade. Periosteal dissection then ensues exposing the underlying bony anatomy. Excesses in bony anatomy are removed using a side-cutting rongeur. 1016. for recording of vertical dimention we use a. Willis Gauge. *** b. caliper. c.Face bow

Oxford Handbook of Clinical Dentistry - 4th Ed. (2005) page 194 Resting face height is assessed using:

A Willis gauge, to measure the distance between the base of nose and the underside of the chin. Is only accurate to 1 mm. Spring dividers, to measure the distance between a dot placed on both the chin and the tip of the patient's nose. This method is less popular with patients and is C/I for bearded gentlemen (or ladies!). The patient's appearance and speech. Questions and Answers for Dental Nurses Willis gauge is used to record the occlusal face height of the the patient. Dental Decks - page 420 A face-bow is a caliper-like device used to record the patient's maxilla/hinge axis relationship (opening and closing axis). It is also used to transfer this relationship to the articulator during the mounting of the maxillary cast. 1017. what name of bur used in proximal surface of laminate veneer??? Radial dimound. *** fissure http://www.brasselerusa.com/documents/Nixon_Porcelain%20Veneer%20II.p df The facial depth cuts are removed with the 850-014 diamond bur, and the long axis of the diamond bur is rolled into the proximal chamfer area to eliminate any sharp line angles 1018. what name of bur use in facial surface of veneer??? Dimond. *** fissure http://www.brasselerusa.com/products/display.cfm?zoom=diamonds&id=38 Round-end diamonds create chamfer margin and facial reduction for direct and indirect veneer restorations 1019. pacifier habit what you see in his mouth? a. Open bite. b. Cross bite Dentistry for child and adolescent Children who were pacifier users were significantly more likely to show open bite, posterior crossbite increased overjet, and alteration in cheek mobility than habit-free children action during sucking, which leads to gingival injury recession, and loss of alveolar bone

1020. In the pulp 1 cell rich zone inner most pulp layer wich contain fibroblast 2 cell free zone rich with capillaries and nerve networks 3 odonotbalstic layer wich contain odontoblast Dental Decks - page 559 1021. What type pontic design would you in a patient with a high esthetic demand when preparing teeth number 9 11 for a F P D : a- ridge lap or saddle pontic b- An ovate pontic c- modified ridge lap pontic. *** Dental Decks - page 483 1022. design of anterior pontic a-modified ridge lab*** b-saddle c-hygienic 1023. Skeletal Bone of skull develop from : a- Neurocranium ossification b- Intramembranous ossification c- Endochondral ossification. Dental decks 287 Endochondral ossification : Short bone and long bone. Ethmoid, sphenoid and temporal bone. intramembranous ossification: Flat bone. 1024. Skeletal face is from: a. Neural crest*** b. Paraxial mesoderm c. lateral plate (somatic layer) mesoderm. Neural crest give rise to : frontal b. sphen nasal - lacrimal zygomatic maxilla incisive mandible _SqTemporal.paraximal mesoderm (somitessomitomeres) gives rise to occipitals - pet temporal- -paraietal bon Sadler's Langman's Medical Embryology 1025. Glenoid fossa is found in: 1/ orbital cavity 2/nasal cavity 3/ middle cranial fossa

D) temporal bone. *** The glenoid fossa = the mandibular fossa. The mandibular fossa: a depression in the temporal Bone that articulates with the condyle of the Mandible and is divided into two parts by a slit. And Anatomy of the Human Body - Henry Gray page 82 1026. The spread of odontogenic infection is based on: 1/ host defense 2/ virulent of microorganism 3/ No. Of bacteria 4/ all. *** 1027. 3rd generation of apexo locator: a. Use with all pt b. Need more research c. increase chair time d. Decrease radiographic film need. *** 1028. pt taken heparins he should do surgery after : 1/ 1 Hr 2/ 2 Hr 3/ 4 Hr 4/ 6 Hr. *** 1029. twins came to your clinic during routine examination ,you found great change behavior between both of them this due to A)hereditary B)environment. *** C)maturation. Dentistry for child and adolescent Human twins are of two basic types: monozygotic (or identical) twins resulting from a single ovum fertilized by a single sperm, and dizygotic (or fraternal) twins resulting from fertilization of two ova by two sperm. It also follows that difference between monozygotic twins result from environmental differences whereas those between dizygotic twins result from differences in both heredity and environment And http://social.jrank.org/pages/666/Twin-Studies.html 1030. Epiliptec patient you will not give him : A) aspirin B)azoles C)metronidazole. ***

D) all of the above 1031. Dylantin (phynotoin) dont give with : A) aspirin B)azoles C)metronidazole. *** D) all of the above 1032. Pregnant 25 years, bleeding on probing, location on papilla of anterior area of the maxilla, Isolated: A)giant cell granuloma B) pyogenic granuloma (pregnancy epulis). *** C) giant cell granuloma 1033. Porcelain, highly esthetic, anterior maxilla area, we choose: A) Dicor B) in ceram. *** C)impress 1034. pt has discoloration on his max ant central, and u planning to do veneer for him . which type of porcelain has high easthetic ? a. in ceram *** b. dicor glass reinforced risen c. impress Fundamentals of fixed prosthodontics page 436 In-Ceram has been used to fabricate fixed partial dentures, but the manufacturer recommends only short-span (three-unit) anterior restorations. Alumina-reinforced ceramic systems (In-Ceram) significantly improve the light reflection characteristics of crowns when compared to conventional metalceramic restorations. However, opaque aluminum oxide diminishes translucency when compared to leucite-reinforced systems (Optec, IPS-Empress). To improve light transmission and reflection in single anterior crowns where maximum strength is not required, a magnesium aluminous spinel may be utilized. The transilluminating qualities seem to be similar to those of natural teeth. 1035. the highest strength in porcelain: A) ZR (zircon) reinforced in ceram. *** 1036. Amalgam pain after restoration due to:

A) phase 2 gamma B) phase 1 gamma C) zinc containing alloy. *** D) Admix alloy 1037. Zinc if added to amalgam a. Increase moisture sensitivity and cause expansion b. Increase marginal integrity and longevity than zinc free amalgam c. A+ b. Dental Decks - page 2312 . 1038. endocrine and exocrine gland is : A) pancreas. *** B) pituitary gland C) thyroid g D) salivary g E) sweat g Wikipedia: The pancreas is a gland organ in the digestive and endocrine system of vertebrates. It is both an endocrine gland producing several important hormones, including insulin, glucagon, andsomatostatin, as well as an exocrine gland, secreting pancreatic juice containing digestiveenzymes that pass to the small intestine. These enzymes help to further breakdown thecarbohydrates, protein, and fat in the chyme. 1039. saline coupling agent .. wetting wall of pulp 1- decrease wall tension*** 2- increase wall tension : 1040. in endo, one of sealer property is to be flowable (or wetability) ,,to enhance this quality we can mix it with a material that have: a. low surface tension b. high surface tension 1041. silane coupling agent: 1/ used with porcelain to enhance wetability of bonding. *** 2/ used with tooth and porcelain. Oxford Handbook of Clinical Dentistry - 4th Ed. (2005) page 146

Remove inlay and clean with alcohol. For porcelain only, place layer of silane coupling agent on fitting surface. Etch enamel and dentine (total etch concept). Wash and remove excess moisture, but do not dry. Dental Decks - page 2102 Silane acts as an adhesive between the inert filler and the organic matrix 1042. for discharged sharp instrument (blades, needle tips, wedges, etc) put in : A) dicharged paper basket B) designed sharp instrument special container. *** C) disinfectant in auto glave then throw D) put it in multifoil 1043. Female patient came to your clinic with continous severe pain related to 1st maxillary molar. After examination dentist diagnose the tooth is carious and has irreversible pulpitis. He decides to do RCT. After enough time for anaesthisation, the patient wont allow the dentist to touch the tooth due to severe pain. Dentist should: A) give another appointment to the patient with description of antibiotics.. B) Extraction. C)Intra-pulpal anaesthia. 1044. 32 years old patient came to your dental office, suffering from a bad odour and taste from His mouth. By examination patient has an anterior mandibular 3 unit bridge that bubbles upon Applying water spray and slight pressure. Cause: A) broken abutment. B) Food impaction underneath the pontic. C) separation between the abutment and the retainer.. *** (dissolving of cement / microleakage) 1045. Patient complaining from pain in the floor of the mouth (beneath the lower jaw) your diagnosis is related to the salivary glands, whats the best x-ray to help you: A)panoramic B)occlusal C)sialograph. *** (Specialized radiograph for the Salivary gland disorders) Dental secrets page 107 Because the salivary glands consist of soft tissue, they cannot be seen on radiographs unless special steps are taken to make them visible. In a technique called sialography. 1046. distal surface for first upper premolar ,contact with the neighbor teeth

A)in the middle with buccal vastness wider than lingual one B)in the middle with lingual vastness wider than bucccaly one. *** 1047. the movement of polymorphic cells in the gaps of intracellular to the blood capillary Outside it called: A)porosity B)slinking C) diapedesis. *** ( source Wikipedia) 1048. Child with cleft palate and cleft lip with anodontia due to: a- Van der woude syndrome. *** b- Treacher Collins syndrome. c- Paget disease. http://en.wikipedia.org/wiki/Van_der_Woude_syndrome Van Der Woude syndrome (VDWS) is a genetic disorder. People with VDWS have cleft lipwith or without cleft palate, isolated cleft palate, pits or mucous cysts on the lower lip, andhypodontia. Affected individuals have normal intelligence 1049. Which of the following canals in # 14 is most difficult to locat: a- palatal b- Distobuccal c- Mesiobuccal. *** d- All of above 6 6 : . 5 1050. Which condition is an apical lesion that develop acute exacerbation of chronic apical abscess: a- Granuloma b- Phoenix abscess. *** c- Cyst d- Non of above Dental decks page 165 1051. Acute exacerbation of chronic pulpitis: a) Reversible pulpitis. B) Irreversible pulpitis.*** C) Acute periodontitis. . 1052. Which tooth require special attention when preparing the occlusal aspect for restoration:

a- lower 2ed molar b- lower 1st premolar. *** c- lower 2ed premolar d- upper 1st molar Dental Decks - page 2295 1053. Pt came 2 u with coloration bluish (or green?? ) and black in the gingival margins .he said that hi has gasteriointensinal problem.this is caused because of : a-mercury b-lead c-bismuth. *** d-arsen. . 1054. How can u repair fractured rest(in the place where it passes over the marginal ridge of the tooth ) in removable partial denture? A-spot welding b-electric soldering c-industrial brazing d-....... 1055. Hunter Schreger bands are white and dark lines that appear in: 1. Enamel when view in horizontal ground. 2. Enamel when view in longitudinal ground. *** 3. Dentin when view in horizontal ground. 4. Dentin when view in longitudinal ground. http://www.wrongdiagnosis.com/medical/hunter_schreger_bands.htm Hunter-Schreger bands: alternating light and dark lines seen in enamel of the tooth that begin at the dentoenamel junction and end before they reach the enamel surface; they may represent areas of enamel rods cut in cross-sections dispersed between areas of rods cut longitudinally. 1056. Sealer is used in RCT to: 1- Fill in voids. *** 2- Increase strength of RC filling. 3- Disinfect the canal. 1057. Child patient presented with swelling in the buccal and palatal maxillary anterior area tow days ago, the pathology of the lesion there is a giant cell, what is the diagnosis: 1. Giant granuloma. 2. Hemangioma. 3. . http://www.turkishjournalpediatrics.org/?fullTextId=227&lang=eng

Central giant cell granuloma (CGCG) is a benign intraosseous lesion of the jaws that is found predominantly in children and young adults. Although benign, it may be locally aggressive, causing extensive bone destruction, tooth displacement and root resorption. 1058. Child with anodontia and loss of body hair, the diagnosis is: 1. Down's syndrome. 2. Ectodermal dysplasia. *** 3. Fructose .. 4. Diabetic . 1059. Cavity etching before applying GIC is: 1. Polyacrylic acid 10 seconds. *** 2. Polyacrylic acid 60 seconds. 3. Phosphoric acid 10 seconds. 4. Phosphoric acid 60 seconds. 1060. to increse retention of GIC u shold use A-37%POLYACRYLIC ACID FOR 15 SECOND B-35%polyacrylic acid for 10second c-10%polyacrylic acid for 10 second d-10%polyacrylic acid for 15 second a,b a,d c,d c only d only 1061. surgery for ridges aim to: 1. Vertical dimension. 2. Speech. 3. Modify ridge for stability. *** 1062. Patient with warfarin treatment and you want to do surgery, when you can do: 1. When PTT is 1 1.5 INR on the same day. 2. When PTT is 2 2.5 INR on the same day. 3. When PT is 1 1.5 INR on the same day. *** 4. When PT is 2 2.5 INR on the same day. Dental secrets page 38 Warfarin affects clotting factors II, VII, IX, and X by impairing the conversion of vitamin K to its active form. The normal PT for a healthy patient is 10.013.5 seconds with a control of 12 seconds. Oral procedures with a risk of bleeding

should not be attempted if the PT is greater than 1 times the control or above 18 seconds with a control of 12 seconds. 1063. Patient with pain on the upper right area, and the patient can not tell the tooth causes the pain, what is the least reliable way to do test pulp: 1. Cold test. 2. Hot test. 3. Electric test. *** 4. Stimulation the dentine. 1064. Pt have denture, after 5 year he complain of ulcer and inflammation in lower buccal vestibule. wt is the Dx: 1/hypertrophic Frenum. *** 2/ epulis fissurment. 1065. Patient presented to you after fitting the immediate denture 5 10 months, complaining pain and over tissue in the mandibular, what is the diagnosis: 1. Epulis fissurment. *** 2. Hypertrophic Frenum. http://en.wikipedia.org/wiki/Epulis_fissuratum
Epulis fissuratum (also known as "Granuloma fissuratum"[1]:808) is an oral pathologic condition that appears in the mouth as an overgrowth of fibrous connective tissue. Also referred to less commonly as inflammatory fibrous hyperplasia, denture epulis, and denture induced fibrous hyperplasia, it is associated with the edges of a denture that does not fit well. The word, "epulis", can be used to describe any gingival tumor, but it is widely used in association with this specific condition.

1066. Main reason for surgical pocket therapy: a. Expose the roots for scaling and root planning b. Remove supragingival calculus c. 1067. Biological width a. 1 mm b. 2mm *** c. 3mm d. 4mm .) ( ) 5.37 = 4.3: + 3.<:(

3.9< ( ) . 3.8 ( )

1068. Biological depth: d. Crestal bone to gingival sulcus *** 1069. Periodontal attachment contain: Epithilum, sulcus, connective tissue. *** a. b. c. d. e. 1070. Periodontally involved root surface must be root planed to: Remove the attached plaque and calculus. Remove the necrotic cementum. Change the root surface to become biocompatible All of the above. A & b only. ***

http://www.asnanak.net/ar/article.php?sid=152 : Periodontal debridement 1071. Best measurement of periodontitis by: a. Pocket depth. b. Bleeding. c. Attachment level.***

Abstract
Clinical measurement of periodontitis has historically focused on the concept of the static condition of periodontal pockets. Observations based on longitudinal periodontitis as a slow, continuous process which has emphasized measurements of measurement of attachment loss in untreated subjects have indicated that periodontal destruction occurs in discrete episodes of short duration. Based on these studies, it has been suggested that chronic periodontal disease proceeds through a series of random episodic attacks. Periodontal sites are considered as existing in 2 states, their probable depth, whereas during the inactive state, no significant change in probing depth can be detected. either disease active or inactive. During periods of disease activity, sites increase in

Oxford - 120 Probing to elicit bleeding (which is the single most useful indicator of disease activity), measuring pocket depth attachment levels, and detecting subgingival calculus.

1072. The tissue response to oral hygiene instruction is detected by: a- Probe pocket depth. b- Less bleeding on propping . *** Oxford 120 Both the MBI and PlI can be expressed as bleeding or plaque-free scores in this way obtaining ahigh score is a good thing, which may be both easier for the patient to understand and a more positive motivational approach. 1073. After scaling and root planning healing occur by: a. Long junctional epithelium. *** b. New attachment. c. New bone and connective tissue formation. d. New attached periodontal ligament fibers.

Dental decks page 266

1074. During examination 34 show gingival recession buccally, the least correct reason is: a. Frenum attachment. b. Pt is right hand brushee. c. Occlusal force. *** d. Inadequate gingival. . . : 1075. Periodontal pocket differ most significantly from gingival pocket with respect to: a.Depth. b.Tendency to bleed on gentle probing. c.The location of the bone of the pocket. *** d.All of the above. Oxford 118 Chronic gingivitis is, as the name suggests, inflammation of the gingival tissues. It is not associated with alveolar bone resorption or apical migration

of the junctional epithelium. Pockets > 2 mm can occur in chronic gingivitis due to an increase in gingival size because of oedema or hyperplasia (false pockets). 1076. All of these are right ways to handle the instrument EXCEPT A- Modified pen handle b- Inverted pen c- Pen handle. *** d- Palm and thumb http://www.slideshare.net/confirm/MjY0NjU5OTE7ZC5zbW8=/3349629b1688b1e7e816bc6cf48bdb24824b43b6184735e-slideshow There are four grasps used with the hand instruments: Modified pen. Inverted pen. Palm and thumb. Modified palm and thumb. 1077. Amalgam is used in extensive cavities : a- When the cusp is supported by dentine and proper retentive preparation b- When Cusps lost and thin supported wall. *** c- When one cusp is lost and need to apply restoration to replace it 1078. what is the most factor encouraging dental caries : A) Xerostomia. *** B) Hypocalcification. C) Smoking. 1079. Incipient caries in the old patients is MOSTLY due to: a)smoking b)saliva d)Xerostomia. *** 1080. the best definition to odontoblast: a- It s subjacent to predentine, odontoblastic process *** b- Odontoblast cell is more in the cellular pulp than radicular chttp://en.wikipedia.org/wiki/Pulp_(tooth) Odontoblastic layer; outermost layer which contains odontoblasts and lies next to the predentin and mature dentin 1081. The last sensation whih disappear after local anesthisea A-pain b-deep pressure. *** c-temperature http://www.scribd.com/doc/17106080/Local-Anesthetics Both sensory & motor nerves are equally sensitive. Order of pain blockade is pain, temperature, touch, deep pressure sense.

Applied to tongue bitter taste is lost first, followed by sweet & sour, and salty taste is lost last of all. 1082. The kind of on lay wax used in cast 1. braffin 2>>>>> 3>>>>> 1083. wax inlay which type contain in much gradient? a) baraffin wax b) bee wax 1084. non odontogenic Lesion similar to Endo Lesion: a-Hyperparathyroidism b-initial stage of cemental dysplasia. *** c-ossifying Fibroma d-Dentigeaus cyst e-ameLobLastoma f-Lateral periodontal cyst j-myxoma & Hemangieoma White and Pharoah, 4th edition, page 451 In the early radiolucent stage periapical cemental dysplasia PCD lesions,the most important differential diagnosis is the periapical rarefying osteitis. 1085. While rct if u penetrate the furcation area of roots what u will do? a. Mineral Trioxide Aggregate (MTA). *** b. ca oh c. formocresol 1086. the mineral Trioxide aggregate is best material for a. indirect pulp capping b. apexogenesis c. apexofication d. root canal obturation e. all except a mta can used with direct pulp capping )http://en.wikipedia.org/wiki/Mineral_trioxide_aggregate( http://webcache.googleusercontent.com/search?q=cache:r5DcCNJhBZIJ:www .drpulp.com/2011/05/use-mta-in-your-endodonticcases.html+best+use+of+mta&cd=7&hl=ar&ct=clnk&gl=sa&client=firefox -a Direct Pulp Capping / Apexification / Perfuration / MTA Internal&External Resportion / Root End Filling http://books.google.com/books?id=zMa...regate&f=false

1087. The fundamental rule in the endodontic emergencies is : a. control pain by inflammatory non steroid. b. diagnosis is certain. *** 1088. Female pt come with endo treated upper central with m, d caries & have incisal abrasion. Porcelain veneer is planned with modification to cover incisal edge. veneer should end: a. fourth lingualy 0.5 mm before centric occlusal. *** b. fourth 1.5 before centric occlusion c. fifth 1.5 before centric occlusion 1089. Pt come with siuns u make gp tracing & take radiograph the gp appear in lateral surface of the root a. periodontal abscess b. periodontitis c. lateral acessory canal. 1090. Tech of endo fill where we use continuous condensation a.vertical condensation*** Principles and Practice of Endodontics WALTON page 273 "Continuous wave of condensation" in the vertical condensation paragraph 1091. Post graduated student use mta the prognosis depend on prevent a. immediate suture b. disturbance during closure of wound. *** c. using a flab 1092. The cause of black cast which prevent pickling due to a. over heat b. contaminate with gas c. incomplete casting Dark discolored casting due to Over heating of the investment lead to decomposition of calcium sulphate hemihydrates binder and release of sulpher which will combine with copper and silver of gold alloy lead to compounds which resist pickling of .casting 1093. Sharpening the curette and sickle, the cutting edge should be at angle: A- 50-60 B- 70-80. *** C- 80-90 D- 60-70 1094. Pt take 40 cortisone in day of procedure a.double the dose just day of procedure b. double the dose day of procedure & day after c. stop the medication 1095. What is the dominant type of fibers found in Cementum:

A) longitudinal B) Circular C) Sharpey's fiber.. *** 1096. Fibers which completely embedded in cementation and pass from cementation of one tooth to the cementation of adjacent tooth is: 1. Sharpey's fiber. 2. Transceptal fibers. *** 3. Longitudinal fibers. 1097. What is the main function of impression tray holes : A)Fixing the Impression material. *** 1098. A Tailor is presented to your dental office, whats the most common feature to be found in His teeth upon examination : A)Attrition B) abrasion. *** C) Erosion D) Abfarcation 1099. Abrasion of enamel and root surfaces may result from the long term use of: a. A hard toothbrush. Tooth abrasive toothpaste or powder. Vigorous use of the toothbrush. A and B only. A, B and C. *** 1100. Patient came to your clinic complaining of pain, upon examination you cant find a cause. Whats the next logical step to do in investigation A) Panoramic x-ray. *** B) CT Scan C) MRI D) Regular tomography 1101. contra indication of implant EXCEPT 1_many dental caries. *** 2_malignancy 3_radiation therapy 1102. dental implant are successfully with min failure: a-premaxilla area in the upper arch. b-posterior area of the maxillary arch. c-mandible between the mental foramen. d-buccal shelf of the mandible.

b. c. d. e.

dental secrets ? .Which anatomic site is the most likely to yield failed implants Implants placed in the maxillary anterior region are the most likely to fail . Because short implants are more likely to fail than longer implants, the longest implant that is compatible with the supporting bone and adjacent anatomy should be used. http://www.syrianclinic.com/vb/threa...A7%D9%85%D9%84 : . . . . : : . : . . . . CT ( : 8

) ( 1103. Whats the best implant type allowing Osseointegration: A) Root-form Endosseous implant. *** 1104. The indications of implantation: 1. Diabetic patient. 2. Loss of one tooth only with the adjecent teeth. *** 1105. what medical condition should prevent the dentist from practicing dentistry : A) Diabetes B) Hypertension C) Influenza. *** D) Headache. 1106. patient complaining of Xerostomia & frequent going to the toilet at night A) Diabetes Mellitus. *** 1107. which of the following materials is NOT a hemostatic agent : A) Oxidized cellulose B) Gelvon C) Zinc Oxide. *** 1108. patient suffering from a submandibular gland abscess, dentist made a stab incision and is fixing a rubber drain to evacuate the pus, the drain is sutured to : A) Intra-oral B) From angle of the mandible. C) Between myloid muscle and.. 1109. patient suffering from a submandibular gland abscess, dentist made a stab incision and is fixing a rubber drain to evacuate the pus, the drain is sutured to: A) Intra-oral between the myeloid muscel B) extraorally from the most fluctant point C) extraorally under the chin d) .. the chin 1110. The best material for taking full crown veneers impression is : A) Poly-sulphide B) Poly-ether C) Irreversible hydrocolloid D) Poly vinyl siloxane (Additional silicone). ***

Dental Decks 1111. what is the concept of Pro-taper system : A) Step down tech. B) Step back tech. C) Crown down tech.. *** 1112. Preparation of tooth for metal ceramic restoration should be done in: A) two planes. *** B) parallel to long axis 1113. Labial reduction for porcelain metal restoration must be: a. One plane for aesthetic. b. Tow planes by follow the monophology. *** 1114. preparation for labial surface in one plane in the preparation for metal crown is: A)more retentive B) less retentive. *** c) less cutting of tissues 1115. when removing lower second molar : A) occlusal plane perpendicular To the floor B) buccolingual direction to dilate socket. C) mesial then lingual 1116. Upon opening an incision in a periapical abscess in a lower 1st molar, you open : A) The most bottom of the abscess. *** B) The most necrotic part of the abscess. C) Extra oral 1117. Whats the test used for HIV: Elisa. *** 1118. Neonate 2 years old, has a lesion on the centrum of the tongue... With the eruption of the 1st tooth: A)Riga-Fede disease. *** <sublingual traumatic ulceration> Mosby Medical Dictionary Riga-Fede disease: a tumor of the tongue (lingual frenum) in some infants. It is caused by early teeth rubbing on it. Also called *Fede's disease. 1119. Which of the following conditions is highly indicated for the short therapy of DOTS and Is directly observed once in the clinic: A) Tuberculosis. *** B) HIV C) H1N1 D) Mental Illness

1120. At the begining of the Operation day in the clinic, you should start the water/air spray for Three minutes in order to get rid of which type of microorganisms : A) streptococcus mutans. B) streptococcus salivaris. *** C) .... D) .... Dental high-speed turbines and handpieces can take up and expel microorganisms during operation and thus need regular sterilization. This study established a method for validating devices used to sterilize high-speed turbines and handpieces. The air and water channels and turbine chambers were contaminated with suspensions of Streptococcus salivarius or endospores of Bacillus stearothermophilus. The effect of flushing and/or autoclaving performed by a new device combining both procedures was evaluated by counting the number of viable bacteria recovered from these devices. Further, the effect on clinically used handpieces was evaluated. In an initial experiment, the device partially reduced S. salivarius, and the endospores survived. In a second experiment 1121. The main link between the pulp and the the periodontium is: A. Apical foramen. *** B. Dentinal tubules C. Accessory canals D. PDL 1122. Patient came with severe pain related to right 1st mandibular molar, there's no swelling related, pulp test is negative, no evidence in radiograph. Diagnosis: A. Irreversible pulpitis B. Acute periodontal abscess. C. Suppurative periodontal abscess 1123. Which of the following teeth has a contact area between the incisal (occlusal) third and middle third: A. 1st maxillary premolar. B. 1st mandibular premolar. *** C. 1st maxillary molar. D. Central mandible Incisor. Textbook of Dental and Oral Anatomy Physiology and Occlusion page 110 1124. A pt came to your clinic after examination you found deformity in the neck and collarbones and supernumerary of teeth what is the diagnosis :

a. cleidocranial dysostosis*** b. amelogensis imperfecta Partly or completely missing collarbones. If the collarbones are completely missing or reduced to small vestiges, this allows hypermobility of the shoulders including ability to touch the shoulders together in front of the chest. A soft spot or larger soft area in the top of the head where the fontanelle failed to close. Bones and joints are underdeveloped. People are shorter and their frames are smaller than their siblings who do not have the condition. The permanent teeth include supernumerary teeth. Unless these supernumeraries are reabsorbed before adolescence, they will crowd the adult teeth in what already may be an underdeveloped jaw. In that case, the supernumeraries will probably need to be removed to provide space for the adult teeth. Permanent teeth not erupting Bossing (bulging) of the forehead. Hypertelorism 1125. Pt with complete denture complain from tightness of denture in morning then become good this due to A) relif of denture. *** (because there may be pressure points or areas that the tissues will try to Adjust to it throughout the day) B)lack of cheeck elastisty (pressure on the flanges <> displacement of denture) C)poor post dam (no posterior seal <> displacement of denture) 1126. old.pt.come with set of compelete denture with tight denture in morning and become loose later in aday what is the cause: a. lack of posterior palatal seal b. deflecting of occlusion c. excessive relining of denture*** d. inelasticity of cheek 1127. A border line diabetic pt came with denture stomatitis you find abundant debris in the tissue surface area of the denture>>the proper management is: A. Systemic antibiotic B. Topical antifungal. *** (topical + relining with a tissue conditioner + rest of tissues at night +Good oral hygiene) C. Systemic antifungal

D. Topical antibiotic 1128. Pain in central incisors from A. Central &lateral incisors. *** B. Lateral & canine C. Canine & premolar D. Premolar & molar 1129. To treat non vital tooth with open apex when doing access openning with gates glidden bur take care to : A. Remove all dentin B. Remove minimal dentine. *** C. Follow conservative method 1130. To treat non vital tooth with open apex when doing access opening with Gates Glidden drills take care to avoid a. Remove all dentin. (Considerable dentin(*** b. Remove minimal dentine. c. Follow conservative method. 1131. 20 years old pt have avulsed tooth for 60 min the management to return vascularity of the tooth: A. Scrap the surface of the root B. Place the tooth in sodium sulfide of X%.... C. Place it in sodium chloride then sodium sulfide. *** Oxford Handbook of Clinical Dentistry - 4th Ed. (2005) page 73: Avoid handling root surface. If tooth contaminated, hold crown and agitate gently in saline. http://emedicine.medscape.com/article/763291-treatment f extraoral time is longer 60 minutes, soak the tooth in citric acid and fluoride to make the root as resistant to resorption as possible. Consult a dentist. 1132. The peripheries of the custom tray should be under extended to all border and clearance from the frenum areas: 2mm. *** 4mm. 6mm. 8mm. 1133. The goal of making the peripheries of the custom tray under extended to all bordered clearance from the frenum areas: To give enough space for the used impression materials to allow border molding the tray. ***

1. 2. 3. 4.

1.

2. To give enough space for the die spacer. 3. To give enough space for the cementation materials. 4. None. 1134. The base plate could bee made by: 1. Acrylic plate. 2. Ceramic plate. 3. Wax plate. 4. A and c. *** 1135. The vertical height of the maxillary occlusion rim from the reflection of the cast is: 1. 12mm. 2. 22mm. *** 3. 32mm. 4. 42mm. 1136. The anterior width of the maxillary occlusion rim is: 1. 5mm. *** 2. 10mm. 3. 15mm. 4. 20mm. 1137. The posterior width of the maxillary occlusion: 1. 8-10mm. *** 2. 8-15mm. 3. 10-15mm. 4. 15-20mm. 1138. The anterior height of the mandibular occlusion rim is: 1. 6mm. 2. 16mm. *** 3. 26mm. 4. 36mm. 1139. The posterior height of mandibular occlusion rim is: 1. Equal to the point representing 1/2 of the height of retro molar pad. *** 2. Equal to the point representing 1/2 of the height of the frenum areas. 3. Equal to the point representing 1/2 of the height of the alveolar ridge. 4. None. 1140. To record the occlusal plane in order to: a. To determine the amount of space between the mandible and the maxilla which will be occupied by an artificial teeth b. To determine vertical and horizontal level of the teeth. c. A and B. ***

d. None. 1141. The protrusive condylar guidance should be set on the articulator at: a. 30 35 degree. b. 50 degree. c. 60 degree. d. 70 degree. https://docs.google.com/viewer?a=v&q=cache:eXYttsI1HVoJ:whipmix.c om/wp-content/uploads/via-productcatalog/product_docs/wide_vue_inst.pdf+protrusive+condylar+guidan ce+should+be+set+on+the+articulator+at&hl=ar&gl=sa&pid=bl&srcid= ADGEESjtKSZdtLbEXpv7Nt8GDGUVFw6eaMBVUjRRp2cb2bvhjwFxJXhD8 gRgLkadLf4z4BaN8i83o8kQ0ZyyOCImZtUejyZuMmRnKuip8PpaIDWuTo WRpSiSQNOQP0LIgQCTTCU7i5y&sig=AHIEtbTka1UBavyxyEMcPl54iDASyrxE 44 dQ 1142. The lateral condylar posts should be set on the articulator at: a. Zero degree. b. 20 degree. c. 40 degree. d. None. https://docs.google.com/viewer?a=v&q=cache:berkhiAZfDEJ:whipmix.com /wp-content/uploads/via-productcatalog/product_docs/Hanau%2520H2%2520InstLR.pdf+lateral+condylar+ posts+should+be+set+on+the+articulator+at&hl=ar&gl=sa&pid=bl&srcid=A DGEESjst8-pYg8C9WS4ScL2O2ghN677vdbMw7jqbKyHPmK0meWA0Z77u15M4Grb9XHx92rtwl3oFe_Qc_a7wuDUJhoWc0U 39fxeAQmfw-3OugT4sqFtlz2qLfMf5Sx6EuckrpsH4u&sig=AHIEtbRU3CXKzLGQgNAzjUlNu7lj 8 y-USxw 4; 1143. The incisal guide should be set on the articulator at: a. Zero degree. *** b. 20 degree. c. 40 degree. d. None. .Zero degree https://docs.google.com/viewer?a=v&q=cache:eXYttsI1HVoJ:whipmix.com /wp-content/uploads/via-productcatalog/product_docs/wide_vue_inst.pdf+protrusive+condylar+guidance+

should+be+set+on+the+articulator+at&hl=ar&gl=sa&pid=bl&srcid=ADGEES jtKSZdtLbEXpv7Nt8GDGUVFw6eaMBVUjRRp2cb2bvhjwFxJXhD8gRgLkadLf4 z4BaN8i83o8kQ0ZyyOCImZtUejyZuMmRnKuip8PpaIDWuToWRpSiSQNOQP0LIgQCTTCU7i5y&sig=AHIEtbTka1UBavyxyEMcPl54iDASyrxEdQ 44 1144. The primary goal of anterior tooth selection is: a. To provide good functional requirements. b. To satisfy aesthetic requirements. *** c. To let the patient feel comfortable. d. None. 1145. The primary goal of posterior tooth selection is: a. To provide good functional requirements. *** b. To satisfy aesthetic requirements. c. To satisfy sychological requirements. d. None. 1146. You need.to get the teeth shade: a. Shade guide. *** b. Incisal guide. c. Acrylic teeth. d. Porcelain teeth. 1147. The teeth materials are: a. Acrylic teeth. Porcelain teeth. c. A and B. *** None. 1148. The width of the lower teeth is: a. 1/2 of the maxillary anterior teeth in normal jaw relationship. 1/3 of the maxillary anterior teeth in normal jaw relationship. 3/4 of the maxillary anterior teeth in normal jaw relationship. *** None. 1149. Generally posterior teeth are classified into: a. Anatomy (cusp) teeth. Non-anatomy (cuspless) teeth or flat. A and B. *** None. 1150. The process of positioning or arranging teeth on the denture base is termed: a. Casting. Investing.

b. d.

b. c. d.

b. c. d.

b.

c. Setting up. *** d. Flasking. 1151. Important functions must be considered when arranging anterior teeth: a. Aesthetics. b. Incision. c. Phonetics. d. All. 1152. Which surface of the central incisor that contacts the median line: a. Distal. b. Mesial. *** c. Buccal. d. Lingual. 1153. The incisal edge of the maxillary lateral incisor is..above and parallel to the occlusal plane: 1. 1/2 mm. *** 2. 1 mm. 3. 2 mm. 4. 3 mm. 1154. The long axis of the maxillary cuspid is inclined slightly to the: 1. Mesial. 2. Distal. *** 3. Buccal. 4. Lingual. 1155. It is called .. When the occlusal surfaces of the right and left posterior teeth are on the same level: 1. Vertical plane. 2. Horizontal plane. *** 3. Compensating curve. 4. All. 1156. The . Of the maxillary first bicuspid is raised approximately 1/2mm of the ocllusal plane: 1. Buccal cusp. 2. Lingual cusp. *** 3. Mesial surface. 4. All. 1157. The long axis of the maxillary first molar is inclined to 1. Buccal. 2. Mesial. 3. Distal.

4. Lingual. *** 1158. All maxillary posterior teeth touch the occlusal plane EXCEPT: 1. First bicuspid. 2. Second bicuspid. 3. First molar. 4. Second molar. *** 1159. The distance between the lingual surfaces of the maxillary anterior teeth and the labial surfaces of the mandibular anterior teeth is: 1. Vertical overlap (overbite). 2. Horizontal overlap (overjet). *** 3. Occlusal plane. 4. All. 1160. The distance between the incisal edges of the maxillary and mandibular anterior teeth is: Horizontal overlap (overjet). Vertical overlap (overbite). *** Occlusal plane. All. 1161. The average distance between the lingual surface of the maxillary anterior teeth and the buccal surface of the mandibular anterior teeth (overjet) is: 1/2mm. 1mm. 2mm. 3mm.*** Overjet: distance between upper and lower incisors in the horizental plane. normal is 2-4 mm. An Inroduction to Orthodontics by Laura Mitchel 1162. Which tooth of the mandibular anterior teeth that touch the lingual surface of the maxillary anterior teeth in normal centric relation? Central incisor. Lateral incisor. Cuspid (Canine). *** None. 1163. The mesial surface of the mandibular lateral incisor contacts: The mesial surface of the central incisor. The distal surface of the central incisor. ***

1. 2. 3. 4.

1. 2. 3. 4.

1. 2. 3. 4. 1. 2.

3. The mesial surface of the cuspid. 4. The distal surface of the cuspid.

1. 2. 3. 4.

1. 2. 3. 4.

1. 2. 3. 4.

1. 2. 3. 4.

1. 2. 3. 4. 1. 2. 3.

1164. The tip of cusp of the mandibular cuspid is one above the occlusal plane to establish . Of the maxillary anterior: Horizontal overlap. Occlusal plane. *** Vertical overlap. All. 1165. The relation involves the movement of the mandibular to the side either right or left in which the act of mastication is to be accomplished. Therefore the side to which the mandible moves is called: Balancing side. Working side. *** Compensating side. All. 1166. When the mandible moves to the working side, the opposite side cusp to cusp contacts in order to balance stresses of mastication. This relation is called: Working relation. Balancing relation. *** Occlusal relation. None. 1167. In order to distribute the primary forces of mastication, to fall within the base of the denture, the mandibular teeth are set: On the bucal edge of the ridge. On the lingual edge of the ridge. On the crest of the ridge. *** All. 1168. The mandibular posterior tooth that has no contact with any maxillary teeth during the balancing occlusion is: First bicuspid. *** Second bicuspid. First molar. Second molar. 1169. The used device in flasking procedure is called: Articulator. Separating medium. Flask. ***

4. None. 1170. We Vaseline the inner surface of the flasks all rounds: 1. To help in the packing procedure. 2. To separate the models (casts) safety. *** 3. A and B. 4. None. 1171. The procedure that follows the flasking procedure is called: 1. Polishing. 2. Deflasking. 3. Packing. *** 4. Curing the acrylic. 1172. Teeth selection in setting up teeth gsf is based of these factors: 1. Shade of the teeth. 2. Size and shape of the teeth. 3. Angle of the teeth. 4. A and B. *** 5. All the above. 1173. direct pulp capping is done in: 1. Primary molar. 2. Primary incisor. 3. Permanent molar. *** 4. None of the above. 1174. Indirect pulp capping done in: 1. Primary molar. 2. Premolar and molar. 3. Incisors. 4. All the above. *** 1175. What do we use as temporary filling material in anterior reign when aesthetic is important: 1. Composite. 2. Glass ionemer cement. *** 3. Zinc oxide eugenol. 1176. The maximum dose of X-ray exposure dose for radiographic technique: 1. 100 milli roentgens per week. *** 2. 10 roentgens per week. 3. 100 roentgens per week. 4. 300 roentgens per week. Dental Decks - page 62

Person who works near radiation can be exposed in one year to a maximum dose of 5 Rem (0.1 Rem per week) 8 . 433 = 85 8 = 1. 2. 3. 4. 1. 2. 3. 4. 1177. Acute periapical abscess characterized by: Varying degree of pain. Varying degree of swelling. Some time not shown on the radiograph. All the above. *** 1178. It is preferable to be the length of the handle of the custom tray: 10mm. 20mm. 15mm. *** 25mm. 1179. is the art and science of functional, anatomic and cosmetic reconstruction of missing or defective parts in the maxilla, mandible or face by the use of non living substances: Complete denture. Maxillofacial prostheses. *** Orthodontics. Partial denture. 1180. .. Is the one that provides application and device to restore aesthetic and functional requirements to patients with maxillofacial defects: Endodontist. Pedodontist. Maxillofacial prosthodontist. *** Peridontist. 1181. The objectives of maxillofacial prosthetics: Aesthetic. Functions. Protect the tissues. All. *** 1182. The type of maxillofacial defects: Congenital defects. Acquired defects. Developmental defects. All. *** 1183. Cleft palate, cleft lip, missing ear, prognathism are:

1. 2. 3. 4.

1. 2. 3. 4. 1. 2. 3. 4. 1. 2. 3. 4.

1. 2. 3. 4. 1. 2. 3. 4. 1. 2. 3. 4.

1. 2. 3. 4.

1. 2. 3. 4. 1. 2. 3. 4.

1. 2. 3. 4.

Acquired defects. Congenital defects. *** Developments defects. None. 1184. Accidents, surgery, pathology are: Acquired defects. *** Developments defects. Congenital defects. None. 1185. Extra-oral restorations are: Radium shield. Ear plugs for hearing. Missing eye, missing nose or ear. *** All. 1186. Lost part of maxilla or mandible with the facial structures is classified by: Intra-oral restorations. Extra-oral restorations. Combined intra-oral and extra-oral restorations. *** All. 1187. The lack of continuity of the roof of the mouth through the whole or part of its length in the form of fissure extending anteroposteriorly is: Obturator. Splint. Stent. Congenital cleft palate. *** 1188. The factors that influence the induction of cleft palate: Hereditary. Environmental. A and B. *** None. 1189. A prosthesis used to close a congenital or acquired opening in the palate is: Stent. Splint. Obturator. *** None. 1190. -.. Are appliances used for immobilization of fragments of broken parts of jaw bones in their original position until repair takes bleeding?

1. 2. 3. 4. 1. 2. 3. 4.

1. 2. 3. 4. 1. 2. 3. 4.

1. 2. 3. 4.

1. 2. 3. 4. 1. 2. 3. 4.

Splints. *** Stents. Obturators. Speech aids. 1191. The prepared surface of an abutment to receive the rest is called: Minor connecter. Major connecter. Rest seat. *** None. 1192. The part of a removable partial denture that contacts a tooth it affords primarily vertical support is called: Minor connecter. Major connecter. Rest. *** None. 1193. The part of a removable partial denture is: Rests. Major connecters. Retainers. All. *** 1194. A rigid part of the partial denture casting that unites the rests and another part of the prosthesis to the opposite side of the arch is called: Minor connecter. Major connecter. *** Retainer. Rest. 1195. The part of a removable denture that forms a structure of metal struts that engages and unites the metal casting with the resin forming the denture base is called: Minor connecter. Major connecter. Denture base connecter. *** Retainer. 1196. The rests are classified into: Anterior rests. Posterior rests. A and B. *** None. 1197. The surveyor instrument consists of:

1. 2. 3. 4.

1. 2. 3.

1. 2. 3. 4.

1. 2. 3. 4. 1. 2. 3. 4.

1. 2. 3. 4.

1. 2. 3. 4.

Vertical arm. Cast platform or table. Small analysis rod. All. *** 1198. The primary guiding surface that determines the insertion for the partial denture is: The tooth surface opposite to the edentulous areas. The tooth surface adjacent to the edentulous areas. *** None. 1199. The one who is supposed to give the correct design of the removable partial denture: Prosthodontist. *** Technician. Assistant. None. 1200. To fabricate a removable partial casting requires making a second cast of high-heat investment material this cast is called: Study cast. Master cast. Refractory cast. *** All. 1201. Kennedy divided all partial edentulous arches: Tow main types. Three main types. Four main types. *** Five main types. 1202. According to the Kennedy's classification, the bilateral edentulous areas located posterior to the remaining natural teeth is: Class one. *** Class tow. Class three. Class four. 1203. According to the Kennedy's classification, unilateral edentulous area with natural teeth remaining both anterior and posterior is: Class one. Class tow. Class three. *** Class four. 1204. We should select the shade for a composite resin utilizing a:

1. 2. 3. 4. 1. 2. 3. 1. 2. 3. 4.

1. 2. 3. 4. 5. a) b) c) d)

Bright light. Dry shade guide. Dry tooth isolated by the rubber dam. None of the above are corrects. *** 1205. 4th canal in upper first molar is found: Lingual to MBC. *** Buccal to MBC. Distal to MBC. 1206. To get file size 24, the following length should be cut from file size 20: 1mm. 2mm. *** 3mm. 4mm. 1207. Red color endo file acccording ADA a. 20 b. 25 c. 30 d. 35 1208. The following canals may be found in an upper molar: Mesio-buccal. Disto-buccal. Mesio-palatal. Disto-lingual. Palatal. 1+2+4. 1+2+4+5. 2+3+4+5. 1+2+3+5. *** .9021 . Dental Decks - page 2078 1210. Patient need fixed bridge after you check in mouth of the patient see change color of bridge to cloudy to milky what causes? a. excessive fired. *** b. reduced fired c. excessive moisture d. increased poursity

Dental Decks2 - Page 712 when porcelain is fired too many times it may devitrify this appears as a milky state and make glazing very difficult 1211. The Ideal crown-to- root ratio of a tooth be utilized as abridge abutment is: a- 3:1. b- 2:1. c- 1:2. *** d- 1:1. Dental Decks 1212. First step in tx of abused tissue in patient with existing denture is to: Educate the patient. *** Dental decks page 401 1213. The primary role of the anterior teeth on a denture is: Esthetics Dental decks - page 407 1214. pt have lesion in toung which sufaring from scar fever>> the lesion when remove leave the bleeding area under it ...diagnosis is a. leukoplaqua b. candida c. ulser 1215. Food low cariogenic affect the following should be characteristic: 1/low buffring capacity 2/ph low than3 3/contain mineral http://www.adha.org/CE_courses/course7/table4.htm 1216. when resection the tip of root in apicsctomy..the cut shoud be.. a. perpendicular to the long axis of tooth b. paraller to long axis c. acut angle. *** d. obtuse angle 1217. Child patient with painfull herpese simplex the treatment is acyclovir with a- Vit C b- Local anesthesia with multi-Vitamin c- Local anesthesia with protine :668 .

) ( 1218. adding of surfactant to irrigation solution during RCT to increase wettability of canal walls by: a-lowering surface tension*** b-increasing surface tension c-passing through dentinal tubules Surfactant - Wikipedia, the free encyclopedia 1219. patient returned to you after 1 month from doing amalgam filling with definite severe pain, due to: a) contamination with moisture leading to amalgam expansion. *** b) unidentified pulpal exposure c) supra occlusion d) gingival access 1220. the function of post a. provide retention for a crown b. enhance the strength of the tooth c. provide retention for a core d. provide the root canal sealing 1221. patient come with severe stained anterior central left maxillary incisor with small distal carie & lost incisal edge treated by a. full ceramic restoration 1222. the ideal post drill for most posterior teeth is a. gates glidden size 3 b. peeso drill size 3-6 c. profile size 60-70 d. peeso drill size 2-3

ENDODONTICS Fifth Edition page 924 1223. the decision to retreat substandard endodontics shoud be based on 1-radiographic evaluation of the endodontictreatment quality 2-------periapical pathology 3-patient symptoms

4-physical exploration of the root canal typ and quality seal 5-restorative treatment plan 1224. the most common injures in child is: 1/ tooth 2/ root 3/ intrusion of the tooth inside the socket well 1225. Child with previous history of minor trauma with excessive bleeding we do test the result is prolong PT & slightly increase clotting time & . Test is +ve. the diagnosis is: a. hemophelia B. b. thrombocytopenia. c. vit.K deficiency ) ( dental decks Prothrombintime - Wikipedia, the free encyclopedia dental secrets page 274 A clot may fail to form because of a quantitative or functional platelet deficiency. The former is most readily assessed by obtaining a platelet count. The normal platelet count is 200,000500,000 cells/mm3 Prolonged bleeding may occur if platelets fall below 100,000 cells/ mm3. Treatment of severe thrombocytopenia may require platelet transfusion. Qualitative platelet dysfunction most often results from aspirin ingestion and is most commonly measured by determining the bleeding time. Prolonged bleeding time requires consultation with a hematologist 1226. pt came 2 ur clinic complain from his gingiva which bleeds alot with any little pressure, on clincal examination u found pin point purple dots and general rash like of eccymosis on his body laboratory finding : highly decrease in platlets(slightly more than 25000) have a history of eccymosis and brusing all over his body: a. thrombocytopinic purpura*** 58333 ) ( 1227. The best test for vitality of crowned tooth is - Cold whis rupper dam 1228. Netros oxide interference with - Vit b12***

- Vit b6 - Vit a - Vit c 1229. Salivary gland role in maintaining tooth and bacteria integrity on the oral cavity is done by: a- Bacterial clearance. b- remineralization. c- Buffering and direct anti-bacterial role. d- Bacterial clearance and reminerlization.*** 1230. physiolgical activity of local anesthesia a. lipid solubility b. diffusbility c. affinity for rotien bendin d. percent ionizing at physiologic pH e. vasodilition properties 1231. physiolgical activity of local anesthesia a. lipid solubility of unionized form b. diffusbility c. affinity for rotien bendin d. percent ionizing at physiologic pH e. vasodilition properties 1232. physiolgical activity of local anesthesia a. lipid solubility of unionized form b. water solubility of unionized form c. lipid solubility of ionized form d. water solubility of ionized form http://webcache.googleusercontent.com/search?hl=ar&site=&btnK=&gs_sm= &gs_upl=&safe=active&q=cache:SbzBndI3vQUJ:http://nursingpharmacology.i nfo/Central/Local_Anes/LAobj1.htm+lipid+solubility+of+unionized+form&ct=c lnk

1233. Intraosseous injection of local anesthesia to lower molar a. Perforate the bone mesial to the tooth b. Give one with 1:50000 epinipherine c. Give 1/4 to 1/5 of the cartridge d. Ask the pt if he has numbness in his lower lip after injection

. 1234. re implant of avulsed tooth , what you do A- optimal reposition and fixed splint B-optimal reposition and flixable splint C-observe D-watch with splint periodontally avulsed tooth Splint tooth in position with (preferably) a flexible splint. Have patient bite into occlusion to be certain that the position is correct before applying the splint. The splint will be kept in place for about one week http://webcache.googleusercontent.com/search?hl=ar&site=webhp&btnK=& gs_sm=&gs_upl=&safe=active&q=cache:XgF8SRTbQYJ:http://www.doctorspiller.com/avulsed_teeth.htm+Splint+tooth+in+pos ition+with+%28preferably%29+a+flexible+splint.+Have+patient+bite+into+occ lusion+to+be+certain+that+the+position+is+correct+before+applying+the+spli nt.+The+splint+will+be+kept+in+place+for+about+one+week&ct=clnk .9021 Nasids *** .9021 1237. Thermal pulp test principle of: 1/blood supply of pulp 2/ nerve supply of pulp 3/AO fibers Assessment of pulp vitality is most frequently accomplished by electric pulp test and/or cold testing. The vitality of the pulp is determined by the intactness and health of the vascular supply, not the status of the pulpal nerve fibers. Even though advances are being made with regard to determining the vitality of the pulp on the basis of the blood supply, this technology is not accurate enough to be used on a routine basis in a clinical setting Pathways of the Pulp, 9th ed 1238. Rideal-Walker test is the tsst for detecting activity of

a. b. c. d.

***Disinfection Antibiotics Sterelyzation by dry heat Sterelyzation by wet heat The Rideal-Walker coefficient is a figure expressing the disinfecting power of any substance and is obtained by dividing the figure indicating the degree of dilution of the disinfectant that kills a microorganism in a given time

.9021 : .9022 . - *** ( ) . : ( ) . ( ) . ( ) : : . : . __________________________________ ............ 1241. Testing a tooth with ceramometal fused to metal with a) cold test b) cold and hot c) cold with rubber dam 1242. die ditching means a) carving apical to finish line b)carving coronal to finish line c) mark finishline with red pen

1243. When esthetic is important,posteriorclass I composite is done in : a. Subgingival box. b. Bad oralhygiene. c. Contact free area. d. Class I without central contact 1244. Child 10 years old came to the clinic with periodontitis associated with the 1ry & 2ry dentition with severe generalized bone destruction and calcificationon the general examination hyperkeratosis of hands & feet is noticed the diagnosis is: a. Hypophosphotasia. b. Pre_puberty periodontitis. c. Papillon lefevre syndrome***. d. Juvenile periodontitis Oxford Handbook 4th 1245. pt. with palm oplanter keratosis and periodontitis in permanent dentition is likely to has: a. papillon lefevere syndrome b. down"s syndrome c. leukemia d. hypophosphatesia 1246. When take x ray in upper premolar to locate lingual root using mesial shift it will apear a: distal b: buccal c: lingual d: mesial.*** 1247. While taking X-ray for upper right first premolar with two equal roots Using mesial slob, its lingual root will move [ comparing to the zigomatic process]: -distal. -Mesial -Palatal Lingual ) ( 1248. Maternal immunity that passed from mother to the fetus through the placenta, IgG, IgA is considered:

1. 2. 3. 4.

a-Natural passive immunity.*** b-Natural active immunity. c-Acquired passive immunity. d-Acquired active immunity. 1249. When u want to make immediate complete denture after extraction all teeth what the type of suture u will use a. horizontal mattress suture b. vertical mattress suture c. interrupted suture d. continous locked suture 1250. Smear layer composed of: Dentine debris. Inorganic particles. Bacteria. All the above. Dental secrets: - The smear layer is a film of microcrystalline debris that remains on dentin after it is cut with rotary instruments. - After removing the organic and inorganic debris of the smear layer by etching Art and science of operative dentistry 2000 - The composition of the smear layer is basically hydroxyapatite and altered denatured collagen. Oxford Handbook of Clinical Dentistry - 4th Ed. (2005) The smear layer consists of an amorphous layer of organic and inorganic debris, produced by cutting dentine. ( ) The smear layer is composed of dentin debris, organic material and microorganisms that adhere to the root canal walls obstructing theopenings of the dentin tubules, which can hinder the action of medicine in the dentin tubules and root canal system. Removal of this layer is important for the success of the endodontic treatment and is obtained with the use of chemical solutions during root preparation8. Pesquisa Odontolgica Brasileira - Smear layer removal capacity of :disinfectant solutions used with and without EDTA for the irrigation of canals a SEM study

1251. during making filing by Ni/Ti it gets fractured due the property of: a- Rigidity & b- Axial fatigue*** c- tostion Basic Dental Biomaterials Sciense :Causes of # are 1- Tensile 2- Fatigue 3- Creep 4- Steress corrosion cracking 5- controled crack propagation Fatigue is: occurs when cyclic loading is applied, like fracture of a denture retentive clasp arm after several years of service .9010 : malhoyid .9012 eruption hematoma 1254. after remove impacted 3rd lower moler,there is parasthesia why? a. irritating the nerve during extruction b. broke mandible 1255. The test for testing the bur all the blades of the burs path through 1 point called a. ronted b. constidty c. routed and constedety d. none of above Two terms are in common use to measure this characteristic of bur heads concentricity and runout. Concentricity is a direct measurement of the symmetry of the bur head itself. It measures how closely a single circle can be passed through the tips of all of the blades. Thus, concentricity is an indication of whether one blade is longer or shorter than the others. It is a static measurement not directly related to function

Runout, on the other hand, is a dynamic test measuring the accuracy with which all blade tips pass through a single point when the instrument the concentricity of the head, but also the is rotated It measures not only accuracy with which the center of rotation passes through the center of .the head 5333 sience &art 668

1256. Mobility in midface with step deformity in front zygomatic suture. Diagnosis: d. Lefort II. e. Lefort III.*** f. Bilateral zygomatic complex fracture. Lefort III step deformity in the orbital rim Lefort II step deformity in front zygomatic suture Oxford Handbook of Clinical Dentistry - 4th Ed. (2005) page 270 Diagnosis Le Fort I may occur singly or associated with other facial fracture. The tooth-bearing portion of the upper jaw is mobile, unless impacted superiorly. There is bruising in the buccal sulcus bilaterally, disturbed occlusion, and posterior 'gagging' of the bite. Grasp the upper jaw between the thumb and forefinger anteriorly, place thumb and forefinger of other hand over the supraorbital ridges, and attempt to mobilize the upper jaw to assess mobility. Spring the maxillary teeth to detect a palatal split. Percussion of the upper teeth may produce a 'cracked cup' sound. Le Fort II and III fracture produce similar clinical appearances; namely, gross oedema of soft tissues, bilateral black eyes (panda facies), subconjunctival haemorrhage, mobile mid-face, dish-face appearance, and extensive bruising of the soft palate. Look for a CSF leak and assess visual acuity. Le Fort II fracture may also show infra-orbital nerve paraesthesia and step deformity in the orbital rim. Peculiar to Le Fort III fracture are tenderness and separation of the frontozygomatic suture, deformity of zygomatic arches bilaterally, and mobility of entire facial skeleton.

And MASTER DENTISTRY- Oral and Maxillofacial, Surgery, Radilolgy, Pathology and Oral Medicine Le Fort I is the lowest level of fracture, in which the tooth-bearing part of the maxilla is detached. Le Fort II or a pyramidal fracture of the maxilla involves the nasal bones and infraorbital rims. Le Fort III involves the nasal bones and zygomatic-frontal sutures and the whole of the maxilla is detached from the base of the skull. step deformity in front 6 suture zygomatic 6 1257. two weeks baby born with 2 anterior teeth which is highly mobile , and his mother have no problem or discomfort during nursing him what is ur managemnt: a. do not do anthing as the baby have no problem during feeding b. do not do anything as the mother don`t feel discomfort c. u must extract as soon as possible to avoid accident inhalation of them d. do nothing , it will shell by it self Natal teeth are usually members of the primary dentition, not supernumerary teeth, and so should be retained if possible. Most frequently affect mandibular incisor region and, because of limited root development at that age, are mobile. If in danger of being inhaled or causing problems with breastfeeding, they can be removed under local analgesia . oxford hand book of clinical dentistry .9011 Maxillary central incisors curve either towards the labial or palatal aspect .at about the apical third level Maxillary lateral incisor the apical third tends to curve distally and the .canal is often very fine with thin walls Maxillary canine The length of this tooth can be difficult to determine on radiographs as the apex tends to curve labially and the tooth will appear to be shorter than it .actually is Maxillary first premolar The roots of these teeth are very delicate and at the apical third they may

curve quite sharply buccally palatally, mesially or distally, so instrumentation needs to be carried out with great care Maxillary second premolar As with the first maxillary premolar the apical third of the root may curve quite considerably, mainly to the distal sometimes buccally Maxillary first molar The palatal root has a tendency to curve towards the buccal and the actual length apparent length on a radiograph will be shorter than its Mandibular second premolar unless the radiograph reveals a sharp distal curve at the apex as shown in the extracted tooth 1259. Most common cause of chipped porcelain in PEM a-Thin layer of metal b-Thin layer of porcelain c- Centric occlusal contact at the junction of porcelain and metal 1260. The forces action through a FPD on to the abutment tooth should be directed 1- As far as possible at right angles to the long axes of the teeth 2- Parallel to the long axes of the teeth 3- By decreasing the facio-lingual dimension of the pontic 4 -By decreasing the Mesio-lingual dimension of the pontic 5- In a mesial direction so that teeth nearer the midline will offer additional support a. 1+3+4 b.1+2+5 c. 1+4+5 d. 2+3 e. 2+4 f. 2+5 1261. which not compatible to the pulp A- GIC B- Zinc phosphate cement C- Zinc polycarboxylate cement 1262. The working time of zinc phosphate cement is shortened by a- concentrating the acid b - warming of glass slab c- incremental mixing of powder d-all of the above 1263. you sent shade of PFM ,technician give you different color with same shade

a-non uniform porcelin b-thick opaque c-thin opaque 1264. most abrasive contact a-tooth to tooth b- porcelin to tooth c-gold to tooth 1265. length of post a-1/2 root b- 2/3 root c-1/2 root containing in bone d-as much longer and leave 4 mm apical seal 1266. concentrating of acid used in etching porcelain veneer a-9.6 % hydrofluoric acid *** b-35%phosphoric acid c-37%phosphoric acid d-37%hydrflouric acid 1267. Placement of maxillary anterior teeth in complete dentures too far superiorly and anteriorly might result in difficulty in pronouncing a) f and v sounds.*** b) d and t sounds. c) s and thsounds. d) most vowels 1268. An advantage of rubber-base impression material over reversible hydrocolloid material is that rubber base impression material a) will displace soft tissue. b) requires less armamentarium. c) is significantly more accurate. d) is more accurate if saliva, mucous or blood is present. 1269. Best provisional coverage for anterior teeth is a- Tooth colored polycarbonate crown b- Stainless steel crown c- Zinc oxide engenal 1270. In a clinical research trial we primarily need to: a. Get written subject in the patient's own native language. b) verbally write subject briefly c) tell patients that they are part of study.

1271. Radiolucent structure occupied by a radioopaque structure that forms a mass of disorganized arrangement of odontogenic tissue : a. Complex odontome*** b. Calcifying Epithelial Odontogenic Cyst. c. Compound odontome. .............Oxford 4th edition 1272. Which of these canal irrigants is UNABLE to kill E.feacalis: -NaOH -MTA -Chlorhexidine .......................1273. The least effective irrigant against E.feacalis: a)sodium hypochlorite b)tetracycline c)iodine d)Chlorohexidine 1274. While dentist making Biomechanical preparation by using NI TI file it broken this is because the property of ; a.. elastisity and memory b.. reigidity and memory c.. axial fatigue*** d ..tarnish the most important reason for breaking ni ti files is cyclic fatigue and torsional stress 1275. root end resiction ...what is the codtioning a. cetric acid b. tetracyclin c. EDTA 1276. during endodontic surgery the irrigation solution used is : a.Saline b.EDTA c.Naocl 1277. Sterilization means killing of a. Virus b. Fungi c. Bacteria

d. Virus,fungi,bacteria,bacteria spores e. Virus ,fungi,bacteria 1278. Edentulous pt cl II kenndy classification 2nd premolar used as abutment when we surving we found mesial under cut what is the proper clasp used: 1/wrought wire with round cross section 2/ wrought wire with half round cross section 3/cast clasp with round cross section 4/ cast clasp with half cross section ) ( . 1279. Balance occlusion in complete denture help in : -retention -stability*** 1280. after a trauma on a primary tooth what is the least possibility? A. change in the permanent tooth enamel color. B. change the primary tooth color C.apix pathologic on primary tooth 1281. In case of infiltration anesthesia we give a) sub mucosal b) intraosseous c) sub periosteal d) none 1282. The patient who have not breakfast , we never give him anesthesia because a) hyperglycemia b) hypoglycemia c) increased heart rate d) hypertension 1283. Gingivitis means a) inflammation of the periodontal ligaments b) inflammation of the bone c) inflammation of the gingiva d) inflammation of the tongue 1284. Patient is suffering a pain during sleep the diagnosis is a) inflammation of dentin b) inflammation of enamel c) inflammation of cementum d) inflammation of pulp

1285. Permanent restoration is a) calcium hydroxide b) amalgam c) alginate d) zinc oxide eugenol 1286. Composite is used mainly for a) anterior teeth b) posterior c) a+b d) none 1287. For injection local anesthesia in the lower jaw we use a) short needle b) long c) none 1288. In case of advanced upper jaw to the lower this is called a) angle class I b) angle class II c) angle class III d) All of the above 1289. The best method for brushing a) vertical b) horizontal c) bass sulcular method d) all of the above 1290. Apicoectomy means a) surgical removal of the apical portion of the root b) removal of one or more roots c) the root and the crown are cut lengthwise d) none 1291. The instruments for examination are a) probe and tweezer b) mirror c) a + b d) amalgamator 1292. Panorama x-ray is used for a) Periapical tissues b) interproximal caries c) giving complete picture for upper and lower jaw d) none 1293. Adrenaline is added to local anesthesia for a- increasing the respiratory rate b- prolonging the effect of local anesthesia c- iecreasing the bleeding

d-none 1294. ......is a white lesion a- lichen planus b- cancer c- heamatoma d-none 1295. fordyce's spots is on a- tongue b- oral mucosa*** c- upper lip d- throat
. . . ... : .

1296. Is an anticoagulant agent a- aspirin b- heparin c- paracetamol d- evex PTT 1297. Duct of submandibular gland is a- warton b- bartholin c- barvenous d- stenson http://webcache.googleusercontent.com/search?q=cache:Z3LP9fCsNQUJ: emedicine.medscape.com/article/882358overview+Duct+of+submandibular+gland&cd=3&hl=ar&ct=clnk&gl=sa&clie nt=firefox-a 1298. leukoplakia is present on a- the mouth b- eye c- heart d- lungs 1299. Cranial nerves are a- 12 nerves b- 14 c- 10

d- 16 1300. According to two digits system 42 means a- lower right lateral incisor b- upper left lateral incisor c- upper right lateral incisor d- none 1301. A preventive agent is a- composite b- Glassionomer c- fluoride d- zinc oxide eugenol 1302. One of the following releases fluorides a- composite b- Glassionomer c- fluoride d- zinc oxide eugenol 1303. Dental plaque is formed after a- 6 hours b- 12 hours c- 24 hours d- 48 hours 1304. Tooth paste with fluoride is a- systemic application b- topical application c- a+b d-none 1305. Fluoride in water its concentrate a- 2 ppm b- 1 ppm (part per million) c- 3 ppm d- none 1306. Too much ingestion of fluoride may lead to a- dental caries b- dental fluorosis c- gingivitis d- none 1307. Saliva ejector is placed a- at the side of working b- under the tongue c- opposite the working side d- b+c

1308. abcd-

Deficiency of vit C leads to scurvy anemia ricket defect in blood clotting

1309. Deficiency of vit K leads to a- scurvy b- anemia c- ricket d- defect in blood clotting 1310. Deficiency of vit D leads to a- scurvy b- anemia c- ricket d- defect in blood clotting 1311. at which age will a child have 12 permanent and 12 primary teeth a. 9 b.11 1312. Medicine ethics aim to a. The dentist should be studied to know patient psycho b. not to compromise or undermine ability to treat patient in community as professional.

c. ability to make decision d. ---e. All of above *** 1313. patient have during recurrent normal check he had ( diffirent sense) on percussion on his tooth and x_ray widening lamina dura apical 3rd? a)chronic apical priodontitis b)acute apical periodontitis c)chronic abcess 1314. during surgery firmly handle forceps of flap tissue: A)stillis forceps b)Adison forceps*** Adson forceps: These forceps are non-toothed, and so are ideal for holding delicate tissues 1315. Lemphadenopathy due to: a) infection b) lymphocytic leukemia c) HIV d) pernicious aneamia e) a.b and c*** 1316. The aim from prosthetic surgery: A) increase stability .retention ,ridge dimension b) increase vertical dimension. c) esthetic anterior. 1317. Composite can be done in: a) conservative class one*** b) uncontrolled application cl2 proximal c) deep gingival margin. 1318. selection of steam heat over dry hot oven: a) protein cell collagene under high pressure*** b)save effect on shard instruments . no dull or corrosion. 1319. most common cause of caries : A) Xerostomia b) diet sugar consumption 1320. EPOXY RESIN: A. contain formaldehyde toxic

B.contain cortico steroids C.resrbable so it weakens the endofill. D.all 1321. Extra canal in upper 6 present in which Root: A) MB ROOT*** B) DB ROOT C) PALATAL ROOT 1322. In case of internal resorption your ttt is: a) Ca(OH)2 application*** b) formocresol medicament c) zinoxide eugenol 1323. In primary tooth for restoration before putting the filling u put: a. base. b. calcium hydroxide. c. varnish. c. you put the filling after proper cleaning and drying *** 1324. a child with thumb sucking habit he will develop: a. anterior openbite and posterior crossbite*** 1325. Immature tooth with external apical third resorption: a) Ca(OH)2 application b) Apexofication & GP filling 1326. colour of normal gingiva in interplay between : a. Keratin- vascularity melanin- epithelial thickness*** 1327. during mentoplasty doctor should take care for injury of what nerve .. a. lower branch of the facial nerve http://webcache.googleusercontent.com/search?q=cache:g7i1Xgtg1H0J:w ww.realself.com/question/nerve-damage-risk-chin-reductionsurgery+during+mentoplasty+doctor+should+take+care+for+injury+of+wh at+nerve&cd=5&hl=ar&ct=clnk&gl=sa&client=firefox-a
I plan to have Rhinoplasty and Chin Redution surgery together but I am hesitant as I read that these surgeries may lead facial paralysis? How true is it?

1328. Alginate contains calcium sulphate in concentration of a) 40% b) 50% c) 12% d) none https://docs.google.com/viewer?a=v&q=cache:FYw2_gty34QJ:www.dent. ohio-

state.edu/courses/d430/Impressions%2520and%2520Casts/Diag%2520cas ts,%2520alginate,%2520stone.pdf+calcium+sulphate+in+alginate&hl=ar&g l=sa&pid=bl&srcid=ADGEESjCQbTD_EPq6ZNCywT_a761qwCBwzTLq7MJp0 h-U-9ywDTqCpHued5ZtnCgaMEDi9hTC89r_4e5TT0eqSYwT1KlpAyLXaclB8BgRQFX0uwU3HtLtG6axFiv2ez0sSXFUWoO2X3&sig=AHIEtb T40_sqW6SS1pQEX9pF8lbOs9puNw&safe=on 1329. Elastic impression material is a) rubber*** b) Plaster c) zinc oxide d) compound http://denture.tv/elastic-non-impression-materials/
Elastomers Main Types Polysulphides Silicones Polyethers

1330. Amputation means a) surgical removal of the apical portion of the root b) removal of one or more roots*** c) the root and the crown are cut lengthwise d) none http://www.stricklanddental.com/Procedures_RootAmputation_5964933. aspx
Root amputation is a specialized dental procedure, whereby one root is removed from a multi-root tooth. The tooth is then stabilized and rendered fully functional with a crown or filling. The multi-root teeth best suited to the root amputation procedure are the molars at the back of the mouth. These large flat teeth have either two or three roots depending on whether they are situated on the upper or lower jaw

1331. Hemisection means a) surgical removal of the apical portion of the root b) removal of one or more roots c) the root and the crown are cut lengthwise*** d) none http://www.simplestepsdental.com/SS/ihtSS/r.==/st.32226/t.31470/pr.3.h tml
Hemisection is the process of cutting a tooth with two roots in half. Each half tooth consists of half the crown (top of the tooth) and one root. Hemisection literally means "dividing in two." It often is done by a periodontist, a specialist in gum disease. But any experienced dentist, endodontist or oral surgeon can do the procedure.

1332. For treatment of pericoronitis

a- extraction of the tooth b- analgesic + sterility + antibiotic** c- cleaning with concentrated phenol d- none http://www.webmd.com/oral-health/guide/pericoronitis ) ( 1. 2. 3. 4. 1333. Outline of Pericoronitis treatment may include: Mouth wash and irrigation.*** Extraction of the opposing tooth. Surgical removal of the causative tooth. All the above. ( ) ) ( 1334. Attrition may be caused by a- friction due to pipe b- friction during sleep*** c- gastric acid d- none http://webcache.googleusercontent.com/search?q=cache:bd5VlX113EkJ:w ww.adha.org/CE_courses/course9/loss_of_structure.htm+cause+of+tooth +structure+loss&cd=1&hl=ar&ct=clnk&gl=sa&client=firefox-a 1335. Sterilization in dry oven a- one hour at 160 c b- 90 minutes at 160 c c- two hours at 160 c*** d- none http://en.wikipedia.org/wiki/Dry_heat_sterilization 1336. According to the universal system 6 means a- upper left first molar b- lower left first molar c- lower right first molar d- none*** http://webcache.googleusercontent.com/search?q=cache:Tuwg09LYOyQJ: en.wikipedia.org/wiki/Universal_numbering_system_%28dental%29+unive rsal+system+number+in+dentistry&cd=1&hl=ar&ct=clnk&gl=sa&client=fire fox-a number 6 refers to upper right canine

1337. The roots of the following teeth are closely related to the maxillary sinus a- canine and upper premolar b-lower molar c- upper molar and premolar d- none 1338. Burs is a- critical items*** b- semi critical c- non critical d- all of the above http://webcache.googleusercontent.com/search?q=cache:qpsH2UFMqxgJ:ww w.alivetek.com/portfolio_loader.php%3Fobjectid%3D275+cemi+cretical+instr ument+in+dentistry&cd=3&hl=ar&ct=clnk&gl=sa&client=firefox-a 1339. Patient positions are a- upright position b- supine position c- sub supine d- all of the above*** http://webcache.googleusercontent.com/search?q=cache:clF5p_vAdc0J:s wathi180.hubpages.com/hub/Patient-and-Operator-positions-inDentistry+patient+position+in+dentistry&cd=1&hl=ar&ct=clnk&gl=sa&clie nt=firefox-a 1340. For the right handed dentist seated to the right of the patient the operator zone is between a- 8 and 11 o'clock*** b- 2 and 4 o'clock c- 11 to 2 o'clock d- all of the above 1341. For right handed dentist, the static zone is between a- 8 - 11 o'clock b- 11 - 2 o'clock*** c- 2 - 4 o'clock d- all of the above 1342. For right handed dentist, the assistant's zone is between a- 8 - 11 o'clock b- 11 - 2 o'clock c- 2 - 4 o'clock d- all of the above http://books.google.com.sa/books?id=r3E1SujL9IC&pg=PA253&lpg=PA253&dq=For+right+handed+dentist,+the+assistan t%27s+zone+is+between&source=bl&ots=ezPvo5shjX&sig=Isq3hROR_2em

yi1DiodlawaTx6g&hl=ar&sa=X&ei=96guT7yQMsbpObiZvIMO&ved=0CCoQ 6AEwAQ#v=onepage&q=For%20right%20handed%20dentist%2C%20the% 20assistant%27s%20zone%20is%20between&f=false 1343. Mouth mirror is a- critical items b- semi critical*** c- non critical d- all of the above http://webcache.googleusercontent.com/search?q=cache:qpsH2UFMqxgJ: www.alivetek.com/portfolio_loader.php%3Fobjectid%3D275+cemi+cretica l+instrument+in+dentistry&cd=3&hl=ar&ct=clnk&gl=sa&client=firefox-a 1344. HVE is placed a- at the side of working*** b- under the tongue c- opposite the working side d- b+c

https://docs.google.com/viewer?a=v&q=cache:Ha3u16OtdfgJ:www.csi.edu/fa cultyAndStaff_/webTools/sites/Bowcut58/courses/408/ch36.ppt+Grasping +the+HVE+is+by&hl=ar&gl=sa&pid=bl&srcid=ADGEESgOx89QA0J9awqq4ix Fr_gDfWXwU55zeB0AGJBEUSGisgRiZQ7iLy2gU6MokyVcULrG_TjafAXvh2yrl sv76YC0gf9WBSTEd2gd5rl7oTg4BjV_03EcXla1UBVdCE5aP_SBjUM&sig=AHIEtbSU47b702Clxj J2yDENjpQxjBUF_Q&safe=on

1345. abcd-

Grasping the HVE is by thumb to nose grasp pen grasp a+b*** none

https://docs.google.com/viewer?a=v&q=cache:Ha3u16OtdfgJ:www.csi.edu/fa cultyAndStaff_/webTools/sites/Bowcut58/courses/408/ch36.ppt+Grasping +the+HVE+is+by&hl=ar&gl=sa&pid=bl&srcid=ADGEESgOx89QA0J9awqq4ix Fr_gDfWXwU55zeB0AGJBEUSGisgRiZQ7iLy2gU6MokyVcULrG_TjafAXvh2yrl sv76YC0gf9WBSTEd2gd5rl7oTg4BjV_03EcXla1UBVdCE5aP_SBjUM&sig=AHIEtbSU47b702Clxj J2yDENjpQxjBUF_Q&safe=on page8-10 1346. Carbohydrate is digested in a- mouth*** b- small intestine c- large intestine d- none 1347. Carbohydrate is essential for a- building the body b- supplying the body with energy*** c- a+b d- none http://www.wisc-online.com/objects/ViewObject.aspx?ID=AP15806 1348. The following factors effect the health 1- heriditary

2- environement 3- social and economic factors 4- family welfare A) 1+2 B) 1+2+4 C) 1+2+3 D) all of the above. http://www.who.int/hia/evidence/doh/en/ ) ( 1349. Diagnostic cast wax up help in: a- Predict the result of treatment b- Explain the treatment plan to patient c-All of the above 1350. in mean of compressive strength ,tensile strength which is strongest a- resin cement*** b- zinc phosphate c- G.I ref. clinical procedure chapter 7 principle of tooth preperation 1351. shoulder is the finish line of choice for a-full veneer b-PFM*** c- crown indication of shoulder :facial margin of metal ceramic crown, complete ceramic crown ref. clinical procedure chapter 7 principle of tooth preperation 1352. Which of the following characteristics of inlay wax is its major disadvantage a) flow. b) rigidity. c) hardness. d) high thermal expansion*** http://books.google.com.sa/books?id=SdOOEbeyNVkC&pg=PA90&dq=maj or+disadvantage+of+inlay+wax&hl=ar&sa=X&ei=YYUyT_eMJonpObfzkYIH &ved=0CDIQuwUwAA#v=onepage&q=major%20disadvantage%20of%20inl ay%20wax&f=false 1353. As the gold content of a dental solder decreases ,the a) hardness decreases.

b) ductility increases. c) corrosion resistance decreases. d) ultimate tensile strength decreases*** http://books.google.com.sa/books?id=Pb_lcACduEQC&pg=PA470&lpg=PA 470&dq=As+the+gold+content+of+a+dental+solder+decreases,the&source =bl&ots=7MuN6i7D0h&sig=SH5ixJfEkX0VyeYwRgrbIh2x9A&hl=ar&sa=X&ei=lIEyT_ShNMOgOsPQhfAG&ved=0CDEQ6AE wAg#v=onepage&q=As%20the%20gold%20content%20of%20a%20dental %20solder%20decreases%2Cthe&f=false 1354. The most ductile and malleable metal is: a) Sliver. b) Gold.*** c) Copper. d)Platinum.. http://webcache.googleusercontent.com/search?q=cache:qfnIT3A2s3UJ:e n.wikipedia.org/wiki/Ductility+The+most+ductile+and+malleable+metal+is &cd=3&hl=ar&ct=clnk&gl=sa&client=firefox-a 1355. In processing an acrylic denture in a water bath, a proper heating cycle is desired because of the possibility of a) warpage. b) shrinkage of the denture. c) porosity due to boiling of the monomer.*** d) crazing of the denture base around necks of the teeth. https://docs.google.com/viewer?a=v&q=cache:xG62SI0KMBMJ:dent.kufau niv.com/teaching/raja%27a/New%2520Folder/Processing%2520the%2520 denture.pdf+In+processing+an+acrylic+denture+in+a+water+bath,+a+prop er+heating+cycle+is+desired+because+of+the+possibility+of&hl=ar&gl=sa &pid=bl&srcid=ADGEESjqq9L2GWyZc5CJlDO90mKIsIZxqWVh2DY7hWWM1 OqHv9VEqk0MHl_wj9OC78UM_X-QLv1GBCTQXEnWY44Tcq4BNj_DV44vLOBsMpdEKJayPmbZCXCJvynmGIJHtExHFg3GQTB &sig=AHIEtbQRTzSHZvC7xHX3vQ7NmlQAVaypbQ page6 ;) 3( )) (433( 1356. In an alginate impression material, tri sodium phosphate is the a) filler.

b) reactor. c) retarder .*** d) accelerator https://docs.google.com/viewer?a=v&q=cache:K7vsQj9Uw1oJ:airforceme dicine.afms.mil/idc/groups/public/documents/afms/ctb_108334.pdf+In+a n+alginate+impression+material,+trisodium+phosphate+is+the&hl=ar&gl= sa&pid=bl&srcid=ADGEESjwMO48dImoJLW_qVgPuudR0HJC0tyofQ6H-TSoncPgUU9R_CEJEJfEYPXdx4ynCIrNx3eaFcJLL6F10LCYptsM0bYrO31EXdPU5c4LoQfENlR90m pcBtM9jSjBgFVtcQQeFo&sig=AHIEtbT1PTIFi8AINPfZJo40LChZy6ukjQ


page 4

1357. The principal function of an indirect retainer is to a) stabilize against lateral movement.*** b) prevent settling of the major connector. c) minimize movement of the base away from supporting tissue . d) restrict tissue ward movement of the distal extension base of the partial denture. http://webcache.googleusercontent.com/search?q=cache:DMl47i8SflYJ:w ww.slideshare.net/shabeelpn/indirectretainer+principal+function+of+an+indirect+retainer+is+to&cd=1&hl=ar&c t=clnk&gl=sa&client=firefox-a 1358. child 8 years old hearing loss , flush around his mouth and notches in incisors and bolbuos molars .what is your diagnosis? a) gardner syndrome b) congenital syphilis 1359. nicotinic stomatiis: a) acanthosis with keratin*** b) brickle cell like shape bases http://webcache.googleusercontent.com/search?q=cache:HXSuOteZ4boJ: en.wikipedia.org/wiki/Stomatitis_nicotina+nicotinic+stomatitis&cd=1&hl= ar&ct=clnk&gl=sa&client=firefox-a
Microscopically, epithelial cells of the palate exhibit signs of hyperkeratosis and acanthosis. There may be metaplasia of excretory ducts, which results in the visible papules if the ducts become hyperplastic.

1360. on radiograph (onion skin) appearance... and under microscope there is glycogen a- osteosarcoma b- pindborg tumor c- ewing sarcoma***

http://webcache.googleusercontent.com/search?q=cache:ygAKSV0aihsJ:w ww.bonetumor.org/tumors-bone/ewingssarcoma+on+radiograph+%28onion+skin%29+appearance...+and+under+ microscope+there+is+glycogen&cd=2&hl=ar&ct=clnk&gl=sa&client=firefox -a 1361. using a larger file while reducing the length in endodontics is called: step back*** http://webcache.googleusercontent.com/search?q=cache:xOLTTilJgAJ:www.maillefer.com/html/filetechniques.html+using+a+larger+file +while+reducing+the+length+in+endodontics&cd=3&hl=ar&ct=clnk&gl=sa &client=firefox-a 1362. What is the name of the instrument used to diagnose halitosis a. Halometer***

http://webcache.googleusercontent.com/search?q=cache:8xhFmsCI2KwJ: www.scielo.br/scielo.php%3Fpid%3DS003472992007000600015%26script%3Dsci_arttext%26tlng%3Den+instument+ used+to+diagnose+halitosis&cd=1&hl=ar&ct=clnk&gl=sa&client=firefox-a

1363. You examined a child and found that the distal part of the upper primary molar is located mesial to the distal outline of the lower primary molar ... This is called a. distal step*** b.miseal step (distal step:Mandibular terminal plane is distal to Maxillary terminal plane\ Mesial step: Mandibular terminal plane is mesial to Maxillary terminal plane)reference:Review the NBDE part 2 section 5 page 150

1364. pt came with prominence in the forehead and the potassium sulfate level increased with curved legs enlarged mandible and maxilla a. Paget disease 1365. what the name of the depressions present on molars in the middle and between the cusps a. developmental grooves 1366. patient had enamel and dentin hypoplasia your ttt: a)Porcelain crowns *** b)splinting with composite c)composite bridge. webcache.googleusercontent.com/search?q=cache:_O5QpFBg3QJ:www.avdsonline.org/info/enamelhypoplasiarest.html+treatment+of+enamel+and+d entin+hypoplasia&cd=3&hl=ar&ct=clnk&gl=sa&client=firefox-a 1367. patient had bulimia and had lesion in palatal surface in upper teeth with recurrent vomiting. What is the type of lesion : a) attrition b) abrasion c) erosion*** http://www.3dmouth.org/2/2_5.cfm 1368. parotitis with purUalant exudate ,what wou will do: A. Immediate coverage with antibiotic 7 days B.delay selection antibiotics until know culture result C.anti viral drug D.give patint wide spectrum antibiotic until result of lab. Culture*** 1369. parotitis with purUalant exudate ,what wou will do: A. Immediate coverage with antibiotic 7 days B.delay selection antibiotics until know culture result C.anti viral drug D. imperialic antibiotic and make culture to know bacteria and antibiotic sensitivity*** http://emedicine.medscape.com/article/882461-treatment 1370. pt come to u needs upper partial denture cII kinnedy classification, he has palatal defect(i don't know what it was but i think torus palatinus) preferable partial denture with: a-horseshoe*** b-palatal bar c-........palatal.......l d-...palatal.... https://docs.google.com/viewer?a=v&q=cache:tsHeKzOuPvgJ:www.dt.ma hidol.ac.th/departments/hospital/rpd%2520design.pdf+indication+of+hors eshoe+in+class+2+kennedy+classification&hl=ar&gl=sa&pid=bl&srcid=ADG EESg9s0JQVnjbNObWNL3v1mm_Xr3oa2kK8IlLzkIEqMq7kal8KiYKZKcmb5u

OETigVZTFtmxRWedQsk1hZfzMSZwXhRAsy47WAYasGf0I1hXqaPDyYndivw QurG4o2r2svTH7vdz&sig=AHIEtbTuDEN9Uu8i4VG8H3jiZkW3XRvJkA&safe=on 1371. In terms of caries prevention, the most effective and most cost effective method is: A- Community based programs. *** b- Private based programs c- Individually based programs. 93 . One-to-one in the clinical environment. This is usually the most successful approach, because the message can be tailored to the individual and reinforcement is facilitated. However, it is expensive in terms of manpower. 1372. One of the main features of acute herpetic gingivostomatitis is the ulcers are confined to the attached gingival and hard palate: a. True. b. False. *** Although most HSV-1 infections are subclinical, a small percentage of patients develop primary herpetic gingivostomatitis.1 Typical symptoms include abrupt onset of fever, anorexia, irritability and intense mouth pain. Patients develop oral lesions of the attached and movable mucosal surfaces in which vesicles develop and quickly break down, coalescing to form large painful ulcerations. The gingivae become erythematous and tender. Acute oral ulcerations -- Treister and Lerman 138 (4): 499 -- The Journal of the American Dental Association
Assessment Findings/Physical Exam
Following an incubation period of 3-7 days (Chandrasekar, 1999; White, 1998), a typical syndrome develops abruptly in healthy children. Fever (up to 103 F); malaise; and bilateral, regionalized lymphadenopathy appear. Cervical, submental, and submaxillary nodes can all be enlarged. White (1998) describes gingivitis followed by the rapid development of fragile vesicles, typically concentrated on the mucosa inside the lower lip (see Figure 1). Vesicles also appear on the gingivae, buccal mucosa, tongue, and hard palate. As underlying epithelial cells are destroyed, the vesicles soon break down into ulcers. Halitosis is present. The oral lesions are very painful and make swallowing, drinking, and eating difficult. The inability to swallow causes excessive saliva to pool in the mouth, and so children drool.

Figure 1. Lesions of Primary Herpetic Gingivostomatitis in a Young Child

(Enlarge Image)

1373. Pt come with bristle even on mucous membrane, u asked for immune test: a. pemphigus b. bullos pemphigoid*** c. lichen planus http://www.medicinenet.com/bullous_pemphigoid/article.htm How is bullous pemphigoid diagnosed?
Bullous pemphigoid is diagnosed based on the results of a biopsy of involved tissue. The biopsy can reveal the abnormal antibodies deposited in the basement membrane layer of skin or mucous membrane tissue

1374. Cheek biting in lower denture can occur if: a. Occlusal plane above tongue. b. Occlusal plane below tongue. c. Occlusal plane at lower lip. d. None of the above. *** Complete Denture 17th Ed Cheek, lip, or tonaue biting: a. Cheek biting is the most common and is mainly due to inadequate overjet. Solution: Increase the overjet by reducing the buccal of the lower posterior teeth. Usually necessary in molar area only. b. Tongue biting -increase the overjet by reducing the lingual of the upper posterior teeth; usually the molars. c. Lip biting is not common and is usually due to poor tooth placement or poor neuromuscular control. . 1375. Permeability of dentine: a. Bacterial product go through it. b. Decrease by smear layer. c. Allow bacteria to go in. d. All of above. ***

1376. Dentin permeability 1- decreases with the increase of cavity preparation. 2- Increase when sclerotic dentin develops under a carious lesion. 3- Increase with smear layer. 4- Bacterial toxins can pass through before the actual penetration of bacteria. *** Art & Science Dentin permeability: increase with the increase of cavity preparation decrease when sclerotic dentin develops under a carious lesion decrease with smear layer 1377. Gracey 13/14 a. Mesial posterior b. Distal posterior c.

) (31- 31

1378. Each of the following is correct EXCEPT which one: a. Bad breath appears to be largely bacteria in origin. b. Bad breath originating from the gastrointestinal tract is quite common. c. Self-perceptions of bad breath appear to be unreliable. d. Fear of having bad breath may be a severe problem for some people.*** ) (

Symptoms
By Mayo Clinic staff Bad breath odors vary depending on the source or the underlying cause of the bad breath. Because it's difficult to rate your own breath, many people worry excessively about their breath even though they have little or no mouth odor. Others have bad breath and don't know it. Because it's difficult to assess how your own breath smells, ask a close friend or relative to confirm your bad-breath questions.

http://www.mayoclinic.com/health/badbreath/DS00025/DSECTION=symptoms 1379. When using the buccal object rule in horizontal angulation, the lingual object in relation to the buccal object: a. Move away from the x-ray tube head. b. Move with the x-ray tube head. ***

c. Move in an inferior direction from the x-ray tube head. d. Move in a superior direction from the x-ray tube head. e. None of the above. Dental Decks - page 72 If the object appears to move in the same direction as the x-ray tube, it is in the lingual aspect. 1380. Radiographic evaluation in extraction: a. Relationship of associated vital structures. B. Root configuration and surrounding bone condition. C. Access to the tooth, crown condition and tooth mobility. D. All of the above e. A & B*** 1381. Radiographic evaluation in extraction EXCEPT: a. Relationship of associated vital structures. B. Root configuration and surrounding bone condition. C. Access to the tooth, crown condition and tooth mobility.*** e. A & B Odontogenic tumors: a. Arise from dental tissue*** b. Can turn malignant but rarely c. Have specific radiographic features d. http://www.ncbi.nlm.nih.gov/pubmed/280645 Abstract
From a total of 54,534 oral biopsy specimens, 706 (1.3%) odontogenic tumors were retrieved and reviewed. Odontomas comprised more than 65% of the odontogenic tumors, ameloblastomas about 10%, and the remaining six categories of odontogenic tumors accounted for approximately 25% of the lesions. The distribution by age, sex, and location of these tumors generally supported the data from other previously reported cases. A possible variant of the calcifying epithelial odontogenic tumor was described, and instances of two granular cell ameloblastic fibromas were reported. The myxomas as a group were characterized histologically more by residual bony trabeculae than by the presence of odontogenic rests. Because the clinical, histological, and behavioral features of the ameloblastic fibroma and ameloblastic fibro-odontoma were similar, these lesions were considered to be essentially the

same. From limited follow-up information, the ameloblastoma was the only lesion that recurred. With the exception of one ameloblastoma found in the lung, no malignant odontogenic tumors were encountered

1382. prophylactic antibiotic needed in .anesthesia not interaligamentary .suture removal .routine tooth brushing .orthodontic band *** "Dental secrets" 1383. Electric pulp tester on the young is not accurate because: a) Late appearance of Fibers A*** b) Late appearance of Fibers C c) Early appearance of fibers A d) Early appearance of fibers C " Pathway Of Pulp 6th edition page 314" The relatively late appearance of A fibers in the pulp helps to explain why the electric pulp test tends to be unreliable in young teeth. ) ( 1384. A patient that had a class II amalgam restoration, next day he returns complaining of discomfort at the site of the restoration, radiographically an Overhanging amalgam is present. This is due to: a) lack of matrix usage. b) improper wedging c) ---- pulpitis

1385. Occlusal splint device: 1/ used during increase vertical dimension 2/ allative muscle of mastication. *** 3/ occlusal plane CR/CO 4/ALL

Occlusal Splints
CONSISTENTLY SUCCESSFUL TMJ TREATMENT
SCDL provide splints and appliances used to alleviate the symptoms of TMJ dysfunction. Most of our splints are used primarily in the first phase of

treatment to restore functional harmony and are fabricated with clear acrylic occlusal pads which allow for easy adjustment as treatment progresses. Our splints are made on either arch and provide skeletal support for the mandible and its musculature.

http://www.scdlab.com/occlusal-splints 1386. Child with late primary dentition has calculus and gingival recession related to upper molar what is the diagnosis: 1. Periodontitis. 2. Local aggressive Periodontitis. 3. Viral infection. Dental Decks - page 776 . 4<-45 1387. Histopathologically, early verrucous carcinomas: a. Have characteristic microscopic features. *** b. Can be confused with acute hypertrophic candidiasis c. Can be confused with Lichen planus d. Can be confused with chronic hypertrophic candidiasis
Background Verrucous carcinoma (VC) refers to a clinicopathologic concept implying a locally aggressive, clinically exophytic, low-grade, slow-growing, welldifferentiated squamous cell carcinoma with minimal metastatic potential. and esophagus.

Verrucous carcinoma typically involves the oral cavity, larynx, genitalia, skin, In 1948, Ackerman first described verrucous carcinoma in the oral cavity as a low-grade tumor that generally is considered a clinicopathologic variant of squamous cell carcinoma.[1] Aird et al first described cutaneous verrucous carcinoma (carcinoma cuniculatum) in 1954, and it was named as such because of its characteristic cryptlike spaces on histologic appearance.[2] http://emedicine.medscape.com/article/1101695-overview

1388. pt came with muliple cyst on his scalp and nick and osteomas on his mandible and multible on his mandible side,, wt is the diagnosis: a. gardner syndrome b. cleidocranial dysplasia c. ectodermal dystosis d. oesteogenesis imperfecta

http://www.rightdiagnosis.com/g/gardner_syndrome/symptom s.htm#symptom_list

Symptoms of Gardner syndrome


The list of signs and symptoms mentioned in various sources for Gardner syndrome includes the 22 symptoms listed below:

Diarrhea Mucus in diarrhea Bloody diarrhea Intestinal polyps Epidermoid cysts Dermoid tumors Fibromas Neurofibromas Benign bone growths Osteomatosis Impacted teeth Extra teeth Retinal pigment epithelium hypertrophy Benign soft tissue tumors Benign bone tumors Polyposis Fibrous dysplasia of the skull Fibrous dysplasia of the lower jaw bone Epithelial cysts Constipation Abdominal pain Weight loss

http://www.rightdiagnosis.com/g/gardner_syndrome/symptom s.htm#symptom_list 1389. Diabatic pt with multiple naevi on the neck and the scalp , and multiple jaw cyst,,,, ur diagnosis will be: a. Eagle syndrome. b. Gorlin Goltz syndrome*** a. Pierre Robin syndrome. d. Non of the above Oxford Handbook 4th

Gorlin-Goltz syndrome ( multiple basal cell neavi syndrome) Introduction


Nevoid basal cell carcinoma syndrome (NBCCS), also known as Gorlin-Goltz syndrome, is an autosomal dominant disorder characterized by a predisposition to neoplasms and other developmental abnormalities [1]. Gorlin&Goltz[2] described the classical triad composed of multiple basal cell carcinoma, keratocysticodontogenictumors (KCOTs) in the jaws and bifid ribs that characterized the diagnosis of this syndrome. In addition to this triad, calcification of the falxcerebri, palmar and plantar epidermal pits, spine and rib anomalies, relative macrocephaly, facial milia, frontal bossing, ocular malformation, medulloblastomas, cleft lip and/or palate, and developmental malformations were also established as features of the syndrome [1,3].

http://www.head-face-med.com/content/7/1/2 1390. pt came to ur clinc have an painless ulcer on the lip , which begin last 6 weeks as elevated border with deep center Ulcer developed very quickly during first 4 weeks then ( slowly growing or stop growing I don`t remember ) have no history of truma but the pt work outside under exposure of the sun. Biobsy reveals PMN , acanthotic exudate ( and something else I`dont remmeber ) , what is the diagnosis: a. sq.cell carcinoma b. keratoacanthoma*** c. verrucus cell carcinoma d. mucoepidermoid carcinoma
Keratoacanthoma (KA) is a common low-grade (unlikely to metastasize or invade) pathologists consider it to be a form of squamous cell carcinoma (SCC).[2] The pathologist often labels KA as "well-differentiated squamous cell carcinoma,

skin tumour that is believed to originate from the neck of the hair follicle.[1] Many

keratoacanthoma variant", because about 6% of KA manifest itself as squamous cell carcinoma when left untreated.[3] KA is commonly found on sun-exposed skin, and often is seen on the face, forearms and hands.[4] The defining characteristic of KA is that it is dome-shaped, symmetrical, surrounded by a smooth wall of inflamed skin, and capped with keratin scales and debris. It always grows rapidly, reaching a large size within days or weeks, and if untreated will starve itself of nourishment, necrose (die), slough, and heal with scarring. enough clinical and histological KA do progress to invasive and aggressive squamous cell cancers, therefore prompt and aggressive treatment is While some pathologists classify KA as a distinct tumor and not a malignancy,

required.[5][6] In reality, the defining characteristics of a KA can not be found when only a small fragment of a large KA is submitted for pathology review. If the keratin debris is submitted, no diagnosis can be made. If a deep core or

peripheral deep wedge is performed, a squamous cell carcinoma is often noted. Only when almost the entire structure is submitted, can a true diagnosis of a KA KA and the pathologic diagnosis of a KA vs a squamous cell cancer. be made. This further complicates the distinction between a clinical diagnosis of a

http://en.wikipedia.org/wiki/Keratoacanthoma 1391. A reline for a complete denture is contraindicated when a) there is extreme over closure of the vertical diamention .*** b) centric occlusion and centric relation do not coincide. c) the denture contains a broken tooth. d) there is resorption of the ridge. https://docs.google.com/viewer?a=v&q=cache:09xcWbOuVRwJ:faculty.ks u.edu.sa/74571/Lecture%2520Notes/Relining%2520and%2520rebasing%2 520of%2520complete%2520dentures.ppt+reline+for+a+complete+denture +is+contraindicated+when&hl=ar&gl=sa&pid=bl&srcid=ADGEESgpYfns4Z1I f2yh6uTVW72gBGoh85si7yFIZVsyqymg1bhMlChbT5p8aoZYhSaO7U1omwjj nCl-_ovANayfArME2TclLlTQYjEt9E3ifjdb6KppudKStHzJWrhQqnM83siBJbP&sig=AHIEt bT7zR0n0yP6sNlulNFqrLmehm-UwA page6
Indications for relining or rebasing: 1. When the residual alveolar ridges have resorbed and the adaptation of the denture bases to the ridges is poor. 2. 3-6 months following the placement of an immediate denture. 3. For geriatric or chronically ill patient because of the long or several appointments required for the construction of a new denture. 4. If the patient cannot afford the cost of having new dentures. 5. Rebasing is additionally required in cases of: a) Porous denture base. b) Discolored or contaminated denture base. Contraindications:

1. If the dentures have poor esthetics or unsatisfactory jaw relationship. 2. If the dentures create a major speech problem. 3. When an excessive amount of resorption has taken place making it difficult to position the denture properly on the ridge. 4. When abused soft tissues are present. The relining is delayed until the tissues recover and return as closely as possible to normal form.

1392. complete re- epithelization after surgery.. after


a. 1-5 days b. 10- 15 days c. 17 21 days I can't remember the exactnumbers

Process

Epithelialization is a complex process of tissue repair consisting of three overlapping phases.

Inflamatory Phase

When the skin is wounded, blood comes into contact with collagen that triggers blood platelets to secrete inflammatory factors. Homeostasis (stopping blood loss) takes place through the clotting cascade. Plasma proteins are released to attract cells that phagocytise (ingest foreign particles). This cellular migration to the wound area is the first line of defense against debris, bacteria, and damaged tissue.

Profliferative Phase

Two to five days after the onset of the wound, patent cells begin to sprout cells (angioblasts) into the wound, forming new capillary loops. Type III collagen is produced forming "granulation tissue."

Remodeling Phase

Remodeling begins when special cells (myofibroblasts) appear. Their muscle-like contracting proteins act on the wound as a whole, shrinking the wound.

1393. A low sag factor in a metal-ceramic FPD and cause 1- Flow of metal under functional load 2- High abrasion resistance 3- Less deformation of bridge spans when fired 4- Poor metal-ceramic bond strength 5- Contamination of porcelain a. 1 only b. 1 and 4 c. 2and 3

d. 3 only*** e. 4 and 5 f. All of the above http://books.google.com.sa/books?id=SdOOEbeyNVkC&pg=PA10&lpg=PA 10&dq=ways+do+surface+characteristics+of+porcelain+affect+the+perceiv ed+from+of+the+final+restoration&source=bl&ots=Avnk7fOnj&sig=jQvaBjOH_3VUoXLaDDV6pB2uHVs&hl=ar&sa=X&ei=qUsxT8jQG9CX Opj7mdsG&ved=0CCsQ6AEwAQ#v=snippet&q=low%20sag%20factor%20& f=false 1394. Nickel-chromium allergic from dentures appear more a)male b)female*** c)equal
Among 564 Danes applying for permanent disability pension due to skin diseases, 222 had allergic contact eczema. Psoriasis was the next most common disorder. Nickel allergy was found in 99 cases, of whom 91 were women with jewellery, suspenders, ect. Besides nickel the patients were often sensitive to cobalt, rubber and formaldehyde. Within Denmark there were geographical hand eczema. In 43 the primary eczema was on the hands, in 38 under costume

differences, as only few applicants came from the rural areas. Ten were atopics. 63 were chrome sensitive, including 21 women who all had hand eczema. Like the nickel sensitive they often had additional allergies to cobalt, rubber and formaldehyde. In 53% of the nickel sensitive the hand eczema started in occupations with a high nickel exposure. 39% had frequent changes in

occupation, often to a new job with occupational nickel contact. As the patients in the present study were all selected on the basis of their poor prognoses, we have tried to establish prognostic criteria through comparison with unselected material from within the same geographical area.

1395. We put the pin very close to line angle because this area -less material of restoration need. -intiate dentin caries. -need less condensation of material. - Great bulk of dentin Art and science of operative dentistry 2000 page 774 - Consideration also must be given to the placement of pins in areas where the greatest bulk of amalgam will occur to minimize the weakening effect of

the pins to tooth structure. Areas of occlusal contacts on the restoration must be anticipated because a pin oriented vertically and positioned directly below an occlusal load weakens the amalgam significantly' Occlusal clearance should be sufficient to provide 2 mm of amalgam over the pin. - Therefore it may be necessary to prepare first a recess in the vertical wall with the No.245 bur to permit proper pinhole preparation, as well as to provide a minimum of 0.5 mm clearance around the circumference of the pin for adequate condensation of amalgam.

5543 1396. Best Root Canal Material primary central incisor: a-iodoform***. b-Guttapercha c-Formacresol. IODOFORM (KRI) PASTE Resorbs rapidly & has no undesirable effects on succedaneous teeth. Material extruded into periapical tissue is rapidly replaced by normal tissue. Has superior antimicrobial action. Does not set into hard mass & can be removed if re-treatment is required. ) ( 1397. HBV disinfection: (enough with intermediate disinfection) 1) iodophors and hypocloride 2) formaldahide 3) dettol / 100% ethyl alcohol 4) ethyl dioxide gas a. 1+2 *** b. 1+2+3 c. 3+4

d. 2+3 http://www.who.int/csr/disease/hepatitis/whocdscsrlyo20022/ en/index2.html HBV is inactivated by exposure to sodium hypochlorite (500 mg free chlorine per litre) for 10 min, 2% aqueous glutaraldehyde at room temperature for 5 min, heat treatment at 988C for 2 min, Sporicidin (Ash Dentsply, York, PA) (pH 7.9), formaldehyde at 18.5 g/l (5% formalin in water), 70% isopropylalcohol, 80% ethyl alcohol at 118C for 2 min, Wescodyne (a iodophor disinfectant, American Sterilizer Co., Erie, PA) diluted 1:213, or combined b-propriolactone and UV irradiation 1398. Filling in RCT must finish a- Exactly the radiographic apex*** b- Few millimeters before apex c- At the half distance between apex and the pulp chamber d- Filling the pulp chamber 1399. Filling in RCT must finish a- Exactly up the radiographic apex*** b- Few millimeters before apex c- At the half distance between apex and the pulp chamber d- Filling the pulp chamber 1400. the organism that not found in newborn mouth: a-streptococcus mutant b-streptococcus salivaris c-e-coli d-skin bacteria 1401. the organism that rarely found in newborn mouth: a-streptococcus mutant b-streptococcus salivaris c-e-coli d-skin bacteria Http://jdr.sagepub.com/content/49/2/415.full.pdf - Breast-fed babies, six months' old, studied by Belding and Belding,6 did not have S salivarius. When diets were supplemented with cereals and sugar, S

salivarius became the dominant organism of saliva. When this supplement was excluded from the diet, S salivarius disappeared entirely. - The absence of S mutans and S sanguis from the mouth of the infants agrees with previous findings that these streptococci require hard surfaces such as teeth and dentures for successful colonization in the mouth.'0"14 Subsequent studies of the infants in the present study will reveal if the occurrence of these streptococci in the mouth is strictly correlated to the eruption of the teeth. And This infection, which generally affects only one breast, usually occurs two to three weeks postpartum, but it might occur after only one week. It's caused by Staphylococcus aureus and Escherichia coli bacteria.1 These bacteria often are carried on the mother's or (hospital) staff's unwashed hands, or in the newborn's mouth.2 The bacteria enter the mother's body through an open, injured area of the nipple, although in some cases there might be no discernible wound. 1402. what can not diagnosised by radiographe a- proximal caries b- abscesses c-severe gingivitis*** d->>>> 1403. initial step to do post and core in RCT tooth is A. remove gutta bercha by hot B. by cleaning cavity of caries and remove old filling C. immediat insertion D. gates glidden drill 1404. How can remove a hard discolored dentine? a. Excavator b.Stanlesstill burs with low speed c. --------Very low speed d. High speed carbide burs*** 1405. class II amalgam restoration with deep caries the patient comes with localized pain related to it after 3 months due to: a)undetected pulp horn exposure b) over occlusion c) moisture contamination during the restoration. d)

: 63-6 1406. class II amalgam restoration with deep caries the patient comes with localized pain related to it after one month due to: a)undetected pulp horn exposure b) over occlusion c) moisture contamination during the restoration. )d .9221 () . -4 *** -5 9-9 .9221 9 -4 *** -5 -6 1409. A child with caries in the incisors we call this caries : a. Rampant caries. 1410. b .Nursing caries***. c. Children caries pt. during routinly cheeck up need preventive tretment pit & fissur ... upon examination the dentist found small caries lesion & he decise to small preparation and do restoration for this pt. these can be called: -pit &f issure seleant -preventive restoration -conservative restoration ?1411. best method to protect teeth after bicuspedization
***Crowning
.9290 .

1413. 8 years child come without complaint while rutine exam you find obliteration of canal in maxillary central incisor what u will do a .Extraction b. RCT c. Pulpotomy d. None of above 1414. All are participating in the determination of the posterior extension of the maxillary denture (posterior palatal extension) EXCEPT: vibrating line hamular notch fovae palatine retromolar (pads) areas. *** 1415. To a great extent, the forces occurring through a removable partial denture can be widely distributed and minimized by the following methods: a. Proper location of the occlusal rests. b. Selection of lingual bar major connector c. Developing balanced occlusion. d. All of the above. . 1416. Which of the following may cause gingival enlargement a. Phenyntoin (Dilantin). *** b. Cyclosporine c. Nifedipine ( a calcium channel blocker) d. Aspirin e. None of the above Dental Decks - page 910 The highest incidence of drug induced hyperplasia is reported to phenytoin (Dialantin) ( Hyperplasia %93-83 %53 ) .Hyperplasia ( Cyclosporine) %63-53) Nifedipine( . 1417. Pt came to u with sublingual space infection ,change in color of mucosa of floor of the mouth. The tongue is stilly elevated haw u will do incision for drainage a. Extra orally parallel to lower border of the mandible

b. Extra orally,,,,,,,,,,,,,,,,,,,,,,,,,< c. Intraorally parallel to wharton's duct d. Intraorally Between mylo-hyoid muscle muscle

1418. Reliability of the measurements reflects that property of the measurements which: : a. Measures what is intended to be measured. b. Produces repeatedly the same results under a variety of conditions. c. Detects reasonably small shifts, I either direction, in group condition. d. All of the above. *** ) ( 1419. You extract tooth with large amalgam restoration, how to manage the extracted tooth: 1. Autoclave and deep buried. 2. Sharp container. 3. Ordinary waste container. 4. Office container. 1420. Polysulfide impression material: a. Should be poured within 1 hour. b. Can be poured after 24 hours. c. Can be poured 6-8 hours. 57 57 ) ; ( 1421. Polysulfide impression material : a. Should be poured within 1 hour . b. Should be poured within 12 hours c. Need a special instrument for>>>> d. Need coolant >>>>>> 1422. Polysulfide rubber base which used for final impression must be : A. Pouring in first 1 hour. B. Pouring in first 2 hour. C. Using cooling water. D. Pouring in first 12 hour.

1423. Type of professionally applied fluoride for mentally retarded pt: 1. Neutral sodium fluoride. 2. Stannous fluoride. 3. Acidulated fluoride solutions. ) ( fluoride varnish 1424. To plan the line-angles in the proximal cavity in a class II you use: A. Straight chisel B. Bin-angled chisel. C. Enamel hatchet d. beveled chisel : diameter round bur "All internal line angles should be rounded to reduce internal stresses. Removing caries with a large Diameter round bur automatically produces the desired shape." 1425. Which of the following burs would you prefer to use preparing a slot for the relation of an extensive amalgam restoration on maxillary molar: a. Number 5 round bur. b. Number 56 fissure bur. c. Number 556 fissure bur. d. Number 35 cone bur. Operative Dentistry A Practical Guide to Recent Innovations page 43 Circumferential grooving is an extension of the amalgapin preparation that involves placing a groove with a No. 33 1/2 inverted cone bur in dentin. 1426. Acyclovir dose for treatments of herpes: a) 200 mg / 5 times a day. Burket- Oral medicine - Cawson Essintials of Oral Pathology b) 200 mg / 4 times a day c) 400 mg / 4 times a day. Dental Decks d) 800 mg / 4 times a day In case of immunodeficiency double the dose to 400 mg Dental Decks: 400 mg / 4 times a day "Cawson Essintials of Oral Pathology and Oral Medicine 7th ed" 533 Burket- Oral medicine page 551 Treatment for HSV-1 infections usually consists of acyclovir (200 mg orally five times daily). 1427. The functions of cement bases are: 1- To act alike a barrier against acids or thermal shocks. -

2- The minimal thickness, which is required, is 0.5 mm of base. a. A ll. b. None of the above. c. 1 only. d. 2 only "Sturdevant's art and science of operative dentistry, 4th edition - page 171" 5-4 or (.) ( ) Bases (cement bases, typically 1 to 2 mm) are used to provide thermal protection for the pulp and to supplement mechanical support for the restoration by distributing local stresses from the restoration across the underlying dentin surface. This mechanical support provides resistance against disruption of thin dentin over the pulp during amalgam condensation procedures or cementation procedures of indirect restorations. Liners are relatively thin layers of material used primarily to provide a barrier to protect the dentin from residual reactants diffusing out of a restoration and/or oral fluids that may penetrate leaky tooth-restoration interfaces. They also contribute initial electrical insulation; generate some thermal protection. 53:; Base Dental Decks - page 2074 . Key property is strength. They can therefore be used in rather thick layers. A thickness exceeding 0.75 mm is needed to achieve thermal insulation.

) non of the above( ) ( 5..0


1428. Early loss of anterior tooth: a. Affect phonetic. b. Affect esthetics. c. Cause space loss. d. A and b. ***

e. All the above. ) ( 1429. Intraosseous cyst in radiograph appears: 1/multiradiolucent may or not expand to cortical bone. 2/radiopaque may or not expand to cortical bone. 3/multiradiolucent may with resorption of cortical bone. 4/radiopaque may with resoption of cortical bone. Central intraosseous ameloblastomas may perforate bone and present a similar pattern.

1430. Type of autoclave used a-hot oven outoclave b-class b autuclave c-class s autoclave d-class d autoclave 1431. Crown with open margin can be due to: a. Putting die space on finishing line. b. Waxing not covering all crown prep. c. Over contouring of crown prevent seating during insertion. d. All of the above. *** ) ( 4; attached gingiva .9220 ... aphthous ulcer recurrent herpes ulcer allergic stomatitis ANUG ) ( 1433. The most desirable finished surface composite resin can be provided by: a. White stones. b. Hand instrument. c. Carbid finishing burs. d. Diamond finish burs. e. Celluloid matrix band. 1434. The best finished composite surface is achieved by:

1. 12 fluted bur. 2. Diamond bur. 3. Matrix band with no additional finish. 1435. Best finishing of composite done by: 1. Carbide bur. 2. Diamond bur. 3. Mounted stone. 4. Best retained under matrix band. The 12-fluted carbide burs (#7901, #7804 ET series) have traditionally been used to perform gross finishing of resin composites. http://www.apad.cc/e-journal/issue2007/daniel.pdf ) 45 ( ) ( 1436. child with 7y old when extrction of lower frist molar the tip of root is fracture in the socket who to manengment: a_ leave it if the mother n't complaine during feeding b -shold be removed by file c-take x-ray to insure if n't swallwed or in respiratory 1437. How many canals can be present in mandibular second molars: a. 1, 2, 3 or 4. b. 2, 3 or 4. c. 3 or 4. d. 3.

%1 %6 ) (%05

1438. The most common immediate treatment reported for fractured teeth was: a. 25%. b. 50%. c. 82%. d. 95%. 1439. What is the proper cavity preparation for V-shaped cervical erosion lesion to be restored with glass ionomer cement:

a. b. c. d.

Cervical groove, incisal groove. Cervical groove, incisal bevel. margin. 4 retention points, 90 No mechanical preparation is necessary. ) ( 1440. If the initial working length film shows the tip of a file to be greater than 1 mm from the ideal location, the clinician should: a. Correct the length and begin instrumentation. b. Move the file to 1 mm short of the ideal length and expose a film. c. Interpolate the variance, correct the position of the stop to this distance, and expose the film. d. Confirm the working length with an apex locator. e. Position the file at the root apex and expose a film. )6 5 4 ( 4

) (
1441. Trauma caused fracture of the root at junction between middle and cervical third: A) do endo for coronal part only** B) RCT for both C) leave** D) extraction. C) Splint the two parts together. Handbook of dental trauma 2001 page 109 Between 20% and 44% of0020root fractured permanent incisors will subsequently lose vitality (Andreasen and Hjorting-Hansen, 1967; Zachrisson and Jacobsen, 1975). :448 . :459 45-; : : . . . :446 Text book of Pediatric Dentistry PAEDIATRIC DENTISTRY - 3rd Ed. (2005) page 343

. . . . 6 : . . . : .

()
1442. Pt with a history of subacute bacterial endocarditis is a medical problem in a surgery because of the possibility of: Bacteremia. Septicemia. Hypertension. Mitral stenosis. Auricular fibrillation. 1- A, b and c. 2- A, b and d. 3- A, d and e. 4- B, c and e. 5- C, d and e. 5 933 5 833 .

a. b. c. d. e.

1443. Blood supply of the palate is from: a. Greater palatine artery. b. Lesser palatine artery. c. Facial artery. d. Long sphenopalatine artery. e. Anatomising braches from all of the above EXCEPT c *branches of the facial artery in the neck: ascending palatine>>>>supply soft palate. *branches of the maxillary artery:

The third part of maxillary artery gives branches which correspond to the branches of the maxillary nerve and the branches of the spheno-palatine ganglion. *Branches of the sheno-palatine ganglion: -greater palatine..supply hard palate. -lesser palatinesupply soft palate. -long spheno-palatine.anterior part of hard palate The blood supply of the palate is provided anteriorly through the incisor foramen and posteriorly through the great palatine foramen where the great palatine artery emerges. The blood supply of the palate is from ascending branches of the facial artery as well as from the branch of the maxillary artery, The palatine vessels and nerves pass through the palatine canal. The greater palatine artery was the main vessel to supply the hard palate and the ascending palatine artery provided the principal supply of soft palate. The branches of greater palatine artery formed abundant anastomoses with the neighbor vessels. 1444. Epithelial cells a. Rest of malassez decrease with age. b. Rest of malassez increase with age c. Hertwig sheath entirely disappear after dentinogenesis. d. Epithelial remnants could proliferate to periapical granuloma 1445. Formation of lateral periodontal cyst due to a-Nasolacrimal cyst b-Hertwig's epithelial root sheath c-Epithelial rest of maLassaz d-The epithelial rests or glands of Serres 1446. the type of cement wich give retention to crown a-zn phosphate b-zn polycarpoxylate c-resin d-resin modified glass ionomer ZN POLYCARPOXYLATE CEMENT NOT ADHERE WELL TO GOLD &PORCELAIN BUT STRONG ADHSVE BOND TO BASE METAL . ) (

1447. Ethics of the study include all of the following EXCEPT: a. Privacy of all subjects. b. Informed consent may be required or not. c. Object if the subject refuse to take part of the study. () 1448. Clinical research: a. No different between blind & double blind. b. If there's need of intervention. () 1449. X- ray show large pulp chamber, thin dentine layer and enamel: a- Dentogerous imperfect. b- Dentine dysplasia. . Dentogerous imperfect Dentine dysplasia imperfect Dentogerous Dentine dysplasia Dental Decks - page 998 . 6 . http://en.wikipedia.org/wiki/Dentinogenesis_imperfecta Type I and II show total obliteration of the pulp chamber. Type III shows thin dentin and extremely enormous pulp chamber.These teeth are usually known as Shell Teeth. http://en.wikipedia.org/wiki/Dentin_dysplasia Type 1: Roots are short, blunt and conical. In deciduous teeth, pulp chambers and root canals are completely obliterated in permanent they may be crescent shaped.

Type 2: The pulp chamber of the deciduous teeth become obliterated in deciduous teeth. While in permanent teeth, large pulp chamber is seen in coronal portion of the tooth - referred to as thistle tube appearance.Pulp stones may be found. a. b. 1450. Provisional luting cement: Prevent restoration from dislodgement. *** Sealing Planning and Making Crown and Bridges page 134 . 1451. Maryland bridge: Use with young. To replace single missing tooth. *** ) ( 1452. Composite resto followup after 2 years showed stained margin: a. Stress from polymerization shrinkage b. Hydrolic destruction on bond c. .... Marginal Leakage or micro leakage Art & Science 1453. high copper amalgam prevent a. marginal leakage 1454. Digital radiography is a technique that shows transition from white to black. Its main advantage is the ability to manipulate the image by computer a. 1st T, 2nd F b. 1st F, 2nd T c. Both T d. Both F : . 1455. The imaging showing disk position and morphology and TMJ bone: a. MRI. b. CT c. ARTHROGRAPHY d. Plain radiograph e. Plain tomography Dental secrets - 107

a. b.

MRI is better at imaging the soft tissue of the disk, but CT is better for almost all other investiagions of the TMJ. 1456. Enamel a. Repair by ameloblasts b. Permeability reduce with age c. Permeability increase with age d. Permeable to some ions art & scince Although enamel is a very hard, dense structure it is permeable to certain ions and molecules permitting both partial and complete penetration. The route of passage appears to be through structural units that are hypomineralized and rich in organic content, such as rod an important role as a transporting medium through small intercrystalline spaces Enamel permeability decreases with age because of changes in the ,enamel matrix though basic permeability is maintained; this decrease .is referred to as enamel maturation b,d b 1457. Upper teeth palatal mucosa supplied by: a. Nasopalatine b. Anterior palatine c. Both *** d. Post superior alveolar nerve ) ( : : . 1458. Indirect retainers mostly needed: a. Class VI b. Class I c. Class III d. Class III with modification

) ( 1459. The difference between cellulitis and abscess: a. Cellulitis acute stage with diffuse swelling no pus b. .. Cellulitis develops as the infection spreads from the bone to the adjacent . soft tissue. Subsequently inflammation and edema occur, and the patient develops a poorly localized swelling. On palpation the area is often sensitive, but the sensitivity is not discrete 1460. x-ray periapical for immature tooth is .generally conclusive .simply inconculosive .should be compered with antermere 1461. child patient take sedation before appointment and present with physical volt. What should dentist do .conscious sedation .redo sedation .tie with baboose board .tie in unite with bandage . ( ) 1462. caries detection dye composed main of: a- 5% acid fuschin b- 5% basic fuchsin c- propylene glycol. *** 6 5 "Paediatric Dentistry 3rd Ed (2005)" page 165: 0.5% basic fuchsin And: "Dental pulp 2002" + "Operative Dentistry" propylene glycol 1463. when increase vertical dimension you have to: 1/ increase minimal need 2/construct anterior teeth first then posterior teeth 3/ use provisional crown for 2 months 4/all 1464. Treacher Collins syndrome is mainly: 1/ mandibular retrognathia. 2/ loss of hearing (50% of cases)

( ) Oxford handbook of clinical dentistry )) 1465. A patient that had a class II amalgam restoration, next day he returns complaining of discomfort at the site of the restoration, radiographically an Overhanging amalgam is present. This is due to: a) lack of matrix usage. B) no burnishing for amalgam insufficient carving : Dental Decks - page 2300 ) ( 1466. Contents of the Anaesthia carpule: a)Lidocaine + epinephrine + Ringers liquid. B)Lidocaine + epinephrine + distilled water. C) Lidocaine + epinephrine only. ) ( "Hand Book of Local Anesthesia"page 92 : ( ) : Local anesthetic drug vasopressor - Ringers liquid (nacl) distilled water - preservative substance (methylparaben) preservative for vasopressor http://faculty.ksu.edu.sa/hkhalil/Do...c%20agents.pdf 1467. Distal fissure of premolar contact oppose: a- Middle of the middle third & buccal fissure is wider than lingual b- Cervical line & lingual fissure is wider than buccal c- Middle of the middle third & vice versa d- Cervical of the middle third & vice versa 1468. You make ledge in the canal. you want to correct this. what is the most complication occur in this step: a. Creation false canal b. apical zip c. stripping d. perforation ) (

1469. To prevent gingival injury place the margin of the retainer: A. At the level of gingival crest. b. Above gingival crest. C. Apical to g . Crest 1 mm d. Apical to g. Crest 0.5 mm. Fundamentals of Fixed Prosthodontics 3rd Edition page 132 Preservation of the Periodontium Dental Decks - page 458 - 466 3.8 Dental secrets page 219 It is better for gingival health to place a crown margin supragingivally, 1 2 mm above the gingival crest, or equigingivally at the gingival crest. Such positioning is quite often not possible because of esthetic or caries considerations. Subsequently, the margin must be placed subgingivally. Margin ends slightly below the gingival crest, in the middle of the sulcular depth, or at the base of the sulcus. In preparing a subgingival margin, the major concern is not to extend the preparation into the attachment apparatus. If the margin gf the subsequent crown is extended into the attachment apparatus, a constant gingival irritant has been constructed. Therefore, for clinical simplicity, when a margin is to be placed subgingivally. It is desirable to end the tooth preparation slightly below the gingival crest. ( ) . . ) (

1470. A completely edentulous patient, the dentist delivers a denture in the 1st day normally, 2nd day the patient returns unable to wear the denture again, the cause is: a) Lack of Skill of the patient b) Lack of Frenum areas of the Complete denture.

) ( 1471. How can you alter the sitting time for alginate a)alter ratio powder water b)alter water ratio c)we cant alter it d)by accelerated addition Dental decks page 672 The best method to control the gelation time (setting time) is altering the temperature of the water used in the mix. Changing the water/powder ratio also alter the gelation time but these method also impair certain properties of the material. Too little or too much water will weaken the gel. Undermixing may prevent the chemical action from occuring evenly ;overmixing may break up the gel ) ( 1472. How can you alter the sitting time for alginate a)alter ratio powder water b)alter water temperature c)we cant alter it d)by accelerated addition 1473. Whats the reason of the wax shrinkage upon fabrication of the bridge/crown SOLDIFICATION SHRINKAGE 1474. patient who has un-modified class II kennedy classification, with good periodontal condition and no carious lesion the best clasp to use on the other side <teeth side> a) reciprocal clasp (aker's clasp). *** b) ring clasp c) embrasure clasp d) gingivally approaching clasp
An Akers' clasp is the classic direct retainer for removable partial dentures. Named after its inventor, Polk E. Akers, this suprabulge clasp consists of a rest, a guide plate, a retentive arm and a reciprocal arm. Akers' clasps, as a rule, face away from an edentulous area. Should they face the edentulous area, they are termed reverse Akers' clasps

http://en.wikipedia.org/wiki/Akers'_clasp

1475. After patient with Hepatitis-B left the dental chair the assistant ask you how to disinfect the dental unit. What will you suggest: 1. Iodole. 2. 100%dittole. 3. Hypo chloride. 4. gas a) 1 and 2 b) 1, 2 and 4. C) x d) x ) ( 1476. Pt with hepatitis B the best sterilization is a)formaldahid**** *****b)detol+100%alkohol c). D).. 1)a+b 2)a+b+c 3) 4). 1477. You should treat ANUG until the disease completely removed. 2. Otherwise, it will change to necrotic ulcerative gingivitis. A) Both sentences are true. B) Both sentences are false. C) 1st true, 2nd false. D) 1st false, 2nd true. 574 AUG CUG Chronic necrotic ulcerative gingivitis : Burket- Oral medicine page 63 The patient must be made aware that, unless the local etiologic factors of the disease are removed, ANUG may return or become chronic and lead to periodontal disease. NUG necrotic ulcerative gingivitis ( ) ) ( 1478. Studies show that Complete Remineralization of surface of an accidentally etched enamel:

a- never occur b-after hours c-after weeks d- after months :4 Oxford Handbook of Clinical Dentistry - 4th Ed. (2005) page 355 Remineralization of etched enamel occurs from the saliva, and after 24 h it is indistinguishable from untreated enamel. :5 6-5 . 57 ( ) 1479. Pontics are classified according to their surface toward the ridge of the missing tooth , ............................... A-Both statment are true b-both are false c-1st is true ,2nd is false d-1st false , 2nd true () 1480. wax properties are: 1/ expansion 2/ internal stress 1481. inlay wax invest fast because flow and quick deformaty this due to a. relaxation of internal stress : : . () 1482. Inlay waxs must invested fast because of flow and quickly deformity of dimension this property due to

a. slow flow b. Internal stress 1483. Teenager boy with occlusal wear the best ttt is : 1/ remove the occlusal 2/ teeth capping 3/ restoration ) ( 1484. In FPD in upper posterior teeth we should have gingival embrasure space to have healthy gingival so the contact: a- in the middle b- depend in the opposing occlusion c- Occlusally As far as you can 1485. Complex amalgam restoration when to do it: -weak of the cusp with undermine enamel. -bevel and contra bevel -weak cusp should strengthen it by resin 1486. Most important sealer criteria to be success: -high viscosity -high retention -high strength -can add colorant -High resilience ) ( 1487. Discoloration of endo treated teeth: a -hemorrhage after trauma b -incomplete remove GP from the pulp chamber c - Incomplete removal of pulp tissue Pocket Atlas of Endodontics page 178 In order to prevent discoloration of the tooth crown by components of the root canal filling material, a heated instrument must be used to sever the filling material 2mm apical to the cementoenamel junction. Pocket Atlas of Endodontics page 88 It must be sufficiently extended mesially and distally so that the pulp horns

can be completely accessed and all necrotic tissue removed. Tissues that are left behind can lead later on to discoloration of the clinical crown. Pathway of the pulp 9ed page 231 The access cavity is positioned too far to the gingival with no incisal extension. This can lead to bur and file breakage, coronal discoloration because the pulp horns remain. 1488. Bacteria release from bacteria in endo canal..: -from dentin bacteriods ?? - .. 1489. Balance occlusion should be utilize in natural dentition. & may all or some of the teeth contact in both side regardless where mandible move. -.1st true & 2nd false - 1st false & 2nd true - both false - both true ) ( 1490. Patient came to your clinic with dull pain in the #6 ,no response to the pulp tester, in radiographs it shows 3mm of radiolucency at the apex of the root Diagnosis is a)chronic apical periodontitis b) acute apical periodontitis c)acute periodontitis with abscess d) ) ( chronic apical abscess 1491. The nasopalatine bone forms a triangle will be parallel to an imaginary lines extended between cemento-enamel junctions of adjacent teeth a. True b.False 1492. 7 years patient come with untreared truma to tooth that became yellow in colour what you should tell the parents a: pulp is dead b: pulp become calcified c: the tooth will absorb normally 1: a and b 2: a and c 3" all of the above

1493. the needle holder used in suturing of lower third molar curved hemostat. allis forceps Adson forceps Regular tweezers. ) ( 1494. instrument which use for grasping a tissues when remove thick epulis figuratum: a- Allis forceps*** b- Adson forcep c- Curved hemostat d- Stilli forceps Describe and discuss the function of Allis forceps in oral surgery. Allis forceps have a locking handle similar to a needle holder and small beaks at the working end of the instruments. These beaks are useful in grasping tissue for removal Adson forceps: These forceps are non-toothed, and so are ideal for holding delicate tissues Allis forceps: These forceps are used to hold soft tissues for a long period while minimising tissue damage. Using the ratchet they can be locked on to tissue such as bowel and can be used to provide gentle traction. 1495. the favored relationship in case of fabrication of a lower class 1 RPD opposing a natural dentition is 1- prognathism 2- working side 3- balancing side 4- none of the above Regardless of the method used in creating a harmonious functional occlusion, an evaluation of the existing relationships of the opposing natural teeth must be made and is accomplished with a diagnostic mounting. This evaluation is in addition to, and in conjunction with, other diagnostic procedures that contribute to an adequate diagnosis and treatment plan.

Diagnostic casts provide an opportunity to evaluate the relationship of remaining oral structures when correctly mounted on a semiadjustable articulator by use of a face-bow transfer and interocclusal records. Diagnostic casts are mounted in centric relation (most retruded relation of the mandible to the maxillae) so that deflective occlusal contacts can be correlated with those observed in the mouth. Deflective contacts of opposing teeth are usually destructive to the supporting structures involved and should be eliminated. 1496. baby born without which bacteria: A) Streptococcus mutans. *** 1497. for a newly erupted tooth, the most bacteria found around the tooth is A) Streptococcus mutans. *** B) Streptococcus salivaris Dental Decks page 774 ) ( Streptococcus mutans 1498. pt with renal transplantation came with white elevated lesion on tongue no history of smoking or tobacco chewing diagnosis is: a-candidiasis B- iatrogenic lesion. c- hyperkeratosis D- stomatitis Pocket atlas of oral diseases Uremic Stomatitis Definition Uremic stomatitis is a rare disorder that may occur in patients with acute or chronic renal failure no smoking 1499. patient with 5 years old denture has a severe gag reflex , upon history he says he had the same symptoms in the first few days of the denture delievery and it went all alone a. patient has severe gag reflex b. patient has underlying systemic condition. c. denture is overextended .

Bone resorpion of the lower ridge lead to overextension of the distolingual area of the lower denture (palatoglossus muscle)

1500. Immature tooth has less sensation of cold hot due to: 1. Short root. 2. Incomplete innervations. 3. Wide pulp chamber. 1501. Electric pulp tester on the young is not accurate because: 1. Short root. 2. Incomplete innervations. 3. Wide pulp chamber.

1502. Completed in centric occlusion is normal but in eccentric occlusion the lower ant teeth & upper ant are interfere with contact wt should be do: 1/reduction of mand incisor 2/'' '' max '' 3/reduction of lingual inclination of max incisior 4/'' '' '' '' '' '' mand '' 1503. pt have premature contact on his ((max) I guess not sure) canine during his centric occlusion & on the eccentric occlusion there is no abnormality , So the treatment is: a. retroclination of the max ant teeth. - b. retroclination of the man ant teeth. -c. grinding of the max ant teeth. - d. grinding of the man ant teeth. 1504. Prematur contact btw upper and lower ant teeth in eccentric occlusion while there is absolutely no contact on the centric occlusion. So the management is by grinding of: - incisal edge of ant max teeth. - Incisal edge of ant man teeth. - Inclination of ant max teeth lingual - Inclination of ant man teeth.

1505. Instrument used to remove dark color in dentin: Round stone bur w low speed Round diamond bur w low speed Large excavator. Carbide bur with high speed 1506. patient feel sever pain upper mouth pain is radiated to eye and ear ,after you check no caries when you pressure on maxilry premolar he feel pain. In xray no change what dignosis? a. Acute apical periodontits b. maxilary sinusitis. c. canine space infection d. dentoalveolar infection In other cases, one tooth may be obviously at fault, but the patient is feeling pain in his ear, eye, temple, or in teeth in the) opposite arch as well as in the obviously affected tooth. Finally, you get the really weird cases in which pain actually coming from, say, a top front tooth is felt in a back bottom tooth. This actually happened to me when a patient appeared with pain in a lone standing lower back tooth (a molar). The molar had no cavities or fillings, and did not react badly to cold air or tapping. I sent the patient home, and she returned the next day with an abscess in a top central front tooth :The classic symptoms of acute (short lasting) sinusitis are fever nasal obstruction raspy voice pus-like (purulent) nasal discharge loss of sense of smell facial pain or headache that is sometimes aggravated by bending over (When .pain is present, this may suggest which sinus is affected ) ( 1507. class III crown fracture in child patient the type of pontic a.ovate. b.egg shaped c.hygienic d.ridge lap

1508. What is the test name for detecting the virulent of (bacteria i do not remember the name may be spirochete) a-hemolysis..... B-catalase 1509. Apiceoctomy what is the right statement: 1. Incisor with an adequate RCT and 9mm lesion. 2. Lateral incisor with good condensing RCT but swelling and pain 14 day after the treatment, the tooth asymptom before the obturation. 3. First upper premolar with lesion on the buccal root.. ) ( 1510. Which will design first in the study cast of RPD with a lingual bar major connector: A. The lower border of lingual bar major connector B. the upper border of lingual bar C. indirect rest and rest seat. *** . 1511. Naocl is used in RCT: A. Oxidative effect. B. irrigant solution of choice . C. efficacy increasing with diluting . D. Better result when used combined with alcohol. ) ( Oxford Handbook of Clinical Dentistry, 4th Edition page 172 Dilute sodium hypochlorite is generally considered to be the best irrigant as it is bacteriocidal and dissolves organic debris. 1512. Receiving the impression after removal from the mouth directly: It must be disinfected immediately. It must be poured immediately. It must be mounted immediately. It must be left for minutes. 57 ) ( 1513. Retentive grooves:

1. 2. 3. 4.

1. Always axiobuccal and axiolingual. 2. Prevent lateral displacement of restoration. 3. Is axiopulpal and axiogingival. ) ( 1514. Tooth number 26, had a root canal treatment since two years, upon xray you found a radiolucency with bone resorption along one of the roots. a. Ca(OH)2. b. resection of the whole root. c. redo RCT. d. periodontal currettage. ( ) ) ( 1515. if you do mouth wash by 10% glucose , the P H can be read from the Carve: 1516. pt have trauma in upper central incisor the tooth and the alveolar bone is move as one piece, in examination intraorally x ray you will see : a. gap between the apicx of root and alveolar bone b. definite line of fracture c. no apear in x_ray 1517. Yellow-brown hypomineralization of enamel with or without hypoplasia Acid-pumice microabrasion 1518. -3months baby had black-blue discolured rapid growing swelling, the xray show unilocular radio...... and displaced tooth bud, is it: a. aneurysmal bone cystb. melanoma......tumorc. ameloblastic fibroma.......................) ( 1519. what first treatment of thump suking? a. consult b. rewarder therpy 1520. A 7 years child has a habit of finger sucking what is the best way to start a therapy with a. Rewarding therapy b. Counseling therapy

c. Punishment d. Remaindering therapy 1521. glass ionomer: a) introduction 1970 b) need dry field when application c) both d) none of the above http://webcache.googleusercontent.com/search?q=cache:pxaK5Gg-7YJ:en.wikipedia.org/wiki/Glass_ionomer_cement+glass+ionomer+introdu ce&cd=1&hl=ar&ct=clnk&gl=sa&client=firefox-a 4<:5 )( 1522. 7 yrs old pt thumb sucking, what is the management a- Rewarding system b- Counseling therapy c- adjunctive therapy d- Nothing 1523. pacifier habit what you see in his mouth 7 year old pt a. anterior open bite with expansion max bone b. anterior open bite, posterior crossbite *** c. move incsial to labial Dentistry for child and adolescent Children who were pacifier users were significantly more likely to show open bite, posterior crossbite increased overjet, and alteration in cheek mobility than habit-free children action during sucking, which leads to gingival injury recession, and loss of alveolar bone 1524. which one of the conditions would delay a dentist's decision of taking full mouth X-ray examination a. pregnancy b. patient had full mouth examination by X-ray 6 months ago c. patient will receive radiotherapy next week d. patient had CT examination last week if the patient is pregnant no contraindication to make x-ray examination but is the .full mouth examination necessary for diagnosis of severe pain or not second answer depends on the patient's caries index, is the patient has high

.risk of caries extension or not the third one is not important at all because no contraindication for full mouth .examination if the patient had a previous history of radiotherapy the forth one is probable if the diagnosis of is concerned with examination for lesions but if the patient's suffer from generalized periodontitis so periapical radiographs are sufficient to make a diagnosis. So the question you ask about .is wrong in its formate with great sincerity El-SAHERBINI { 1525. Pt has bad oral hygine and missing the right and left lateral insicor what ttt 1_implant 2_rpd 3_conventional fpd 4_Marylad bridge. "Summery of Operative dentistry page 44" Small occlusal forces (adhesive bridges in bruxist patients or when replacing maxillary canine have poor results) Intermediate restoration. Missing lateral incisors. Virgin abutment teeth. Favorable occlusal scheme (deep overbite unfavourable, Class III occlusion favourable) Splinting teeth. ( ) ).( http://www.dentalfind.com/info/maryland-bridge ) ( 1526. The narrowest canal found in a three root maxillary first molar is the: Mesio-buccal canal. Disto-buccal canal. Palatal canal. Disto-palatal canal. Mesio-palatal canal.

1. 2. 3. 4. 5.

) ( 1527. which part of root canal diameter THE SMALLEST a. radigraphical apex b. apical foramin c. apical construction 1528. To record the vertical dimension in order to: a. To determine the amount of space between the mandible and the maxilla which will be occupied by an artificial teeth. b. To determine vertical and horizontal level of the teeth. c. A and B. d. None. By definition, vertical dimension of occlusion is the distance between the mandible and maxilla when the opposing teeth are in contact ) . ( 1529. radial walker test for disinfection 1530. TB patient in active stage (sputum) when we de treatment : a. Emergency case b. With rupper dam c. With mask d. Postpone the treatment 1531. when root perforation we close it by -A G.I -B ca oh -C silver point -D composite ) (

1532. 1234-

Irrigation solution for RCT cause protein coagulation is: Sodium hypochlorite. Iodine potassium. Formocresol. None of the above.

) . ( .9122 -4 ; -7 -5 57-45 -6 -7 .9122 5-4 -4 6-5- -5 7-6- -6 ;-9- -7 1535. Selection of shade for porcelain is done EXCEPT: a) before preparation b) we must rest the eye by looking to a yellow color. c) we must look to the tooth only 5 sec 1536. 6 year child in routine examination .explorer wedges in the pit of 2nd molar other teeth free dental caries what is the management a-fluoride gel application b-fissure sealant c-restore it with amalgam restoration d-restore it with composite restoration 1537. the treatment of Weeping canals.is a-ca oh b-formocresol 1538. parotid malignancy in perinural is seen as a. Warthon.s path b. Ductal papilla c. Polymorphic adenoma d. Adenoid cystic carcinoma ) (5 1539. we want to do a maxillary PD to a patient using anterior-posterior strap...we want it to be rigid ,,we use a- gold b- cast gold

c- co-cr 1540. which of the fllowing event occurs during dentinogenesis a. odontoblasts become long cubooidal b. the matrix and proteoglycans maturate with collagen fibers .. (cant remember it precisely but i hade chosen it ) (
http://en.wikipedia.org/wiki/Dentinogenesis Dentinogenesis is the formation of dentin, a substance that forms the majority of teeth. Dentinogenesis is performed by odontoblasts, which are a special type of biological cells on the outside of dental pulps, and it begins at the late bell stage of a developing tooth. The different stages of dentin formation result in different types of dentin: mantle dentin, primary dentin, secondary dentin, and tertiary dentin.

Odontoblasts differentiate from cells of the dental papilla. They begin secreting an organic matrix around the area directly adjacent to the inner enamel epithelium, closest to the area of the future cusp of a tooth. The organic matrix contains collagen fibers with large diameters (0.1-0.2 m in diameter). The odontoblasts begin to move toward the center of the tooth, forming an extension called the odontoblast process. Thus, dentin formation proceeds toward the inside of the tooth. The odontoblast process causes the secretion of hydroxyapatite crystals and mineralization of the about 5-30 m thick.(Linde& Goldberg 1993)

matrix. This area of mineralization is known as mantle dentin and is a layer usually

1541. what is the form of local anesthesia when enters the NERVE tissues : a- lipid soluble ionized form b- lipid soluble non-ionized c- water soluble ionized d- water soluble nonionized 1542. The degree of taper for crown prep: a.3-5 b.15 : .9122 : ; .9122 - - -

- .9121 .9121 -4 -5 -6 -7 ) ( : ( ) enucleation .9121 .9121 1549. patint with haemophelia what is the additional anaesthetic procedure used with him? 1550. What is the material whih we use after apex ectomy ? a. titric acid b. EDTA c. Tetracycline 1551. temprature that damage the bone during implant procedure 1552. attaust patient mean the patient complain after dental treatment from a. the dentist had abused the patient while treatment b. the patient fill sicological stress while treatment 1553. hemiseptal fracture a. horizontal recession-one wall fracture b. horizontal recession- two walls fracture c. horizontal recession- three walls fracture 1554. franklene appliance used in a. prevent oral habit b. anterior cross bite 1555. maximum porcelain thickness 1.5mm 1556. High mylohyoid crest in patient for complete dnture the surgeon must avoid vital strcture which is a. lingual nerve

1557. in the normal tissues the the basal cell layer adhere to 1558. Three years old pt came to clinic with his parents he has asymptomatic swelling blush incolor fluctant in midline of palatal raph diagnosis is : a- bohn's noduls b- herps semplex virus c- lymphepithelial cyst d- gingivalcyst : Bohn's nodules are smooth whitish bumps or cysts which are sometime found in the mouths of newborns. They are found at the junction of the hard and soft palate, and along lingual and buccal parts of the dental ridges, away from the midline. These nodules are 1-3 mm in size, and filled with keratin. ........ 1559. Mandibular foramen is: -above occlusal plane in elderly people -at the occlusal plane inadult -below the occlusal plane in children. - all of the above 1560. Dentine etching often takes: a. 15 sec b.30 sec c.45 sec d. 60 sec 48 ) ( 1561. Preparation to small occlusal cavity to premolar the width of cavity is : a. 1/4 inter cuspal distance b. 1/2 inter cuspal distance c. 1/5 inter cuspal distance d. 3/5 inter cuspal distance )( 1562. pt of HBV came to ur clinic and during dental procedures have a sever Injury and bleeds alot , what is ur management: 1. squeeze the wound but don`t scrub 2. wash the wound with water and put waterproofe plaster 3. asset the virulunes of the pt and refer him for infectous disease consltant 4. ask him to apply pressure on the wound to stop bleeding

a. b. c. d.

1+2+3 1+4 1+ 2+4 3+4

1563. For best results when using an elastic impression material the prepared tooth should be a-Very dry b-Free of surface moisture c-Covered thinly with water soluble lubricant d-Covered with a surface tension reducing agent and air dried e-All of the above 1564. Tissue displacement for making an impression is improved by 1. Removing sufficient tooth substance subgingivally 2. Improving the health of the gingival tissue before the preparation 3. Using reversible hydrocolloid rather than polysulphide rubber silicon or polyether impression material a. 1+2 b. 1+3 c. 2+3 d. All of the above 1565. Ring liner is used as a lining in a casting to a- Insulate against the thermal conductivity b- Allow for expansion of the investment c- Prevent fracture of the investment during heating d- Facilitate removal of the investment after casting e- All of the above 1566. Heating gypsum casting investments above 13000F in the presence of carbon results in a-Fine grain size b-Shrinkage porosity c-Oxidation of castings d-Sulfur gases being released 1567. In articulator , incisal guidance represent: a-Horizontal guidance b-Condylar guidance c- Equivalent of horizontal and vertical overlap

1568. The most accurate impression least distortion if poured after 24 hours a-Polyether b-Silicone c-Reversible hydrocolloids in humidor d-Compound impression 1569. Post should set passively in root canal and crown should set with slight resistance a-Both statement are false b-Both statement are true c-First statement is true and second is false d-First statement is false and seconds statement is true 1570. In the metal-ceramic technique , the bond between porcelain and the noble-metal alloy dependent on the 1- Proper matching of coefficients of thermal expansion of metal and porelain 2- Formation of base-metal oxides 3- Formation of noble-metal oxides 4- Surface roughness of the alloy a. 1 only b. 1 and 2 c. 1and 3 d. 2 and 4 e. 2 and 4 1571. In casting the substructure for a metal ceramic restoration it is necessary to use 1- Gypsum -bonded investment 2- Phosphate- bonded investment 3- Gas-air torch 4- Crucible which has not been used for other gold alloy a. 1 and 3 b. 1 and 4 c. 2only d. 2and 3 e. 2 and 4 1572. In order to maintain the health of the tissue beneath a pontic, it is desirable to 1- Scrape the ridge area on the cast and use gold for the ridge contact 2- Scrape the ridge area on the cast and use porcelain for the ridge contact

3- Have passive contact with ridge tissue 1 when the restoration is placed in the mouth 4- Have slight blanching of the ridge tissue when the restoration in the mouth 5- Have minimal tissue coverage a. 1and 4 b. 1and 5 c. 2 and 3 d. 2and 4 e. 3and 5 f. 4 and 5 1573. The most frequent cause of porosity in a porcelain restoration is a- Moisture contamination b- Excessive firing temperature c- Excessive condensation of the porcelain d- Inadequate condensation of the porcelain 1574. Cobalt-chromium alloys, the constituent responsible for corrosion resistance is a-Silver b-Nickel C-Cobalt d-Chromium e-Tungsten 1575. Three weeks after insertion FPD marked discomfort to heat and cold occurs there are no other symptoms. The most likely cause is a- Gingival recession b- Unseating of the FPD c- Deflective occlusal contact d- Torsional forces on one abutment tooth e-Incomplete coverage of cut surfaces of prepared aboutment teeth 1576. The most frequent cause of failure of a cast crown restoration is a-Failure to extend the crown preparation adequately into the gingival sulcus b-Lack of attention in carving occlusal anatomy of the tooth c-Lack of attention to tooth shape, position and contacts d-Lack of prominent cusps deep sulcus and sharp marginal ridge 1577. A permanently cemented FPD may become loose because of: a- insufficient retention in the abutment preparation

b- deformation of the metal casting on the abutment c- lack of embrasure space d-torque e-passive contact of the pontic to the ridge lap a, b a, b ,d b, d ,e c, d ,e d, e 1578. from properly mounted diagnostic casts ,determination can made for fixed prosthodontic treatment planning concerning a-axial alignment of the abutment teeth b- physical condition of the abutment teeth c- gingival tissue contour and pocket depth d- tentative design of abutment preparation a only a ,b. c a, b, d b ,c, d all of the above 1579. which following condition in a FPD could cause porcelain bonded to metal to loosen a-removal of 0.7 mm of bulk from the facial surface of the abutment teeth b-contamination of the metal framework c-baking the facing too rapidly d-insufficient mechanical locks a,b a,b,c b,c b,c,d d only all of the above 1580. A patient is missing a mandibular first molar The maxillary molar has extruded approximately 2.8 mm into the space periodontally is acceptable. Restoring satisfactory occlusion with FPD will require a-extraction of maxillary molar b- reducing and restoring the maxillary molar to normal occlusal plane

c-reducing and polishing the maxillary molar to normal occlusal plane d-none of the above.no ttt indication 1581. which the following applies when selecting a shade for metal-ceramic rstoration a-it is a better select a shade with too low a value than too high a value if staining to be improve the match b-it is a better to select a shade with too high a value than too low a value if staining is not to be used to improve match c- the basic shade selected should be that of the middle third of tooth to be matched d-none of the above 1582. cement producing mechanicl bond with gold alloy A- GIC B- Zinc phosphate cement C- Zinc polycarboxylate cement D -All of the above ZN POLYCARPOXYLATE CEMENT NOT ADHERE WELL TO GOLD &PORCELAIN BUT STRONG ADHSVE BOND TO BASE METAL 1583. all expect one are present in Zinc phosphate cement liquid a- Water b- H2SO4 c- H3PO4 d- ALPO4 1584. the strength of Zinc phosphate cement is increase by increase powder/liquid ration a-true b-false 1585. Zinc polycarboxylate cement are better than Zinc phosphate cement in a- compressive strenght b- low solubility c- film thickness d- adhesion to enamel 1586. open margin in crown could be due to a- proximal contact b- failure to remargination of wax c- die spacer in the margin d- all of above

1587. to create space for cement a-die space b-roughen of metal +tooth c- investment expansion d- electro 1588. patient with sensitivity may be due to a-crack b-gap between tooth and restoration 1589. in soldering (PFM,FPD) grenish staining on porcelain without effect glazing this staining due to a-over heating firing b- flux c- investment contact 1590. during try in and rocking FPD ,what will do a- gap will fill with cement b- adjust tooth preparation c- adjust metal and disconnect and soldering 1591. cause of fracture porcelain bonding to metal a-thick opaque b-occlusion on junction of porcelain and metal 1592. most acceptable theory of bonding porcelain +noble metal a-formation of base metal oxide b-formation of noble metal oxide c-adhesion 1593. cement appear in radiographic like caries cannot distinguish from it a)calcium hydroxide include hydroxyl group b)zinc phosphate c)glass ionomer e)zinc polycarboxlate f)none of above 1594. When porcelain is baked against metal, it should possess a a) high fusion expansion. b) high fusion temperature. c) linear coefficient of thermal expansion less than, but close to,that of the metal. d) linear coefficient of thermal expansion greater than but close to that of metal

1595. Reversible hydrocolloids exhibit the property of transformation from sold to gel and gel to sold as a function of the a) concentration of the fillers and plasticizers. b) percentage of composition by weight of water. c) concentration of potassium sulfate. d) temperature. 1596. The principal feature of a sealant that is required for success is : a) High viscosity. b) Adequate retention. c) An added colorant to make the appearance slightly different from occlusal enamel. d )High strength. 1597. The retention of a pin: a) increase as the diameter of the pin increase. b) Increase as the diameter of the pin decrease. c) Decrease as the diameter of the pin increase. d) The retension of a pin has nothing to do with the diameter of the pin . 1598. In which of the following properties does a type IV partial denture gold alloy exceed a base-metal partial denture alloy in numerical value a) hardness. b) specific gravity. c) casting shrinkage. d) fusion temperature. Base metal alloy better than gold alloy: High strength and rigidity wrought wire clasp better than to gold alloy: More ductile and resilient 1599. Within practical limits, when the water/ powder ratio is increased beyond the recommended amount in mixing plaster, the a) setting time is increased. b) setting expansion is decreased. c) compressive strength is decreased. d) all of the above 1600. An over load of the mucosa will occur if the a) teeth used for replacement are non-anatomic. b) bases covering the area are too small in out line. c) bases covering the area are too large in out line. d) bases covering the area are overextended distally.

1601. Over extension of a mandibular denture base in distobuccal area will cause dislodgement of the denture during function as the result of the action of the a) masseter muscle. b) buccinator muscle. c) pterygomandibular raphe. d) lateral tendon of temporalis muscle 1602. The most important criterion for a gingival margin on a crown prepration is that a) it is dull knife edge. b) its position is subgingival. c) its position is supragingival. d) its position be easily discernible. 1603. A thin application of cavity varnish over the cut surface of a prepared tooth just prior to the cementation of a crown or a bridge with zinc phosphate cement will a) impede the seating of the restoration. b) insulate the tooth against thermal change. c) increase the possibility of thermal sensitivity. d) reduce the possibility of irritation of the pulp. 1604. Metal-ceramic restorations may fail due to fracture of ceramic material. This can be best be avoided if a) occlusal forces are minimal. b) the metal is not over 0.5 mm thick. c) the ceramic material is at least 1.5 mm thick. d) the casting is designed to reduce stress concentration in the ceramic material. 1605. For a removable partial denture, the lack of indirect retention would be manifested by a) tissue ward movement of the distal extension base of the prosthesis . b) movement away from the tissue of the extension base of the prosthesis . c) settling of the major connector of the prosthesis. d) lateral and medial movement of the extension base of the prosthesis 1606. The posterior palatal bar should be placed in a relationship ? a) to incorporate the hamular notch on each side. b) which is anterior to the junction of the movable and immovable soft palate.

c) which is posterior to the junction of the movable and immovable soft palate. d) extending from one hamular notch to the other hamular notch across the fovea palatinae 1607. most elicit painful area is a) gingival wall b) proximal wall c) pulpal wall d) none of the above 1608. success of implant are 55% in max and 60% mandible a) both true b) both false c) first statement true and second is false d) first statement false and second is true %<; 1609. most common types of porcelain in dentistry a)high fusion b)medium fusion c)low fusion d) all of the above 1610. .to select shade of porcelain: a) one light b) before preparation c)wet tooth d)shade guide must be wet which true -a,b -a,c,d -b,c,d -a,b,c,d 1611. Casting shrinkage gold alloy IV related to: a)thermal expansion b)coefficient of thermal expansion c)none of the above 1612. most common occur due to denture: a)sore spot b)angular chelitis

1613. reversible hydrolloide properties a) hydration b) dehydration 1614. radiation stent used to locate the properly position of implant. surgical stent make some modification to use as radiation stent a. Both true b. Both false c. First true ,second false d. First false, second true 1615. fiber composite FPD used for: a) posterior teeth b) patient have allergic for metal c) none of the above 1616. implant absolute used for a)patient take radiotherapy b)smoking c)none of the above 1617. patient take radiation need implant: a)wait 6 mouth b)cessation of smoking c)none of the above 1618. path of preparation depend on: a) esthetic b) contour c) pulpal wall ) ( 1619. Sequence of colour shade: a) chroma, hue, value b) hue, chrome, value c)value , chroma ,hue 1620. Crowns are to be placed on abutment teeth for a RBD, the a-wax pattern contours should be surveyed a-Crowns should be placed prior to surveying for clasp design b-Wax pattern should carved to the original morphology d-All of the above 1621. Before an accurate face-bow transfer record can be made, it is necessary to determine a-The axial center of opening-closing rotation peoples

b-The inclination of each condoyle c-The physiologic rest position d- Centric relation e-All of the above 1622. The non-rigid connector may be used in FBD in those cases involving a-Long span bridges replacing two or more teeth b-Short span bridges replacing one missing tooth where the prepared abutment teeth are not in parallel alignment c-Long span bridges opposing a mucosa- borne partial denture where the anterior retainer of the bridge strikes an opposing natural tooth, but the distal portion of the bridge is in occlusion with the removable partial denture d-Long or short span bridges where one of the abutment teeth has limited periodontal support 1623. Splinting of several teeth together as abutments for a FPD is done to a- Distribute occlusal load b- Facilitate plaque control c- Improve retention of the prosthesis d- Preserve remaining alveolar support E- Assure optimum design of embrasure 1624. Advantage of shoulder finishing line is: a- It provide enough thickness of porcelain b- Make the margin of restoration obvious in impression c- Assist the technician to locate the finish line on the die ) ( https://docs.google.com/viewer?a=v&q=cache:eurNBeuUSIcJ:www.medka au.com/album/download.php?image_id%3D393%26sessionid%3Dbb38c43 3df9fbcc5b572893fff9a236d+Advantage+of+shoulder+finishing+line+is&hl =ar&gl=sa&pid=bl&srcid=ADGEESgy_cZ0JGdRr5RdN6NQqgMXLLDR2Yf5d2 dnpxNrZagrnbSjjc7Wo1wq-mD915THsPDKLxEtgyUdkyrhizvguithJ86j4PURYuYfRThaexXXnZdiCQO4lBYrKKnIlceC1Au_jR&sig=AHIEtbQDu4REiiyOXKGQEb0iY31vFs7YZg&safe=on 1625. Regarding gingival retraction, the following are true except one is false, the false is: a- Retraction by electro-surgery is contraindicated for patient with cardiac pacemarker b- It can be done by chemical, mechanical, and electrosurger

c- Retraction cord impregnated in epinephrine is the best for all cases d- Retraction cord can't be used in severely inflamed gingival 1626. The most important property of cement for durable restoration a- Low co-efficient of thermal expansion b- Compressive strength c- Solubility in oral fluids 1627. The most retentive pin is: a- Cemented pins b- Self threaded c- Fictional type 1628. The least preferable abutment for FPD is: a- Rotated and tipped tooth b- Endodontically treated tooth c- A tooth with short tapered root and long clinical crown d- A tooth with little remaining tooth structure 1629. The use of amalgam post-core depends on a-Remaining tooth stracture b-Width of root canal 1630. Correct incisal and gingival color of metal-ceramic restoration, the color may be modified by 1- Use of stains 2- Use of stained porcelain 3- Re-firing at high temperature 4- Changing the light reflection by grinding and re-polishing a. 1 only b. 1and 2 only c. 1 ,2 and 3 d. 2 only e. 2 and 3 only f. 4 only 1631. In which of the following ways do surface characteristics of porcelain affect the perceived from of the final restoration? 1- A surface smoother than normal will give the impression of a larger size 2- Horizontal highlights suggest increased length 3- Vertical highlights give an illusion of width 4- Changes in contour and resulting highlights can be used to alter the apparent long axis tooth a. 1,2 and 3

b. 1,3 and 4 c. 1 and 4 only d. 3 and 4only e. All of the above books.google.com.sa/books?id=SdOOEbeyNVkC&pg=PA10&lpg=PA10&dq= ways+do+surface+characteristics+of+porcelain+affect+the+perceived+fro m+of+the+final+restoration&source=bl&ots=Avnk7fOnj&sig=jQvaBjOH_3VUoXLaDDV6pB2uHVs&hl=ar&sa=X&ei=qUsxT8jQG9CX Opj7mdsG&ved=0CCsQ6AEwAQ#v=onepage&q=ways do surface characteristics of porcelain affect the perceived from of the final restoration&f=false 1632. Which of the following contribute to the bonding of dental porcelain to metal casting alloys? 1- Metallic bonds 2-Chemical bonding 3- Adhesive bonds 4- Mechanical bonding a. 1 and 2 only b. 1, 2 and 4 c. 1,3 and 4 d. 2 and 4 only e. 3 and 4 only f. All of the above 1633. In an ideal centric occlusion, the mesio-facial cusps of the permanent maxillary first molar oppose the a-Central fossa of the mandibular first molar b-Mesial fossa of the mandibular first molar c-Facial embrasure between mandibular second premolar and first molar d-Sulcus of the disto-facial groove of the mandibular first molar e-sulcus of the mesio-facial groove of the mandibular first molar 1634. In children pulp damage is less frequent than in adults due to : a- Minor subluxation doesn't cut the blood supply. b- More hemoglobin content in children. c- Less nutritional deficiences. 1635. Which of these is used for gingival contouring: -PK.2

-PK.4 -Bard Parker 1636. To increase the retention of Full crown for a short molar : a.Use zinc phosphate b.Retintive vertical groove c.Shoulder finish line 1637. Pt infected by anthrax after examination doctor should disinfect with : a.12% cholorihexiden b.Antibacterial hand scrup c.Non antimicrobila soap Chlorine dioxide http://webcache.googleusercontent.com/search?q=cache:3A_0ls38kGoJ:ww w.epa.gov/pesticides/factsheets/chemicals/chlorinedioxidefactsheet.htm+ disinfect+after+anthrax&cd=1&hl=ar&ct=clnk&gl=sa&client=firefox-a 1638. 8 y old pt sollowed 10 ml of 1o% f what the immediate action : a.Ingest milk b.Hospitalization 1639. Decrease the polymerization shrinkage of composite by: a. Incremental placement with increase time of curing b. Incremental placement with high intensity light cure ) ( webcache.googleusercontent.com/search?q=cache:t0UDsudqXWYJ:www.sciel o.br/scielo.php%3Fpid%3DS151774912003000300013%26script%3Dsci_arttext+Decrease+the+polymerizati on+shrinkage+of+composite&cd=2&hl=ar&ct=clnk&gl=sa&client=firefox-a 1640. Pt with sialolithiasis we want to take x-ray with ordinary film in order show the stone we should a. Take x-ray in the same way as usual b. Increase the intensity of x-ray http://en.wikipedia.org/wiki/Sialography 1641. Retrograde filling is indicated In what condition!!? a.Max central incisor with Good filling with 9 mm radiolucency b.Max pre with post and core buccal root with 4 mm short filling and RL at the apex but the palatal root with good filling

c. 1st molar with MBR and DBR short filling and platal root with fracture instrument 1642. 2 walls defect in perio what is the best graft to treat this defect : a. Cortical freeze dried bone allograft b. cancellous freeze dried bone allograft c. All are the same 1643. test for determinig the efficiency of sterilizing agent is : a. Fungi b. Virus c. Bacteria d. Bacterial spores 1644. which of the following has doesn't have damage effect on hand piece A. Apply great pressure during use B. Infrequent mosturization C. Fall down of the head of the hand piece mosturization 1645. pt with lesion in the post of the tongue we want to take excisional biopsy how to pull the tongue forward: A. menesota tongue retractor B. mirror Handle C. towel clip http://webcache.googleusercontent.com/search?q=cache:aEDpAaDdq8QJ: oralcancerfoundation.org/dental/screening.htm+instrument+used+to+pull +the+tongue+forward&cd=1&hl=ar&ct=clnk&gl=sa&client=firefox-a 1646. pedo pt with extremely ve behavior to restrain the extremity: A. use mouth prope B. belt C. Board 1647. during anathesia what's true: A. The needle should be inserted before cartridge B. The needle cap is inserted before the stopper C. Excessive force should be applied to allow insertion of the cartridge into the harpoon 1648. Vasoconstrictors are a) lidocaine b) adrenaline

d) sulpha c) all of the above 1649. When we extract roots we start with a) forceps b) periosteal elevators c) elevators d) needles 1650. In case of bad odor in the entrance of the pulp we a) extract the tooth b) remove the pulp c) R.C.T d) none 1651. Insulin is a medicine for a) hypertension b) cardiac diseases c) diabetes d) decrease in platelet count 1652. In case of infiltration we give a) anti inflammatory b) antibiotic c) a and b d) none 1653. For the mandible we prefer a) infiltration b) block c) intraossous d) none 1654. Phosphoric acid is applied for a) 10-20 seconds b) 30-45 seconds c) 20-30 seconds d)none 1655. The lower teeth are supplied with a) maxillary nerve b) infra orbital c) mandibular d) all of the above 1656. The main cause of inflammation of gingiva (gingivitis) a) smoking b) calculus c) bacteria d) fluoride 1657. The pregnant woman is suffering from

a) gingival recession b) pregnant granuloma c) Periapical abscess d) none 1658. Muscles of the tongue are a- 17 b- 18 c- 19 d- 20 1659. Sterilization in autoclave a- 20-30 minutes at 121 c b- 2-10 minutes at 134 c c- a+b d- none 454 53 48 http://en.wikipedia.org/wiki/Autoclave 1660. Cross bite means a- upper teeth occlude inside the lower teeth b- upper teeth occlude outside the lower c- a and b d- none 1661. The efficiency of the autoclave is decreased due to a- sterilization without cleaning the instruments b- over loading c- dry blood on the instruments d- all of the above 1662. The food which build new cells a- carbohydrates b- proteins c- fats d- minerals 1663. Brushing of the anterior teeth from the lingual side is a- vertical b- horizontal c- oblique d-none 1664. When we delay the cleaning, sterilization of instruments we put it in a- holding solution b- sodium hypochlorite c- a+b d- none

1665. Before doing vitality pulp test , the tooth must be a- moist b- dry c- moist or dry not affect d- none 1666. ttt of trigeminal neuoralgia all except -eostigmene -nsaids -propanolol -amiphityline http://en.wikipedia.org/wiki/Trigeminal_neuralgia 1667. one plane labial reduction lead to overcounterd crown -improve retetion occlusal disharmony 1668. Patient 3 year old had injury in the primary teeth ,the permanent teeth are rarely under go to: -Micro abrasion in the enamel. -Discoloration. -Dilaceration -Partially stoppage of the root formation http://en.wikipedia.org/wiki/Turner%27s_hypoplasia 1669. galgavin - vermilion formula used in: calculation of flouride in water according to temperature 1670. at age four years A child took antibiotic. Which teeyh are affected a. central and lateral incisor and 6th b. central and lateral incisors only

1671. the punch hole in the rubber dam for a child is: a. small b.medium c. large d. the hole size isn't important http://webcache.googleusercontent.com/search?q=cache:S1K_W1VgyXcJ:

www.dentalcare.com/en-US/dental-education/continuingeducation/ce379/ce379.aspx%3FModuleName%3Dcoursecontent%26PartI D%3D2%26SectionID%3D1+rubber+dam+size+for+a+child&cd=6&hl=ar&ct=clnk&gl=sa&client=firefo x-a the size depend on which tooth we need to treat

1672. the reason of separation or fracture of opaque layer in ceramo metal crown in gold a. technique not using conditioning material on opaque layer b. contact of occlossion on opaque layer 1673. Patients with high caries activity has low PH and the PH fall on the lower level a. Carbohydrate retained to the tooth has prolonged effect b. After rinsing by 10 % glucose the PH fall within 2 3 minutes below the critical level of PH and remain for about 30 50 minutes c. Fall the PH below the critical level of PH , the enamel can be remineralized 1674. birfurcation involvement ... what to do (non surgical perio)? a. tissue regeneration

1675. ptrygomandibular raphe: a)the needle should be inserted origin from infratemporal fossa b)between hamular notch and coronoid process c)all 1676. patient had Intrinsic stain in anteriors you decide to put full veneer crowns porcelain fused to metal had Irreversible pulpitis ,you decide to make RCT and make access opening from palatal side .WHAT is the most appropriate filling to do will not disturb the crown cementation ? A) reinforced cement and any appropriate filling b) Towl ___ c) pin amalgam seal 1677. Rubber dam using in children: a)Just scientific role b)Improve safty c)increase visualization

1678. when we can put composite restoration in recently bleached tooth : after 1 week 1679. *in prefabricated twisted pin in amalgam it should be go equaly into dentine alloy : a. one mm b.two to three mm c. five mm d. only in enamel 1680. Inferior orbital fissure located a. lateral wall & floor of of orbit b. the medial part of orbit c. between the two wings of sphenoid d. 1681. in standard instrument for proper work the cutting edge should be: a. perpendicular to the handle b.parallel to the handle c. acute angle d. inverted angle 1682. Doing CANTILEVERS, we consider all of the following EXCEPT: a) small in all diameters b) high yield strength c) minimal contact d) small occlosogingival length. 1683. which of the following not considered type of healing of fractured root a. healing by bone deposition in proximity b. healing by cementium deposition in proximity c. healing by cementium and soft tissue in proximity d. healing by soft tissue inflammation in proximity 1684. 6 years old came to u with carios lower molar sinus drenage a. Pulp pathosis 1685. symptoms of conginital syphilis deafness, notch incisor bolbous cuspless molar oral pigmentation

1686. child with white patchies on facial surface of anterior teeth , the treatment include 1687. treatment of grade 2 furcaiton involvement 1688. permanent tooth had a trauma what the cause of hyperemia

1689. Stephen curve for plaque a. rapid drop under critical ph 5.5 follow by slowlly raise

1690. For the prevention of mercury toxicity in the clinic put it in 1. Water 2. Sodium chloride 3. The appearance of radial 4. Fixer of radial 1691. 21 year old pt has submandiulaspace infection swelling in 36 area and 38 is missing on radiograph he has radioloucency extend in 36 37 38 area with septal tubercula and root resorption Kerato cyst Ameloplastoma Dentegerous cyst 1692. Pt has high mastication forces has caries onposterior teeth and he want only esthetic restoration a. Composite with beveledmargins b. Composite without beveled margins c. Light cured GI d. Zincphosphate cement

1693. Anomalis during initiation and profilration of tooth germ will lead to a. Amelogenasis imberfecta b. Dentinogenasis imberfecta c. Dentinal dysplasia d. Oligodontia

1694. *pt. come with deep carious lesion uponradiographic examination show wellcircumscribed radio-opaque mass 16 withradio-leucent boundery and no other symbtom is: -odontogenic tumer -ostitis -benign cementoblastoma

1695. 8 year Patient came to your clinic has impairedhearing, upon


examination his mouth you found copper color lesion , notchedincisor and mass on the occlusal surface of the molars . this patient has: -Congenital syphilis -Gardner's syndrome

1696. child 8 years old hearing loss , flush aroundhis mouth and notches in
incisors and bolbuos molars .what is yourdiagnosis? a)gardner syndrome b)congenital syphilis***

1697. 2Child 2 year old came to your clinicwith his parents ,he has bony
lesion bluish-black in color , the most probablediagnosis is: -Melanotic neuroectodermal tumor. -Gaint cell granuloma. -Ameloblastic fibroma.

1698. Autoclaving technique is depending on :


a. Dry heat. B. Steam heat*** c. Chemicals.

1699. *pt. come complaint of pain an discomfort inupper second or first molar upon
examination reveal that 15 these tooth isabutment to four-fixed partial denture and on radiograph all thing is normal ... discomfort is likely because: -fracture of connector -vertical rootfracure -mobility of tooth -loose of retainer onabutment

1700. in the mouth there is 12 permenantteeth 12 primary by the age 9 years

1701. radiographicallyinterdental bone appear perpendicular to the CEJ _ if


the CEJ of many number ofadjacent teeth are not in the same level .. the interdental bone will appear orcreate angulations>>the both statements are: a- first true b-both true c-both false d-second true

1702. degreeof disinfection in ethanol alchohol 70% 1703. pathfinder survay for plauqe index andperiodontal health piolt 2 subgrob
only national all grops

1704. best luting for crown to be retentedis resin then after resin resin reinforece gic
increase retention decreasemicrolackage

1705. .35What is incorrect about veneer


facial Windows veneer are moredifficult to seat during final cementation

1706. errosion acid induse tooth wear 1707. complete reepithilization after surgary take month 1708. silogram contraindicated for iodine sensetive pt allergy we can use for them
mri .9121 : amalgam and GIC .9192 Enamel will absorb the fluoride

1711. anterior under


cut + under in post area near to tuberosity Answers : -1remove under cut from anterior are -2remove under cut from post area -3remove under from post. Area & make relaine in denture for anterior area --4remove under from ant .Area & make relaine in denture for post area

1712. Blade of pd instrument should be:


A- perpindicular to long acsses B- parallel to long acsses C- perpindicular to shank

Das könnte Ihnen auch gefallen